NBDE II remembered questions, late June 2012 1. Hypertelorism + midface deficiency _ beaten metal appearance  Crouzon’s syndrome 2. NUG which one except fetid odor, fenestration of gum * , rapid onset, poor oral hygiene 3. Xray target made of tungsten 4. Group with most untreated caries in permanent dentition  blacks 5. Something about pain perception in different cultures  different threshold, different perception, stimulus awareness ( not sure) 6. Picture of leukemia 7. Median rhomboid picture most associated with  candidiasis 8. HIV cell numbers are important, t4 is 30 most immunocompromised 9. 8 months Pregnant patient fell unconscious what to do  turn her on her left side 10.Primary etiology of in teenagers and pregnants  Plaque 11.Questions about caries factors 12.Management of angry patient 13.Rapport most related to the concept of empathy 14.Carbamazepine used for both epilepsy and management of pain of neuropathic origin 15.Ginseng interferes with aspirin 16.Acetaminophen doesn’t have anti-inflammatory properties 17.Prilocaine methemoglobin 18.Picture of lingual varicosities 19.4 year old lost both mandibular first molars bilateral band and loop 20.7 year old lost both mandibular Es  lingual arch 21.A lot of questions about management of pediatric patients 22.How much fluoride is in 1 liter of 1 ppm water in mg 23.Garre's sclerosing osteomyelitis onion skin apprearance ( also seen in ewing sarcoma) 24.Average flouride in community waters  1 ppm 25. affect which teeth severely :maxillary anterior 26.Definition of implant analogue 27.Average/ normal amount of crestal bone loss on implant after one year  no more than 1 mm 28.Perio bone defects, something about hemiseptem, look it up 29. picture 30. target shaped lesions on hand picture 31.Referred pain of heart attack in jaw area 32.Which one of the following resembles chancer in syphilis  squamous cell carcinoma, primary herpatic gingivostomatitis , etc ( don’t remember all choices) 33.Panorama , identify structure lateral nasal wall 34.Panorama identify structure coronoid process 35.Panorama identify structure shadow of soft 36. Bite wing, obliterated root, short, bulbous crowns dentinogenesis imperfecta 37. Patient management of patient arguing that your fees are too high 38.Patient with will most likely have wear facets 39.Features of cemento osseous dysplasia ( 2 questions) 40.Trigeminal neuralgia can’t remember 41.Treatment of primary central intruded by 5 mm in 3 year old 42.Treatment of permanent central intruded for 1 mm in 8 year old 43.If tooth doesn’t have enough band of keratinized gingiva  don’t do gingivectomy 44.Most important factor affecting success of posterior composite  technique and case selection 45.Composite postoperative sensitivity related to polymerization shrinkage and cusp flexture 46.MOD inlay and pain on biting on French bread  M-D fracture 47.Management of cusp fracture without exposure 48.Glass ionomer cement which one is not true less soluble than resin cement 49.Best cement for onlay  resin cement 50.Xray for 51.Parkinson patient facial features 52.Treatment plan for Alzheimer patient based on preventive and alleviate pain 53.Needle tract infection after ID block will spread to which space 54.A lot of questions about fascial spaces and spread of infections 55.Treatment of necrotic first molar in 8 year old  apexification 56.Turner tooth definition of one tooth 57.10 year old have black central incisor, most probable cause  trauma 58.Definition of parulis an elevated nodule at the site of a fistula draining a chronic periapical ABSCESS 59.Chronic suppurative infection but no sinus tract  drain thru periodontium 60.Periodontal apparatus includes all but  gingiva 61.Suppuration most associated with  62.Diagnosis, asymptomatic tooth with periapical radiolucency, no response to vitality test  periapical granuloma 63.Management of patient who wants dentures but has moderate gag 64.Several questions about epidemiological studies 65.Nikolsky sign  66.A questions about arbitrary facebow, don’t remember details 67.If you want to increase vertical dimension for a patient you need  another centric relation record 68.Ortho treatment before perio surgery for gum, might change shape of gum line ( not sure) 69.What antibiotics can be prescribed for patient with of non odontogenic origin  amoxicillin and clavulanate 70.Infective endocarditis prophylaxis (2 questions) 71.Intravascular injection of local anesthesia most probable in  ID block 72.Modeling behavior management technique in children ( 3 questions) 73.How to decrease prenumbra in Xray films  shorter source to object distance 74.Which drug is the most potent bronchodilator 75.Asthma  either inspiration wheezing or exhalation wheezing 76.Scopolamine  for motion sickness 77.Benadryl (diphenhydramine) uses 78.How to treat transverse discrepancy in 40 year old  Rapid palatal expansion, W wire, Quad helix or segmental osteotomy of maxilla ? 79.Morphine bind to receptors of  enkephalines 80.Opioids are contraindicated with --? MAO inhibitors 81.Epinephrine contraindicated with nonselective beta blockers 82.Brachial cyst  located along anterior border of sternocleidomastoid muscle 83.Insurance adverse selection policy means takes only low risk patient beneficiaries ? 84.Green and orange stains on teeth means  poor oral hygien 85.What type of hemoglobin does sickle cell anemia patient has 86.Which of the following is not a complication of sickle cell anemia 87.Best brushing technique  Bass 88.Metoprolol is selective B1 blocker  is it contraindicated with epinephrine 89.What is the normal salivation rate in ml/min 90.Which forcepts can not be used for extraction of premolars  #150 , #151, #23 , #286 -opend apex tooth after truma no periapical no EPt --- wait -over triturated amlgam ---- short working time -vertical x ray errors ----- distortion -horzantal anglulation 10 - Frankfort used in ortho not in CD Bilat RPD moving up ward ---- lack direct support -lefort I sinus only lefort ii naso palatine I think -persistent 5 mm pocket good oral hygiene whats the next step ?! -steriod dosent cause ----u have to know the side effects -giving pt atropine cause all except---- know the action -definitions---- agonist\ partial agonist\ potency \efficacy Metal thermal expantion more than porcaline -increased liquid in MMM --- high shrinkage -SNA SNB angles ANB class I \II\III - dark tooth with amalgam --- corrosion products -stripping perforation in max MB canal ---- distal surface -2ed most md fracture --- ?angle -2end most dentist x ray exposure --- pt scatter \wall scatter \air?!? -best tooth brushing method?! - repeated incisal veneer fracture -- edge to edge -the least effective in tx deep jv periodontitis ---- occ adjustment \ antibiotics \ flap surgery \ ?! -autism pts ?!?!? MR \ unresponsive \ …. ….. ?!? Short term memory loss --- dementia not Alzheimer I think that -deaf pt -- talk direct with pt and then let the translator explain that to pt -most difficult impression material to remove from pt mouth --- ether -gutta percha not do --- \ anti septic \ adaptation \ 3D sealing ?!?! -multiple radiolucency – dentinal dysplasia -primate space - n2o know every thing on it ---- contraindicated with -4gry radiation ---- erythema -HBs + ----- chronic carrier -co\cr know every thing very imp - incisor turner tooth when the trauma happened --- 4 month to 4 yr -most common nasal sinus fungal infection ? mucormycosis - pathophesiology of dry socket --- fibrinolysis -upper infection --- cavernous infection ,, dangerous triangle valve less veins -tea spoon how much ml nseet !! 15 ml !!? - signs of cavernous thrombosis -not causing gingival hyper plasia --- digitalis - clindamycin pro --- 600mg -most heamatoma --- PSA - most intra arterial --- ID Replacing amalgam with gold ---better outline only -sialolith --- sub md gland -tooth fracture below alv bone level --- extraction - pain in ear ---- md molars - retention except --- depth of the under cut -sever CNS o2 deprition --- miosis + light reflex “ I don’t know “ - black male --- periodontitis - acute periapical abscess ---- drainage + occlusal adjustment -Q about drugs and their types except --- type III cant refilled on the phone “ the answer “\ eshi dentist number \ raje3hom in pharma -drugs cause extrapyramidal ++ ----??! - LA mg ---- % * 10 *ml = -water with 1 ppm pt drink 1 lither how many mg --- 1 - 1 pmm = 0.0001 Gorlin syndrome goltz syndrome down syndrome know them well ---dental feature -intensity = 1 \d^2 - x ray sharpness depend on --- Balancing contact and working contact and the interferances --- which surfaces -creater defect on the x ray is --- under estimated - palatine toris removal ---- interfere with palatal seal -incisal reduction 2 mm -most common ant composit replacement---- discoloration - fractured porcelain cervical margins repair --- micro etch ??!? I don’t know - warfarine – vit k - INR 2.5 --- extract -sealed caries --- arrested -root caries best by --- clinical detection -245 bur and 330 – 245 longer I think - laryngio spasm –symptoms “ stropid nseet sho el kelmeh :P -MRSA --- vancomycine -kelles syndrome know every thing about it ---- only lower ant teeth best mnage bilateral balanced occlusion -short clinical crowns how do u increase retention and resistance form ?!?! ---- buccl groove wall a proximal boxes ?? -metho trexate ---- foliat like ?!?! -incresed foliat papilla with ?!?!?! ---- hairy tongue \ lymphangioma \ ?!? - 5 mm probing 2mm from CEJ ---- pocket 2 mm -alterd cast teqnique --- more functional movement to RPD -pemphigous \ \ I herpetic gingivostmatitis\verroca vulgaris\ aja 3lhom as2eleh w 9war - cII posterior composite most affected by ---- size of preparation + technique I think \ tooth size \ resin type ?! -insurance \ 3rd party \ capitation \ epidemiology \ billing habal el pt management - ASA classification -pregnant need NSAD – acetaminophen - 1 dose of aspirin last for --- week - thermal emmition in x ray tube – the filament I think -icreased kvp --- increased energy -most needle injury in recapping -drug that cause asthma similar to aspirin ?!!?! --- ibuprofen I think -the highest heat with --- dry heat -pregnant coma management --- I think position the pt on his left side decreased vena cava pressure -tx plan sequence – I think --- emergency –tx the disease – evaluate – definitive tx- maintenance - EpT – indicade responsive pulp from unresponsive and doesn’t reflect the pulp state normal or diseased - case series case control cohort cross sectional Q ?!?!?!  - proximal caries --- under the contact - most reduction in caries ---- smooth surfeces or pits and fisser I don’t know I put it smooth surface -best esthetic implant the connection between implant and abutment --- below the gingival margin \ at the bone margin !?!? I don’t know - prevent crestal bone resorbtion in implant by ---- small abutment \ abutment implant interface at the bon \above the bone ?!? -periapical --- showing radio opacity in the upper jaw I think the best answer was the nose ?!?! -photograph for sever gingival bleeding and enlagment within 5 weeks – leukemia -photo for a tongue with upthous -cross bite 3mm md deviate to ---- affected side ?!? I don’t know \ un affected side \ -the I cause to restore I tooth ---- space maintenance -contraindication for gingivectomy -adv of periodontal pack ---- comfort only -

Biological width Pedo: How much for max NO?palate and lip cleft disease rateectodermal dysplasiadown-NSAIDindirect retainerPT normal valuebone graftvalue,hue,chromaII amalgam have overhang, why?class III cavity in two proximal teeth, one big,one small, fill which one first bzp antibiotics hla wt kg kids n how much la narcotics antifungal endo pedo anug herpetic gingovitis ameloblastoma- dd radio - extra, intraoral mucus retnsion cysts salivary glands impression materials crown root ration pfm- where fravture- ?porcelain malleability, ductilty cements inlay,onlay- mod- fracture- ga- nitous oxide extractions occlusion- reduce where rpd- class, guiding plane cd- muscle molding pps anticoaglants ceph band n loop incidence,prevelance,sensitivy, specificity pics

1denti.imperfecta 2.lympho epithi.cyst 3.mucocele 4.cemetoblastoma 5.facial palsy 6. 7.migratory glossitis 8.candidiasis

2.pt with inlay have pain during bitting..no radiogrphic evidence-(ans)m-d cusp fracture 3.cracked tooth with more symptom to---cold,heat,pressure 7.most common area of fracture in children---symphysis,condyle,coronoid 8.apical root closes---21/2-31/2,4yr,5yr 9.tooth with most favourable prognosis---small internal resorption 10.face divided vertical by---3,4,5; Horizontal is 3 (it should 4 planes that divided it to 5 area) 11.frankfort horizontal line---(ans)porion to orbitale 12.occlusal rim parallel to----ala tragus line ( camper's line); 13.sterilization most destructive to carbide instruments----steam heat,dry heat,ethylene oxide,cold chemical dry heat does not Corrode or dull instruments. bacillus spores are the benchmark for sterilization I think it is Clostriduim Botulinium) First aid page336 Mycobacterium is the benchmark for disinfection 14.sterili cause rust----steam heat,dry heat, unsaturated vapour(does NOT rust or corrode) 18.orange stain is important ---to change chroma 19.which procedure is most unsuccessful in primary tooth with deep caries---- direct ,indirect,pulpectomy,partial pulpectomy,pulpotomy 21.resorption of bone takes place in which direction after extraction----downward inward,downward outward,forward inward (something) 22.which procedure least likely to produce bacteremia--extraction,non surgical endo,oral prophylaxis 24.which gracey curet is used for the mesial surface of distal root in max tooth---11-12,13-14 25.which determine energy level of photon in xray---kvp,ma 26.which structure is most radio sensitve----hemopoitic bone marrorw 27.revesal occlusal plane---chin tilted too upward 29.which is most important for diagnosis of maxillary sinus xray---- occlusal,panaromic,MRI 31.effect of xray---genetic mutations 32.collimation 34.tranillumination method is most useful in carious diagnosis ---anterior proximal,posterior proxmal USE OF ZINC CHLORIDE IS NOT RECOMMENDED=necrosis Aluminium chloride is hemodent, most commonly used Epinephrine is not used due risk of increasing BP in hypertensive pts. 38.procelein lamnated veeneer fixed by resin show black margin in 2 days cause--- porcelein break,resin wears off??? 40.active part of ultrasonic 41.torche available fungal agent---clotimazole 49.U shaped radiolucency in max molar---zygomatic process J shape radiolucency - 50.pt says i dont have time to stop smoking--- contemplatory,precontemplatory,denial,action 51.pt asking more about mask,glove which is NOT correct responds---dont worry about it we will take carry of it,expaining about each precautions using in clinic 52.using ATM card of elder is not applicable but some suitation is not under consideration---both true,both false.1st true 2nd flase 53.elderly people abuse question --under reported 54.adult child can decide a treatment plan of his old patient his parent in what suitation---when he is payig money,when he is impaired either ear or eye,patient want his decision??? 56.dentist do the treatment for 2 crown but the insurance company pay the money for one crown wha t is it. bondling 57.more questions from - cross control study 58.case series study,case study 61.which area most difficult matrix adaptation--max mesial 1 prem 62.which impression material has good wettability---Hydrocolloid>Poly Ether>hydrophilic Add Sillicone>Poly Sulfide>Hydrophobic addition silicone and condensation silicone has same wettability 64.uses of chlorhexide--reduce plaque accumulation 66.pt has composite restoration with severe pain with localized swelling---- Incision & Drainage 67.action of sodium hypochloride- Antibacterial 69.most crucial in replantation----time managment 72.ectopic eruption of man 1st molar in relation to pri man 2nd molar cause some resorption -managmant---extraction of 2nd molar,separation, disking of 2nd molar 73.calcification of premolar tooth at birth----NO

74.patient with tooth 12 of pri and 12 of perma---8 1/2 75.mouth breathers have facial feature of incompetent , convex profile, narrow palatal vault, bilateral cross bite 76.most common cause of frequent urination during 3 trimestor--pressure of uterus on bladder,gestatory diabetic 77.facial profile of class 2 ---convex, Class III is concave 78.normal biological width---????? 2mm Biologic width is the distance established by "the junctional epithelium and connective tissue attachment to the root surface" of a tooth.[1] This distance is important to consider when fabricating dental restorations, because they must respect the natural architecture of the gingival attachment if harmful consequences are to be avoided.

79.tooth after acid etching show decalcification at the area but did give the appearance of frosty why?more fluoride in toothe,etchent not removed correctly...??? 80.beveling in acid etching composite use more surface area. 81.distobuccal flange of denture determined by---masseter 82.removing the mylohoid ridge have the common error in--lingual nerve damage 83.cheek bit due to---facial placement of tooth, reduced horizontal overlap,vertical overlap...??? if it is mandibular facial placement of teeth, then it is the option. 84.ant flange in max teeth---esthetics 85.facebow transfer for recording---max with hinge axis positioning 86.post po managment after denture delivery pt has pain in crest of the ridge---defect in post occlusion 87.placement of rubber dam affect the colour selection by----black background,reflection of light.. 89.know about the tooth reduction for various materials 90.Tooth set in 20 degree for balanced occlusion...but adjusted to 45 degree wat is need to corrected for balanced occlusion--compensatory curve 91.dna probind in perio pocket --for bacterial specific 92.tooth with endo treated and post with crown have pain after several days esp during biting and cold..vertical root fracture 93.use of indium with alloy is mainly to provide chemical bond with porcelein 94.classification of pontic mainly depend on their relationship ---to the ridge 95.know about pier abutment and cantilever 96.physiological rest position 97.working side and non workingside inter fence 98.bennett shift mainly on --lateral movement or working side 99.narcotics mainly contraindicated in ---MOA inhibitor 100.severe alcoholic now recovering need 24 tooth extraction which test needed---- INR,CBC 101.easy to extract impaction in max tooth---distoangular 103.property of interocclusal recording material----low resistance to jaw closure 104.incisal guidance mainly depend on ---hori and verti overlap 105.occlusal truma symptoms - fremitus is the best test to test TFO, vascular - blood clots in PDL, , widening of PDL 106.erythema in the palate of the patient wearing denture...infla papillary hyperplasia 107.after having denture (partial) the pat have pressure for few day.later it subside--- ,pressure of clasp, pulp damage 108.which dont need replacement or repair--recurrent carious at margin,pulpoaxial line fracture,ditched restoration 109.pt has restoration shows demarcation b/w restoration, which is fracture in middle of restoration wats next step---look for further fracture line,replacement of restoration...??? 110.sensitivity following composite restoration in post most comman cause---???due to resin,polymerization shrinkage in margin,shrinkage floor...??? 111.without indirect retainer---outward displacement of distal extension base 113.flexiblity of alloy depend on all expect----composition,taper,undercut (ans) 114.epithilium of free ging graft----degenerate 115.distractive osteogenesis defer from osteotomy by..??? distractive osteogenesis is a surgical process used to reconstruct skeletal deformities and lengthen the long bones of the body. An osteotomy is a surgical operation whereby a bone is cut to shorten, lengthen, or change its alignment 116.signs of gingivitis 117.bacteria in healthy mouth---facultative gram positive anerobic bacteria 118.which of the following is not true about local agressive periodontitis----affect less than 30%,treatmt scaling and systemic antibiotics, genetic 120.disease with ---pemphigus and cicatrial pemphigoid 121.lateral perio cyst common location---bicuspid lower 122.some measurement about attachment loss 123.primary reason for replacing og overhanningrestoration---interfere in plaque removal 124.which one is common in pregnancy and in normal condition-- 125.best longterm care after perio treatmnt---self,professional..??? 126.which type of interleukin in most common after perio disease--1 128.if implant with width of 4 is used what should be the bucolingual width of the ridge----6 minimum Vertical height of bone to place implant – 10 mm minimum Width of bone is 6mm minimum distance of apex of implant From nerve - 2mm platform of implant from adjacent CEJ - 2-3 mm between implants 3mm between implant and tooth (height of coutour) is 1,5mm Mini implant is 2.4mm When there is FPD from natural tooth to implant, the max stress is concentrated on the SUPERIOR PORTION OF THE IMPLANT. RPD: Retention: Direct retainer , and indirect retainer Stability: Minor, proximal, ligualul plate, denture base Support: Rest, major, denture base H2O2 - less than 10% OTC H2O2 - 30% (superoxol) used in in-office bleaching Carbemide peroxide - 15-18% tray bleaching (commercially available in syringes) 129.most acceptable root sensitive theory---hydrodynamic 131.---normal wear 132.incisal beveling 133.something about festooning bone,margining bone,flattening bone(option) 134.post evalution after perio therapy---3wks 136.pt with diabetic having sedation IV and LA---ask the pt to take high calorie food with insulin,low calorie food which inusulin 138.mucosal graft epithelization by---connec tissue from underlying tissue(recipient site) 140.lefort frac 1 associated with- what fracture--nasoethmoidal air cell,frontal sinus,max sinus,mastoid air cell Le Fort II - separation of the maxilla, attached nasal complex from the orbital and zygomatic fractures Le Fort III - Nasoethmoidal complex, the zygomas, and the maxilla fromt the cranial base which results in craniofacial separation 143.2 question about apexification ...pt 8 without root closure 144.symptoms of sjogren all expect one---sicca,xerostomia,arthirtis, (another option is ans) 145.acute gingistomatis virus associate with---chicken pox virus(herpes virus) also 147.acylovir---in some viral infection 148.most common cause of failure of in restoratn in primary tooth---cavity prep,moisture contamination... 150.fusion---two tooth join only by 151.while giving inf inferior alveolar injection the pathway from?contra lateral premol,contra canine,contra molar,ipisi lat premol 153.how to split the tooth using bur tech of tooth removal---spilt buccal and lingual upto furcation.. 155. constantly exposing the pt to get from the fear factor is---desensitation Fear decreases pain and anxiety increases pain 158.most common site of herpes--attached gingiva 159.scopolamine-commonly used for motion sickness Anticholenergic drug The drug is used in eye drops to induce mydriasis (pupillary dilation) 160.prostaglandin inhibitor cause all expect---increase grastic mucous, PG:decrease gastric acid and increase gastric mucous 162.unconsicous most commonly---psychogenic 163.in asthmatic patient===nsaid contraindications - NSAIDS cause bronchospasm. longterm asthma give corticosteroid 164.pt with often---have 4-5gy of radiation therapy 165.pt with bisphosphanate 3ml IV for 3 yrs,have carious and unrestorable tooth managmnt---stop regmen 1 mnth extract,do non invasive, endo treatmnt. bisphosphonate has half life of 6 months, it is advisable to wait for atleast 6 months before invasive procedures http://www.ada.org/prof/resources/topics/osteonecrosis.asp 167.most common side effect of N2O2---nausea and vomitting 169.laryngeal obstruction in---anaphylatic shock 170.steriod supplement indicated in pt with---10mg with 1 yr,10 in 2yr 171.erosive crust in skin with target----erythema multiforme 172. Diazepam action in GABA both relaxing, anti-anxiety and anti-convulsive effects 173.common malignant potential---paget's Caractheristic of Paget: maxilla>mandible.Lion-like facial deformity.Teeth demonstrate extensive .Development of malignant bone tumor,usually an osteosarcoma is recognized complication of Paget,frequency of osteosarcoma is 1%,mostly long bone.NEVILLE 178.absence of primary tooth---premant eruption slow 179.something about visible light curing - 400 - 499nm 181.for caries restoration---prepare large 1st,fill small 1st 182.advantage of gold on oclusal surface,porclien in facial surface----conserve tooth struc,minimal rsduction...???? 184.interaction of proponolol with epinephrine is best described by...???? 185.drug most likely cause xerostomia---lithium(anti psychotic) others that cause ca channel blockers, nefidipine and anticonvulsants. 186.Pt have swelling after extraction,he is under penicillin therapy---other family of antibiotic--drug of choice ????tetracycline,erythromycin???? 187.beta lactamase---inhibit the action of penicillin--- Methacillin is an example resistant to beta latamase 189.both drug have same intrinsic effect and different receptor affinity---same potency,same efficacy 190.pt is addicted to oxycodone which contra indi---codiene,pentozocaine 192.all drug induce drug reaction expect---teracyclinewith antiacid 193.antipsychotic with irreversible side effect----Tardive dyskinesia ---Sudden purposesless movement(parkinson like disease) 194. lantaprost indication----Glaucoma(Latanoprost) 196.large radioopaque lesion in carious affected tooth---condensing ostitis

199.syndrome with ,micrognathia,cleft lip,mandibular hypoplasia--- pierre robin syndrome 200.granulomatous lesion---crohn's disease(ans),ulcerative colitis 204 .patient with labial lesion or 7 days,similar kind for lesion wk before---recurrent apthous 205.supparative lesion---actinomycosis

Hypercementosis day 2

Localization of mesiodens is done with occlusal Occlusal in 2 angulations, diagnostic is 1 occlusal, Neuropraxia: It is a transient episode of motor paralysis with little or no sensory or autonomic dysfunction. Neurapraxia describes nerve damage in which there is no disruption of the nerve or its sheath St John's wort is today most widely known as an herbal treatment for depression,may work against gram negative bacteria,ADHD Pt is a child and is diabetic undergoes hypoglycemia in the chair if conscious give him orange juice ( unconscious give him 50% dextrose IV)

Type I Diabetes leads a) Aphasia b) Ataxia c)Blindness d)Deafness

Amantidine is used as Anti Viral

Strawberry tongue seen in scarlet fever

Prophylactic treatment for peacemaker – no premedication required

Prophylactic treatment for Prosthetic heart valves – premedication required

Pano picture mostly pointing towards earlobe

LA calculations 4.4mg/kg

One carpule of 2% xylocaine has 36mg

Primate spaces are seen between maxillary Lateral incisors and canine’s mandibular canines and molar

Which of the following Medication is not by cell wall synthesis?

A) Penicillin B) Amoxicillin 3)Vancomycin 4)Azithromycin - inhibits protein synthesis CLEAN (50s):Clindamycine,erythromycine,azithromycine,chloramphenicol,clarithromycin e TAG(30s):tetracycline,aminoglycosides,gentamycine,streptomycine Other antibiotics that Inhibit protein synthesis:Azithromycin,lincomycin

Lot of questions on public health case studies

Prospective cohort studies: A prospective cohort study is a research effort that follows over time groups of individuals who are similar in some respects (e.g., all are working adults) but differ on certain other characteristics (e.g., some smoke and others do not) and compares them for a particular outcome (e.g., lung cancer).[1] It should be emphasized that prospective studies begin with a sample whose members are free of the disease or disorder under study (e.g., free of lung cancer or free of major depression).

Retrospective cohort studies: A retrospective cohort study, also called a historic cohort study, is a medical research study in which the medical records of groups of individuals who are alike in many ways but differ by a certain characteristic (for example, female nurses who smoke and those who do not smoke) are compared for a particular outcome (such as lung cancer).

A retrospective (historic) cohort study is different from a prospective cohort study in the manner in which it is conducted. In case of Retrospective Cohort Study, the investigator basically collects data from past records and does not follow patients up as is the case with a prospective study. However, the starting point of this study is the same as for all Cohort studies. The first objective is still to establish two groups - Exposed versus Nonexposed; and these groups are followed up in the ensuing time period.

Case-control is a type of epidemiological study design. Case-control studies are used to identify factors that may contribute to a medical condition by comparing subjects who have that condition (the 'cases') with patients who do not have the condition but are otherwise similar (the 'controls').

What liquids used in GIC – Polyacrylic Acid

Focal spot influes resolution Collimation influences pentration Milliampere influences intensity KVP influences energy and pentration 2) If you mA Þ intensity a) Doubles b) ¼******** 1/8

Pt is taking methotrexate will have drug interaction with A) Beta blockers B) Alpha blockers C) NSAID’s D) Beta lactamase (but also beta lacatamse) With methorexate we cannot give amoxillin Most common odotogenic ectodermal is ameloblastoma Most common odontogenic mesenchymal is myofibroma (odontogenic mixoma)

Recommended daily intake of fat Intake as per USDA is 30% of total daily caloric intake or 10% saturated fat intake of caloric intake

SRP is used

American Society of Anesthesiology patient classification status ASA I Normal healthy Pt ASA II No functional limitations; has a well-controlled disease of one body system; controlled hypertension or diabetes without systemic effects, cigarette smoking without chronic obstructive pulmonary disease (COPD); mild obesity, pregnancy

ASA III Some functional limitation; has a controlled disease of more than one body system or one major system; no immediate danger of death; controlled congestive heart failure (CHF), stable angina, old heart attack, poorly controlled hypertension, morbid obesity, chronic renal failure; bronchospastic disease with intermittent symptoms

ASA IV Has at least one severe disease that is poorly controlled or at end stage; possible risk of death; unstable angina, symptomatic COPD, symptomatic CHF, hepatorenal failure

ASA V Unstable moribund Pt who is not expected to survive 24 hours with or without the operation ASA VI Brain-dead Pt whose organs are removed for donation to another

Periapical cyst pic

Odontoma pic

Recurrent apthous pic

Location of Inferior Alveolar Nerve -They then pierce the buccinator muscle between the palatoglossal & palatopharangeal folds, lying lateral to the medial pterygoid at the mandibular foramen.

Syphilis Chancre resembles 1) Cancer 2) Herpes 3)Herpangina 4)Apthous Ulcer

Most common Non-

1) dermoid cyst 2) thyroglossal 3) Nasopalatine 4)

Odontogenic Myxoma: (myofibroma)

 Most common of mesenchymal origin

 Post Mand

 Honeycomb & multilocular appearance

 Rx: similar to ameloblastoma & Giant cell Granuloma

 Tx: Curettage, possible recurrence

Ameloblastoma:

 Most common EPITHELIAL ODONTOGENIC TUMOR…mand molar area

 Age 40’s 50’s

 Histo: reverse polarity

 Ameloblastic fibroma: compared to ameloblastoma…happens in a. younger age b. slower growth c. does not infiltrate Ameloblastic fibro-odontoma . similar to the above except it occurs in the max & mand in equal freq.

Ameloblastic odontoma: . same as above except it occurs in max & mand pre-molar & molar area

All of the following applies to except

1)Male 2) Smoking 3)African American 4)Low socio economy 5)sex predilection for location

Metabolism of plasma esterase

Which of the following will not occur in over contouring of crown is A)Gingival problems 2)caries on the adjacent tooth 3) bone loss

Which of the following will occur more force on opposing dentition? a) complete denture b) tooth borne partial c) tissue born partials d) over dentures enamel pearls most commonly seen on Molars gingival palatal groove most like seen on seen in Max laterals mean energy of Xray photons is increased by increasing KVP collimation makes the X ray photon 1)decrease of fogging of the film 2) decrease the radiation to the pt 3) decreese the radiation to operative Fogging of film is in overdevelopment, contaminated sols, deteriorated films and light leaks more Kvp - less contrast for restorative purposes Dark radiographs - Overdevelopment, excessive mA, excessive peak kilovoltage, film- source distance too short.

Osteoradionecrosis occurs because of decrease in vascular supply Traditionally 1-2 wks btw TE and radiotherapy is suggested. however is better to delay radiotherapy 3 wks after TE. After Radiotherapy if tooth has necrotic pulp 1-endodontic Tx with systemic antibiotics can be performed 2- if difficult to do RCT(bcoz of sclerotic pulp)>>>tooth can be amputated above the gingiva and left in place

Pt is on IV bisphosphonates which of the following procedures can be done 1)Scaling and Prophy 2) 3) surgery 4) extractions

Treatment of mid face deficiency is

1) lefort I 2) lefort II 3) lefort III

Crouzon's syndrome exhibits severe proptosis (exapthalmous) Hypertelorism is seen in Crouzon's syndrome.maxilla underdeveloped.brachycephaly

Which endocrine system does thick hair become thin hair thyroid - hypothyroidism (cretinism in kids and myxoedema in adults)

Extrusion of canine what flap technique is used except 1)Envelope flap 2) Semilunar flap 3) Apical repositioning flap

Collagen disorder seen in advanced is seen in Diabetes and Rheumatoid Arthritis

Mechanism of action of on GABA receptors increasing the frequency of chloride channels by Benzodiazepines

Barbiturates increase the duration of chloride channel opening

Neurotransmitter in Parkinson’s disease is Dopamine

Pt has Asthma and is allergic to Aspirin what pain medicaction will be given? 1) Acetamenophen 2) Ibuprofen 3) Diclofenac Sodium NSAIDS - are contraindicated in asthma pts.

When you place a implant,widening of crestal bone is seen because of which force? a) Horizontal 2) Oblique 3) Vertical 4) Aapical

What is the reason for unhappiness of denture Instability of the lower denture

1) Mode of action of miconazole - alters the cellular wall permeability. It works by inhibiting the synthesis of ergosterol, a critical component of fungal cell membranes. 3) Cause of indiscrete margins on xray film 4) Antibiotic contraindicated with alcohol are Metronidazole, Tinidazole, Antimalarial, flurazolione, Griseofulvin - , nausea, vomiting, irregular heart beat, tachycardia, flushing, breathlessness, low BP 5) In a # of rt side of body of , which other # is to be suspected - condyle on the contra lateral side of the subcondylar region  Trauma to one side often produces an ipsilateral body fracture and a contralateral subcondylar fracture.

 A heavy blow to the symphysis produces a symphyseal fracture and bilateral subcondylar fractures.

 It is also important to exclude damage to the cervical spine and to ascertain that the airway is not compromised. 6) In condylar hyperplasia, mand deviates to which side? to the contralateral side (the unaffected side), in condylar hypoplasia it deviates to the affected side 7) Treatment of : a) marsupialisation, enucleation, inj of steroid into it for mucocele - enucleation 9) Pt complains of high fees of dentist, how should the dentist answer? Fee is fine according to the geographic area, it is fair and reasonable, I have to make a living too 10) Handling of an ADHD pt and how to get them to maintain oral hygiene - no modifications 11) Working side interferences are seen on what surfaces? palatal inclines of buccal cusp of upper and buccal incline of lingual cusp of lower; In MIP or CO, the buccal incline of palatal cusp of upper and lingual incline of buccal cusp of lower Balanced side interferences are buccal incline of palatal cusp of upper and lingual incline of buccal cusp of lower 13) Protection for inf alv nerve while placing an implant : surgically move the nerve, meticulous placement of the implant, place a barrier 14) Action of chlorhexidine? membrane disruption Denaturation of the proteins - alcohol and autoclave; Coagulation of proteins - dry heat 16) Source of epithelium for graft: host, graft, adjacent tissue. Source of the blood supply is the host connective tissue 17) After flap surgery, how does repair occur? Pdl moves occlusally, apically, laterally 18) Up on using the fluorescent light, in which of the conditions is the complete tooth illuminated: vertical#, periapical abscess, split tooth? 20) Colour of nitrous oxide cylinder - blue, oxygen cylinder is green 21) Description of a biohazard waste box. which one of the following is wrong about it: made of metal, closed, puncture proof, leak proof. 23) Disadvantage of hydroxyapatite graft - control of granules is hard and in the region of lower bicuspds, graft might be solidify over the mental foramen and cause parasthesia. 26) For a pt with head and neck cancer who is to be irradiated, when should be questionable teeth removed: before irradiation, after, during? 27) Treatment for lingually erupted #9 at age 7 – no tx 28) Advantage of bundling instruments before sterilization 29) What happens if penicillin and erythromycin are given together: summation, potentiation, idiosyncrasy 30) Numbers of surgical forceps for tooth extraction 31) Angulation of cutting edge to the tooth surface while root planning more than 45 less than 90 32) After using a gingival retraction cord, tissue reacts by recession. Where do you see this the most; lingual, buccal, interproximal. 33) Which aspect most commonly needs convincing for the pt with new CD: speech, chewing, max denture retention, mand denture retension 34) Which of the following is absent in facial palsy: , inability to wink, loss of msl tone, excessive salivation- dry mouth and dry eye 35) Consequences of tooth extn in a pt with adrenaline crisis – probably delayed healing, infection, inflammation If Pt uses more than 10 mg prednisone daily then: double the dose of daily steroids at the day of surgery as well as double the dose the day after surgery. (always after consultation with MD 36) Contraindications for the use of opiates – cannot be used with MOA inhibitors, incase of head injury 37)  Bacteria that is responsible for progression of caries but not causing the initial phase - 38) pt comes with eroded occlusal surface, lingual surface of max teeth, fillings raised from the surface. What should be suspected – possible GI regurgitation 39) pagets disease 40)  radiographic feature of osteoporosis 41) how is the caries rate in downs syndrome is less the patients with DS had a significantly lower prevalence of caries and significantly higher levels of salivary sIgA in this study. This finding tends to support the hypothesis that higher levels of salivary sIgA may protect against dental caries. But is more 42) Cleft lip and palate is usually associated with which kind of malocclusion? class III 43) Purpose of dietary analysis of pt -Caries risk assessment 46) Cause of dry socket: fibrinolysis of clot, physical dislodgement of clot? 47) Which of the following is a dentist not supposed to do: talk about options provided by other specialist, risks of not having rx done, benefits of having treatment done, discuss about copay? Note:If you see a fact e.g broken file,or tooth perforation you have to inform the Pt but not blame the previous dentist 48) Onset of action of antipsychotic is 5-6 days 49) Child with blue lips, thickened nail beds, pale skin. What disease is to be suspected( I think cyanotic heart disease) 50) msl that decides posterior extension of lingual side of dental flange? Mylohyoid, superior pharyngeal constrictor? 53) Define chief complaint - it should be in pt's own words 54) Oral hygiene instructions for ADHD pt: hygienist to pt, hygienist to pt and parents, dentist to pt, printed material 55) All of the following can be used for plaque removal except: water irrigation, tooth picks, toothe brushing, floss 56) Tooth brushing technique best suited for removal of interproximal plaque- Modified Charters

57) Where does interproximal caries generally occur: above contact area, below contact area, at contact area, area between marginal ridge and contact area? 58) How is wheeze heard? (description) - gasp followed by high-pitched whistling 59) Pregnant woman goes into syncope in second trimester. What should be the immediate action? turn her to the left in order to remove the pressure from inf. vene cava 60) Dealing an angry patient 61) Behaviour shaping and modeling 62) Diabetes is common on which race? balck men 63) Use of pulp testing: check disease or health or extension of damage of pulp, check the responsiveness of pulp nerves. 65) How does a tooth covered with crown react to pulp testing--- cold is better test 67) Periodontitis is common for which race? Black 68) Night guard is used for: treating , redistribute occlusal forces 69) Should be able to identify defect in hue, color, chroma on a photograph. Displacement of fractured segments of mandinble by the action of associated muscles. If the # is at symphysis the muscles are genioglossus and ant. Digastric. Read more abt it. pt is addicted to oxycodone which contra indi- Patients receiving CNS depressants such as other opioid analgesics, general anesthetics, phenothiazines, other tranquilizers, centrally-acting anti- emetics, sedative-hypnotics or other CNS depressants (including alcohol) concomitantly with PERCOCET tablets may exhibit an additive CNS depression. When such combined therapy is contemplated, the dose of one or both agents should be reduced. The concurrent use of anticholinergics with opioids may produce paralytic ileus.

.bioavailability---amount of drug avilable in systemic circulation .antipsychotic with irreversible side effect----tartive kinesia .lantaprost indication---- Xalatan® (latanoprost ophthalmic) – used for gluacoma .common site for granular cell myoblastoma(granular cell tumor)---tongue .large radioopaque lesion in carious affected tooth---condensing osteitis .lesion commonly with dysplasia and carcinoma in situ--- .common finding in patient with ataxic cerebral palsy---ant tooth fracture .common reason for angular chelitis---decreased vertical dimension,skin fold,poor diet (Vit B2 and B12) In edentulous pts, when there is decrease vertical dimension, most common sign is ANGULAR CHELITIS Ideal depth of a bone for an implant · What group has the highest prevalence of caries? Blacks, Whites, Native Americans Hispanics. Prevalence of caries in permanent teeth (DMFT) among adults 20 to 64 years of age, by selected characteristic– White >Black>Hispanic Prevalence of untreated caries in primary teeth –hispanic>Black >white Prevalence of untreated decay in permanent teeth (DT) among adults 20 to 64 years of age- Black >Hispanic> White Mean number of decayed, filled, and decayed or filled permanent teeth among adults 20 to 64 years of age – White>Hispanic>Black Moderate periodontal disease was most prevalent in males, blacks Prevalence of Class II malocclision more in Europian , Class III maloclusionin Asian Class III malocclusion with cleft lip and palate is more in Native Americans > Oriental and Caucasians > Blacks. Cleft lip is more common in Asians Clift lip prevalence in USA is 1:700- 1:800 Cleft lip is done in 2-3 months after the child is born Clept palate is not done until the canine erupts and maxillary growth completes at age 10-12 The lip is corrected as early as is medically possible.the soft palate is closed btw 18-24 months of age,leaving the hard palate cleft open. the hard palate cleft is then closed around age 4 or 5.PETERSON Alveolar correction surgery is at age 8 before the eruption of canine (Before canine erupt------surgery to correct major defects of clefts in the lip and hard and soft palate is done in infancy - before the age of two. But correcting the alveolar cleft is usually left till a later time, around the age of 8 or 9. This is because if the surgery is conducted when the child is younger than this, the scar tissue that forms during healing will interfere with the normal development of the face. An alveolar cleft is corrected with a graft of bone and soft tissue.) · Sodium hypochlorite in has the following advantages EXCEPT: 1.lubricating, 2.Anti-microbial, 3.Chelating agent, 4.Disinfection · Gutta percha has the following advantages EXCEPT: 1.easy manipulation, 2. Adapts to tooth surface, 3.Anti-microbial,4. Biocompatible · Temperatures for autoclaves is governed by FDA. · For carbide stainless steel burs, what form of sterilization does not corrode the surface? 1. Dry Heat, 2. Steam , 3. Both Dry Heat and Steam, 4. Neither Dry Heat nor Steam · ?To improve the quality of radiographic film, what parameter would be altered? 1.mA, 2. kVp, 3. Exposure time, 4. · high Kvp – low contrast for perio. mA is quantity · What bacteria is involved in the carious process but NOT an intitator? 1. Streptococcus, 2. Lactobacilli, 3. Actinomyces, 4. Staphylococcus · During root canal treatment, the operator creates a ledge. What is the next step? 1. Fill up to the ledge and leave the ledge intact, 2. Extract and replace with an immediate implant, 3. Take a smaller file to working length, remove as much debris as possible and gently try to remove the ledge. · What is the NOT an advantage of stainless steel files? 1. More flexible, 2. Less chance for breaking, 3. Allows the file to be centered in canal, · Which tooth is least successful for a patient to remove plaque with floss? 1. Maxillary premolar, 2. Mandibular premolar, 3. Mandibular molar, 4. Maxillary molar · A study failed to report 5 cases of caries. What is this called? 1. True Positive, 2. True Negative, 3. False Positive, 4. False Negative · Upon extraction of a maxillary molar, a 2mm sinus defect is noticed. What is the next step in treatment? 1. Prescribe antibiotics and nasal decongestant, 2. Open a flap and close the sinus involvement defect with suture, 3. No treatment necessary, follow up in 3 days. · A teenager (11 yo) presents with an intruded maxillary lateral incisor (#7). What is the next step of treatment? 1. Extract and Implant, 2. Splint for 2 weeks with RCT treatment after, 3. Splint for 2 weeks and follow up · For a young patient (6 yo), the treatment of choice for a necrotic pulp on permanent first molar would be: 1. Apexification, 2. Apexogenesis, 3. Root Canal Treatment · Which treatment has the least successful long term prognosis on a deep carious lesion on #3? 1. Direct pulp capping(correct answer), 2. Indirect pulp capping, 3. Pulpotomy, 4. Pulpectomy and RCT · What would be the most reasonable cause for a tooth’s symptoms to change from reversible to irreversible ? 1. Accumulation of traumatic injuries, 2. Bacterial involvement inside pulp chamber, 3. Increased intra-pulpal pressure · What is the most accurate pulpal test to determine vitality of a tooth with a full-gold crown? 1. Electric testing, 2. Percussion test, 3. Palpation test, 4. Thermal test · What is the outline shape of the access cavity for a permanent mandibular first molar? 1. Triangular, 2. Trapezoid, 3. Oval, 4. Circular · On a radiograph, you notice a distal carious lesion on the mandibular first premolar. Upon restoring the adjacent 2nd premolar, with direct vision, you notice that there is no decay. What is the most probable cause? 1. Cervical burnout, 2. Root caries, · For a lesion in enamel that has remineralized, what most likely is true? 1. The enamel has smaller hydroxyapatite crystals than the surrounding enamel, 2. The remineralized enamel is softer than the surrounding enamel, 3. The remineralized enamel is darker than the surrounding enamel, 4. The remineralized enamel is rough and cavitated · On the cementation day for a full gold crown, what is the first step? 1. Evaluate margins, 2. Evaluate occlusal contacts, 3. Evaluate proximal contacts, 4. Evaluate proximal contours · For a surgical extraction of mandibular first molar, what is the best course of action? 1. Cut a Y into the coronal portion and extract each root separately, 2. Cut the coronal portion from the roots and extract the roots separately, 3. Cut the tooth bucco-lingually and extract the roots separately · For a surgical extraction, what does not contribute to developing post- operative pain? 1. High-speed drill, 2. Low speed low torque drill, 3. Sharp burs, 4. High-speed drill with water spray · Know the onlay wall preparation, which walls are convergant, divergent and which wall is axiopulpal, axiogingival, etc. · Axial walls converge in inlay preps. · pulse n bp what kind of measurement - nominal, ordinal, interval, ratio temperature - apirin - single dose - how much time- 4 hours, 1 day, for baby aspirin dosage is 81mg/day aot - radiograph sore muscles in the morning – Myofacial pain dysfunction syndorme gingevectomy, alveoloplast , alveolectomy diff, free gingival graft , modified flap- full trephination- open rct, conv rct, release pressure by incision through bone. what is th immediate treatment plan - pulpectomy , initial treatment ginseng- antiplatelet ( interferes with coagulation – not given with aspirin). pt on warfarin, aspirin width periodontal ligament – .2mm hue – color, staining changes, chroma – saturation(lightness or darkness of the color), value – number of rods In natural teeth 2 types of hue – yellow and yellow red. Yellow stain – increases the chroma in the yellow hue Orange stain – increase the chroma in the yellow red hue. Pink purple – changes the hue of the teeth. Move from yellow to yellow red. crown - unesthectic- value pocket, depth elimination procedures perodontal stability- attachment loss is the most imp criteria in diagnosing periodontal conditions, followed by mobitiliy. buproprion – smoking = Bupropion (Wellbutrin, Zyban), previously known as amfebutamone,[1] is an atypical antidepressant and smoking cessation aid. It acts as a norepinephrine and dopamine reuptake inhibitor, veracity - being truthful to patient dentist doesnot complete the treatment - pt abandonment, protection act- good smaritan act – is accepted in all states, but dentists are not included in all the states.

1) Collimation does everything except –reduce pt exposure, reduce operator exposure, film fog, reduce average energy of x-rays 2) The Dentist completes exam and advises x-rays but the pt refuses. What should the dentist immediately do. ANS. Explain the need for X-rays. 3) What is true of osteoradinecrosis. ANS. Commonly affects mandible X-ray identification: 4) Median Palatal Suture 5) Zygomatic process of maxilla 6) Dorsal Surface of tongue 7) Odontoma 8)The severity of response increases with the amount of X-ray exposure.This efeect is called. Deterministic, Stochastic, Genetic? 9) Radiosensitive cells ANS. Hemopoeitic bone marrow. 10) When there is no barrier, protection of dentist. 6 feet; 90-135 degrees 11)If the buccal root of maxillary ist premolar appears distal to palatal root ANS.The X ray was taken from mesial angulation 12)Reverse occlual plane on a panoramic view. ANS.Chin tilted upwards; smiley (happy face) is chin down. 13)Best view to observe of maxillary sinus .CT , Panoramic, Lateral skull view ( all these are good to view max sinus)waters 14)What is the reason for increased radio-opacity on the mesio-occlusal surface of maxillary I molar. Cusp of Carabelli, restoration on disto-lingual surrface, dens in dente 15)Transillumination is useful in the diagnosis of .Class 1, class 2, class5, class3 16) The dentist sees caries on the distal root of maxillary 1 molar, he prepares cavity for 2 molar and realized that the surface is smooth and non-carious. Why? ANS.Cervical burn-out 17) Commonest tooth with vertical root fracture .Maxillary incisor, mandibular incisors, maxillary molars, mand.molars. 18) Shape of access opening for mand.molar with 4 canals. ANS.Trapezoidal 19) Which tooth refers pain to the ear ANS. Mandibular molar Referred pains: Max Incisiors – forhead maxillary Canine and premolar – nasolabial, maxillary second Premolars – Temporal region, Mand. Molars – Ear and infratemporal region. 20) Which are the two most imp. steps for diagnosis. ANS.History and clinical examination 21) A no. of questions on pulp vitality testing and the conditions of pulp and peri apex deending upon the symptoms i.e, normal pulp, normal peri apex, reversible pulpitis, irreversible pulpitis. 22) Which of the following is not a property of gutta-percha . radiopacity, Biocompatibility, Antibacterial,Adaptation 23) A no.of questions on pulpal therapy depending upon the pts tooth condition and age i.e, apexogenesis, apexification e.t.c, 24) Which is not a property of NAOCL . Dissolves tissue, lubricant, bleaches, antimicrobial. Chelating. 25) Which method of sterilization does not corrode instruments – Dry Heat, Ethylene oxide 26) Which method of sterilization does not dull carbide instruments – dry heat 27) Which procedure does not require antibiotic prophylaxis . Non surgical endodontic treatment, Scaling and root planing, Extraction, surgical treatment. 28) Material used for mouth guard vital bleaching ANS. 10% carbamide peroxide. 29) Most critical for pulpal protection ANS. Remaining dentin thickness 2mm 30) Critical factor determining the success of implantation of avulsed tooth - time 31) Tooth with the best prognosis ANS. Internal resorption 32) What is the effect of extraction of primary 2 molar on the eruption of 2nd premolar with one third root formation . Eruption is faster. Eruption is slow, no effect on eruption rate, condition varies depending on the patients age. 33) which structure exhibits synchondrosis (Where the connecting medium is cartilage, a joint is termed a synchondrosis. An example of a synchondrosis joint is the sternocostal joint (where the first ribs meet the sternum). In this example, the rib will join up with the sternum via the costal cartilage.) 34) What is the effect on growth of a child with unilateral sub-condylar fracture. – retarded growth 35) Gingival index is an example of .ordinal scale, nominal scale 36) what is the reason for loss of shiny surface but not the frosty appearnce after acid-etching of enamel with 37percent phosphoric acid for 30sec.improper etching,improper washing of enamel,fluoridated tooth 37)Hybrid layer. ANS.primer within intertubular dentin 38)Causes for increased soreness on the crest of the ridge after placing C.D ANS.Improper occlusion, if generalized redness over the denture area, then it would be due to allergy to acrylic. 39)The provoking factor for pain after placing bridge. Heat, cold, sweet, biting (occlusion) 40) Kerrs syndrome Kerrs syndrome shows all except . Maxillary ridge resorption, flabby tissues, enlarged tuberosities, increased vertical dimension of occlusion. 41)Lack of function of indirect retainer is manifested by ANS. Lifting of distal extension away from tissues. 42)Primary function of labial flange of maxillary denture ANS.esthetics. 43) Function of face bow 44)Frankfurts plane porion to orbitale 45) occlusal plane should be parallel to ANS.ala-tragus line 46)Common feature between porcelain veneer and all-ceramic crown preparation – rounded internal 47)How do you decrease the width of artificial teeth. Deepen the facial line angle proximally and increase the interproximal embrasure, Deepen the facial line angle proximally and decrease interproximal embrasure, take the facial line angle labially and increase the interproximal embrasure, take the facial line angle labially and decrease the interproximal embrasure. 48) The dentist cements the porcelain veneer with light cured resin and the patient returns with brownish discoloration at the margins.why?not enough cement or microleakage(depends on duration of pt return) Pregnant women in syncope – turn the pt to left to relieve inferior vena cava 49) patient receives a blow to the chin who has a MOD inlay placed on the maxillary molar 3 months earlier.Now the patient has a vague pain on biting ,there are no other symptoms.why? maxillary sinusitis,m-d fracture, b-l fracture 50) questions on pier abutment and cantilever 51)when you wax the removable partial denture on a cast; what is it called- Master cast, refractory cast 52)Purpose of addition of tin and indium to metal ceramic alloys ANS.chemical bond-covalent bond 53)What is the basis for classification of different F P D pontics. ANS.relation of the pontic to the supporting tissue. 54)Cause for post-operative sensitivity of composites - shrinkage 55) The condylar guidance is increased from 20 to 45 degrees,what do you do. Increase the compensatory curve - the curvature of alignment of the occlusal surfaces of the teeth that is developed to compensate for the paths of the condyles as the mandible moves from centric to eccentric positions. A means of maintaining posterior tooth contacts on the molar teeth and providing balancing contacts on dentures when the mandible is protruded. Corresponds to the curve of Spee of natural teeth. curve of Spee, n.pr 1. an anatomic curvature of the occlusal alignment of teeth, beginning at the tip of the mandibular canine, following the buccal cusps of the natural premolars and molars, and continuing to the anterior border of the ramus, as described by von Spee. 2. the curve of the occlusal surfaces of the arches in vertical dimension, brought about by a dipping downward of the mandibular premolars, with a corresponding adjustment of the maxillary premolars. curve of Wilson, n.pr the curvature of the cusps, as seen from the front view. The curve in the mandibular arch is concave, whereas the one in the maxillary arch is convex. curve, reverse, n a curve of occlusion that is convex upward when viewed in the frontal plane.

56) What is rest position – muscle guided Centric occlusion is tooth guided, centric relation is ligament guided. 57) Non-working interferences 58) You are giving cusid-cuspid bridge and want to change the canine guided anterior disclusion.What is required. – change to group function 59) All are requirements of interocclusal record material except. Hard when set, capable of trimming with sharp knife after set, resistance to biting. 60) Anterior guidance is determined by and 61) Minimum ridge width for 4mm implant is. 6mm, 8mm, 4mm, 10mm 62) All are symptoms of TFO on an implant except. Gingivitis, pain, loosening of implant, breakage of abutment screw. 63) Incidence of cleft lip&/ cleft palate is 1 in 700 64) Heart rate of 4 yr child is Age 3 is 110 and age 5 is 100 65) Highest incidence of untreated dental caries is seen in .Americans, blacks, Asians, Hispanics, 66)Highest incidence of dental caries in children is seen in hispanics 67) Greatest incidence of malignancy is seen in .Pagets disease, monostotic fibrous dysplasia monostotic fibrous dysplasia: malignant change ,usually development of osteosacoma has been RARELY associated with fibrous dispalsia.radiation for this lesion is contraidicated because it carries the risk for developmant of post-radoiation bone sarcoma .Development of malignant bone tumor,usually an osteosarcoma is recognized complication of Paget,frequency of osteosarcoma is 1%,mostly long bone. NEVILLE 68)Oral granulomas, apthous ulcer, rectal bleeding is seen in. Wegeners granulomatosis, ulcerative colitis, crohn’s disease. 69) Pierre-robin syndrome consists of a triad of retrognathia, glossoptosis, cleft palate. 70) Most important factor in shade selection. ANS-value 71) Flexibility of a clasp arm depends on all except. Length, taper, circumference, depth of undercut, 72) Which is most rigid .pd-Ag, typeIII gold,typeIV gold. 73.Least wettability is seen with which impression material.polysulfide, condensation silicones, hydrocolloids, polyether. 74) Most stable in moisture environment. Polysulfide, condensation silicones, addition silicones, polyether. 75) Setting of polyvinylsiloxanes is retarded by. ZOE, latex gloves, ferric chlorides, aluminium chloride 76) Which produces least change on implant surface while removing . Ultrasonics, sonics, curettes, plastic curettes 77) What has ultimate effect on the thickness of epithelium of free gingival graft. Recipient epithelial tissue, donor epithelial tissue, donor CT, recipient CT 78) Disadvantage of partial thickness flap are Visibility, access??? 79) The purpose of GTR is to prevent. Long J.E, migration of PDL cells, Migration of CT cells. 80) The resorption of bone in PD disease is caused by.IL1, IL8, IL10 IL 8 – chemotaxis, IL 10 is macrophages. 81) The biological width is. 2mm,3mm, 4mm, 1mm, 5mm. 82) The internal bevel gingivectomy extends approximately from? (undisplaced flap) 83) Indications for gingivectomy – gingival hyperplasia, 84) The purpose of barrier. Apical movement of PDl cells, Coronal movement of cells. 84) Gingivectomy is contraindicated in .The sulcus is apical to gingival groove, sulcus is apical to convexity of tooth, sulcus is apical to the crest of alveolar bone. 86) periodontal pathogens in health- ANS Gram +ve facultative cocci and filaments. 87) Desquamative gingivitis is associated with which 2 conditions. Pemphigus and bullous pemphigoid, and erythema multiforme Pemphigus – supra, acantholysis Pemphigoid - basal 88) The depth of sulcus is 5mm,the distance between CEJ and the base of sulcus is 2mm.what is the attachment loss. – 2mm 89) The role of chlorhexidene is cos of.ANS- Substantivity – high concentrations for long time 90) Condition seen in normal individuals and also in pregnant patients.ANS Pyogenic granuloma or pregnancy tumor. 91) Antibiotic seen in GCF- ANS Doxycycline, minocycline 92) First thing u need to check while placing crown.ANS Esthetics then Proximal contact 93) The normal recall appointment between periodontal treatment. 3 months, 1 month, 4 month, 6 month 94) 45 year patient comes for appointment schedule.His B.P is 160/100.What should the dentist do.Call up the physician, reschedule appointment when the B.P comes down , check B.P again after 15 mts 95) Frequent urination seen during 3rd trimester ANS. Pressure on the bladder. 96) Curettes used for distal surface of maxillary II molar ANS. #13-14 97) Pt comes with carious involvement and localized swelling of cheek.What is the immediate treatment. ANS.establish drainage 98) After periodontal surgery, the dentist leaves interproximal bone apical to radicular bone.What is this called negative architecture. 99) Evaluation after scaling and root planning is done after how many days. 7-10 days, 14-21 days 4-6 weeks 100) Mode of action of ultra sonics. ANS. Vibration in elliptical(magnetostrictive) , sonics is linear(work with air) 101) Which of the following is not an advantage of Ni-Ti over stainless steel file. Maintains the shape of canal, flexibility, resistance to fracture. 102) Best theory to explain . ANS Hydro dynamic theory 103) Gastric acids cause. ANS. Erosion 104) Primary effect of an over hang. ANS interferes with plaque removal 105) Medications associated with hyperplasia. Calcium channel blockers, Dilantin sodium, Cyclosporine 106) In most of the cases, localized fibromas are often. Dysplasias, metaplasia, anaplasia, hyperplasia. 107) The major environmental risk factor for periodontal disease. Diabetes, smoking 108) which is not true of elder abuse. Most of the elder abuse is at victims home, mostly it is by victims relative, elder’s abuse is often over reported and exaggerated, un-authorized use of ATM card is some times considered crime but not abuse. 109) Most common condition affecting elders. ANS Depression 110) features of dementia are all except. Short term memory, long term memory 111) All are seen in DI except. Large bulbous crowns with short tapered roots, large irregular dentinal tubules, accentuated DEJ, opalescent hue. 112)In a scientific article ,where should the description of study sample be.materials and methods, results, discussion 113) Common form of leukemia in children – Acute lymphocytic leukemia; acute leukemias respond to tx well, chronic doesn’t . AML is worst prognosis, Viral load for HIV pts – test sensitivity – detect more than 48 viral copies., Neutrophils – less than 500 no tx Platelets – below 50000 no tx CD4 – should be 200, but 50 is critical. 114) Increased no.of teeth + supernumerary and impacted teeth is seen in ANS. Cleidocranial dysplasia. Gradner’s syndrome has supernumerary teeth. 115) when should the dentist not use para-phrasing. When trying to speak to a patient in his second language, When the dentist is upset with what patient says, when giving factual values. 117) The patient who is diagnosed with Basal cell carcinoma, says to the dentist “give me the report,do I have cancer”, what should the dentist’s initial reaction be. DO you have any one accompanying you, this cancer has better prognosis than other cancers. 118) Which of the following is not recommended for a patient who is on Nicotine de-addiction .mucous patches, nicotine gum, Bupropione, nicotine nasal sprays. 119) Which of the following does not have cauliflower like , pebbly appearance. , fibroma, condyloma accuminata, papilloma. 120) Common finding in a patient with cerebral athetoid palsy. ANS. Anterior teeth fracture 121) Anti-fungal used as troches for treatment of .ANs Clotrimazole; swish and swallow is nystatin. AmphotericinB is only antifungal given in IV, Fluconazole (tab) is given for systemic candidiasis. 122) The direction of healing of wound after extraction of tooth in mandibular arch .outward and upward, inward and down ward, etc 123) Infections from premolars spread to which space.ANS. Submandibular, because the roots are below the mylohyoid. Anterior roots above the mylohyoid has submental spread. Cellulitis most of the time involves unilateral , ludwigs angina is bilateral and complication is edema of GLOTTIS. 124)The patient who is a tobacco addict says to the dentist,”I want to quit the habit”,what should the initial reaction of the dentist be. 125) All are seen in a patient with sjogrens syndrome except Xerostomia, lymphoma, arthritis, other choice is option. 126)Amount of fluoride supplementation required in 16 yr .0 mg 127) The greatest incidence of dysplasia and carcinoma in situ ANS.erythroplakia 128) Clinical photographs- Lymphoepithelial cyst, cheek biting, smokers palate, Dentigerous imperfecta, facial palsy, migratory glossits, mucocele, fordyces granules 129) Dentist realizes that the distal margin of crown preparation is within 1mm of interdental bone.what should the dentist do. ANS crown lengthening Necrotizing sialometaplasia minor presents on the palate which is most commonly confused with carcinomas due to the ulcerated presentation. Heals without scarring. 130) Transillumination in children is used for.ANS 131) Questions on herpes ulcers and apthous ulcers 132) The patient develops oral ulcers and target or bulls eye skin lesions within 24 hrs.ANs Erythema multiforme 133) Suppuration is seen with ANS.Actinomycosis – sulphur granules, seen in the neck. 134) A chronic alcoholic requires extraction.what test should the dentist advise for.ANS INR 135) which of the following is not true for a patient requiring surgery and on oral anti coagulants. PT should be 1.5-2 times that of control, INR should be above 2.5 Norma INR is 1, can be treated till 3(acc to manual) 136)Question on parulis bumps on the gum parulis, Pyogenic granuloma, peripheral gaint cell granuloma, peripheral ossigying fibroma,peripheral odontogenic fibroma, 137) A patient received radiation therapy and requires extraction,what should the treatment be.Extraction, extraction with alveoloplasty and sutures, extraction with alveoloplasty of basal bone and suture, pre-extraction and post extraction hyperbaric oxygen 138) Virus associated with Chicken pox also causes ANS herpes zoster Unilateral lesions after herpes zoster infection. Reactivation of the virus from sacral ganglion causes , induced by stress and sunlight exposure. 140) Fluoride supplement required in a 2.5 yrs child in a non-fluoridated area. 0.25mg Community water fluoridation – 1ppm, school water fluoridation – 4.5ppm 141)A 4 yr old child management-ANS .empathy and respect 142) Management of moderately apprehensive child Replacing words like LA with sleepy juice is called as Euphenism. 143) Question on tell, show , do 144) Use of praise, smile and appreciation is-Token(positive) reinforcement, social reinforcement 145)What is not an advantage of rubber dam when compared to not using it. Improved properties of materials, shortens operative time,facilitates the use of water spray 146) Closest resemblance of deciduous mand II molar ANS permanent mand I molar 147) The success of implants does not depend on.ANS .Age of the patient 148) lefort I fracture involves .Frontal, ethmoidal, maxillary ,mastoid 149) Prilocaine above 500mg causes. Apnoea, cardiogenic shock, methemoglobinemia. 150) complication associated with removal of internal oblique ridge(mylohyoid) ridge. ANS Lingual.n 151) The drug which causes withdrawal symptoms in pts taking oxycodiene 152) The drug contra indicated in pt taking gingko biloba ANS Heparin Ginseng used for male impotency, diabetes Ginkgo biloba – used for memory loss. 153) The drug of choice in patient with bradycardia - Atrophine, Epinephrine. 154)Which of the following does not have anti-inflammatory action. ANS Acetaminophen 155) The immediate choice of treatment for large radiolucency in the mandible. Biopsy, aspiration biopsy and wait for the bone to fill in Safest of all biopsies – aspirational 156) The common symptom of is associated with which space involvement. Sub-masseteric, superficial temporal During IAN block, pierce Buccinator muscle and inject in pterygomandibular space. 157) The patient returns to the dentist the next day after extraction with pain and swelling.The drug of choice in this penicillin allergic pt. Clindamycin 300mg qid 158) The restraining of unco-operative 2 yr child should be done by.Dentist, Assistant, Parent 159) The information about hazards of chemicals used in the office should be present in ANS. Material safety data sheets Red – flammable; White – Personal protection, Blue – health hazard, Yellow- identitifies the reactivity or stability of a chemical. 160) The failure of a test to detect the presence of 5 cases of disease. ANS. False negative 161) The 16 yr old can take the decisions for the elder pts if. If the elders are deaf and dumb, if the boy makes the payment, if the elders are over 60yrs, if the kid has the power of an attorney 163) The dentist separately for core- build up and the crown but the insurance company says that the core build up is part of crown.what is this called. bundling 164) the investigator studies the occurrence of oral cancer in pts in a private nursing home.what is this . Cross-sectional study, longitudinal study, Descriptive epidemiology(prevalence and incidence) 166) common missing permanent tooth ANS. Maxillary third molar 167) The main role of chlorhexidene before surgery ANS. Reduce the no.of microbes 168) What is an allograft 169) Crowing sounds are seen with. Acute asthmatic attck, COPD. Asthma – Exhalation wheezing, COPD – expiration wheezing. Croup------Barking Cough 170) Blood transfusion before surgery should be done when the platelet concentration falls below. 20,000, 50,000 , 100,000 etc 171)The most effective method of caries reduction. ANS. Systemic water fluoridation 172) 5 A’s of cancer prevention : Ask, Advise, Assess, Assist, and Arrange 173) The Child has 12 deciduous and 12 permanent teeth.what is the age of the patient 8-9 yrs 174) When does the calcification of permanent incisors occur 3-4 months(decks) Excepting maxillary lateral incisors 10-12 months 175) The drug of choice in status epilepticus Diazepam 176) The bacteria that causes rheumatic fever.streptococcal infection like sore throat or scarlet fever 177) What is the correct method of excavation of deep caries. Long bur from periphery to the center, large bur from center to periphery, small bur from periphery to center, small bur from center to the periphery. 178) purpose of post. ANS.retention of core 179) Apical closure of permanent tooth occurs. ANS.21/2 to 31/2 yrs after eruption 180) Face is divided vertically by how many planes.2, 3, 4, 5 Vertically in 5's, horizontally thirds 181) The dentist realizes that there is formation of ledge.what should he do next.ANS Gently by pass the ledge. 182) Pt asks questions regarding the face mask, gloves and other sterilization methods.what should the dentist say. probably something to do with standard (universal) precautions 183) The patient retires and loses health benefits.the treatment is done on the next day.the pt requests the dentist to enter the previous day date and the dentist does so.what is this called.ANS. Fraud 184)The frequent intake of juices by the child does all except. ANS. Increase ph., juices decrease PH 185) Which is true of intra-pulpal anesthesia.produces anesthesia after 30 sec, it does not cause the discomfort to patient, produces anesthesia by pressure. 186) The position of permanent incisors in relation to deciduous incisors. ANS.inferior and lingual 187) Advantage of distraction osteogenesis over ostectomy . better patient compliance, wider range movements, less relapse and can be done in yonger patients 188) What is not true of .ANS pain starts on the first day, curettage should not be done in pts with dry socket, no antibiotics. 189) Acid-etching does not cause. Reduced leakage,better esthetics, increased strength of composites. 190) Purpose of placing posterior palatal seal ANS.compensates for shrinkage. 191) Patient comes back to the clinic 5 hrs after extraction with bleeding .what should the dentist do. ANS.search for the source of bleeding 192) flexibility of the clasp depends on all except. Depth of the undercut, cross- section of the clasp arm, taper of the clasp, length of the clasp. 193) Pt after scaling and root planing and with excellent oral hygiene does not show reduction of pocket depth, what is the next step for the dentist to do.ANS. Periodontal surgery. 194) Common location for periodontal cyst.ANS. Lower bicuspid.? is detected only during radiographic examination.65% of cases occur in mandibular canine-premolar area.NEVILLE 195) Patient has 2mm communication with the maxillary sinus.what is the treatment of choice. 2mm don’t do anything and follow up 2-6 mm place gel foam (surgicel), suture ,decongestant and antibiotic , inform patient more than 6 mm buccal flap.( Peterson) 196) How to split the impacted mandibular tooth when viewed from occlusal.+, Y slope, split buccal to lingual till the furcation. 197) Questions about potency and efficacy. potency - response to a drug over a given range of concentrations.Potent = depend on dose of drug- less mg for same efficacy has more potency efficacy - effect of a drug -efficacy is the max effect of the drug. Max effect is also called as intrinsic activity. Therapeutic index - is an estimate of the margin of safety of a drug. TI = Lethal dose 50/Effective dose 50 198) What is bioavailability of a drug. - amount of drug that is available is blood. 199) What is the effect seen when propranolol and epinephrine are injected simultaneously - in cases of mild reactions it causes hypotension; in severe reaction it is malignant hypertension Synergism - same target of action-increase action additive = target of action is different

Day-2

1) patient c/o burning sensation at the corner of the mouth.- angular chelitis 2) Peri apical abscess 3) Trauma to the anterior area of the mouth 4) Cheek biting 5) 6 yr old complaining of halitosis. 6) Pt c/o frequent dislodgement of anterior crown.(post and core)

New q’s 1. Angular chelitis - common reason is malnutrition; due to decrease VD in removable pts 2. Neuropraxia - nerve damage in which there is no disruption of nerve or it's sheath, but causes interruption in conduction of impulses 3. Recommended daily dose of fat - 30% of total calorie and saturated fat is 10% of daily calorie intake 4. Extension of lower denture - till retro molar area : a. primary support area mandible : Buccal shelf , maxilla- Ridge, secondary on max - rugae. 5. What incision is given to remove palatl tori - double Y incision was not the choice ; other possible answer might be a linear incision from A-P with oblique releasing incisions 6. How do u clean implants - scaler with a plastic sleeve; rubber cup and paste, stailess steel scaler, titanium scaler 7. It is required mandatory to report all except - child abuse, reaction to drug, one more choice, Abuses that have to be reported to authorities - colleague practicing with chemical impairment, colleague advertising on electronic media, child abuse, domestic violence, elderly abuse 8. Asthama - wheezing on inspiration; COPD - wheezing on expiration 9. Why do you splint teeth in FPD - to distribute forces equally 10.Listerine - Phenolic compound - Listerine is a combination of alcohol + phenolic compound. Burning after use of mouth wash is due to alcohol (causes drying of the mucosa). 11.Patient had old PFM on # 9, he complains that PFM is lighter than other teeth, what will u do? - change PFM, bleach teeth 12.When will you bleach teeth in anterior veneer prep- before veneer prep, wait for 2-3 weeks, after preping veneer and then bleach, after cementing veneer and bleach a. bleaching affects both enamel and dentine 13.something on incisal guidance - guided by overjet and overbite 14.IRM is - what is the compoisition - zinc oxide-eugenol with polymer reinforcement. 15.where will you place the margins in a anterior PFM prep - Subgingival, at the crest 16.PVS impression material has which of the following - is unstable, doesnot come out completely from tissue undercut, releases alcohol as byproduct, may release H ion 17.Mostly periodontits occurs in : Diabetes, Tobacco 18.St John Wort(herb) is used for - mild depression--St John's wort is today most widely known as an herbal treatment for depression,may work against gram negative bacteria,ADHD; has drug interaction with BNZ 19.Drug that interferes with coagulation - Saw palmetto; Saw palmetto products could theoretically interfere with blood coagulation, so concurrent use with blood thinning medications, such as coumadin (warfarin,aspirin and other NSAID’s) should be avoided. 20.Differentiate hypothyroidism and hyperthyroidism 21.Hypoglycemia is common in : Hypotension, Hyponatremia, Hypokalemia 22.Picture of : herpetic gingivostomatitis; steven johnson syndrome 23.X ray of odontoma; 24.Drugs used to control salivary secretion - Pilocarpine- increases salivary secretion, Atropine - decreases salivary secretion 25.To expose a mandibular lingual torus of a patient who has a full complement of teeth, the incision should to a. semilunar b. paragingival c. in the gingival sulcus and embrasure area d. directly over the most prominent part of the torus e. inferior to the lesion, reflecting the tissue superior 26.Periodontitis is most prevalent in - Black female, Black male, white male, white female 27.Arcon and Non arcon articulator - when u use acron articulator 28.Patient has a stable bite, ant. guidelines are fine and you need to fabricate FPD. What kind of articulators do you use? - Semi adjustable. Non-Adjustable - (Class 1) A simple holding instrument capable of accepting a single static registration. Vertical motion is possible. (Class 2) An instrument that permits horizontal as well as vertical motion but does not orient the motion to the temporomandibular joints. Uses: Single restorations Multiple restorations if good bilateral cuspid disclusion exists Certain FPD’s.

Semi-Adjustable - (Class 3) An instrument that simulates condylar pathways by using average or mechanical equivalents for all or part of the motion. These instruments allow joint orientation of the casts and may be arcon or nonarcon instruments. Examples: Hanau H2 (condylar/non-arcon) Hanau University Series (arcon) Whip Mix (arcon)

Uses: Multiple restorations Fixed partial dentures with: - minimal occlusal pathology - no loss of VDO - no immediate side shift Full mouth restoration if good anterior guidance exists.

Fully-Adjustable - (Class 4) An instrument that will accept three dimensional dynamic registrations. Those instruments allow for orientation of the casts to the temporomandibular joints.

Uses: Full mouth restoration Extensive occlusal pathology Group function/posterior guidance Restoring at a different VDO

ARCON ARTICULATOR - An instrument following anatomic guidelines such that the ball of the condyle analogs are carried on the mandibular element, and the fossa assemblies on the maxillary element.

CONDYLAR ARTICULATOR - A non-arcon articulator.

Advantage of Arcon over Condylar Articulator - When using an occlusal record to relate opposing casts, the thickness of the record increases the VDO upon articulation. When the record is removed the condylar guidance inclination would be decreased in the non- arcon articulator creating a potential negative error in excursive movements. 29.Purpose of making plaster index of complete denture -to preserve face bow record 30.why do we remount complete denture-- 31.Lesion that blanches - - it disappears on streching, it is always bilateral. 32.Apexification and apexogenesis 33.Tooth responds with lingering pain to cold - irreversible pulpitis with normal periradicular tissue; irreversible pulpitis with chronic periradicular tissues 34.Hallmark of periradicular abscess - sinus tract 35.Hallmark of acute periradicular periodontitis - sensitivity to percussion 36.Rubella embryopathy - also called chronic rubella panencephalitis.occurs in early stages. a. http://books.google.com/books?id=uRg- cYkTT1AC&pg=PA147&lpg=PA147&dq=rubeola +embryopathy&source=bl&ots=AZ7lTSnhHA&sig=oGAF6VDLHlmLXpqHo OhGdCj_qV8&hl=en&ei=Qu91S_WXMYnkswO24cnKCA&sa=X&oi=book_ result&ct=result&resnum=10&ved=0CCsQ6AEwCTgK#v=onepage&q=rub eola%20embryopathy&f=false 37.Periodontium surrounding the implant - no periodontium, bone and implant. 38.Differentiate perio and pulpal abscess- Endo to perio lesion is outward, Perio to Endo lesion is inward. 39.Reliable test for teeth with open apex - heat, cold, ept--- same when tooth has gold , or porclain crown 40. http://www.endomail.com/articles/blm03properdiagnosis.html 41.Organisms in - : Aggresive perio - Loclaized: AA, Gen is , Eikenella corrodens 42.Midline swelling all except - a. midline swellings - nasopalatine, median rhomboid, thyroglossal duct, dermoid cyst, 43.granuloma + bleeding + apthous ulcer -: ulcerative colitis, chrons disease 44.Question of mucocele and ranula 45.CT and MRI recognition 46.how to prevent proximal displacement of Cl II filling - retention grooves 47.how to increase the resistance and prevent from fx of the restoration: dove tail it provides the retention form 48.Coefficient of thermal expansion is most for which material - tooth

Situation Agent Regimen – Single dose 30–60 minutes before procedure Regimen – Single dose 30–60 minutes before procedure Regimen – Single dose 30–60 minutes before procedure Situation Agent Adults Children Children Oral Amoxicillin 2 gm 2 gm 50 mg/kg Unable to take oral medication Ampicillin OR 2 g IM or IV* 2 g IM or IV* 50 mg/kg IM or IV Unable to take oral medication Cefazolin or ceftriaxone 1 g IM or IV 1 g IM or IV 50 mg/kg IM or IV Allergic to penicillins or ampicillin – Oral regimen

Cephalexin**† 2 g 2 g 50 mg/kg Allergic to penicillins or ampicillin – Oral regimen

OR

Allergic to penicillins or ampicillin – Oral regimen

Clindamycin 600 mg 600 mg 20 mg/kg Allergic to penicillins or ampicillin – Oral regimen

OR

Allergic to penicillins or ampicillin – Oral regimen Azithromycin or clarithromycin 500 mg 500 mg 15 mg/kg Allergic to penicillins or ampicillin and unable to take oral medication Cefazolin or ceftriaxone† OR 1 g IM or IV 1 g IM or IV 50 mg/kg IM or IV Allergic to penicillins or ampicillin and unable to take oral medication Clindamycin 600 mg IM or IV 600 mg IM or IV 20 mg/kg IM or IV ####2.2 mg of Naf will provide 1 mg of flouride.its a standard memorize it been calculated according to the atomic weight of both sodium and flouride

1. When will the BULL rule be utilized with the selective grinding a. working side b. balance side c. protrusive movement d. all of the above 2. Pan showing lucency going inferior over the body of mandible close to the angle. Informed the patient was involved in an accident. Identify the lucency a. pharyngeal airspace b. fracture c. artifact-retake radiograph 33. There are more detached plaques within supragingival plaques that subgingival plaques. The detached plaques within subgingival area are the ones that are more toxic to tissue than attached plaques a. both statements are correct b. the first statement is correct but not the second c. the first statement is wrong, but the second statement is correct - supragingival plaque is always attached and subgingival is unattached. d. both statement are wrong 8. Organism implicated on causing severe spreading abscesses include a. Fusobacterium b. Campylobacter c. Enterococci d. Bacteroides --- black pigmantation unbundling, Bundling, Up coding, down coding: The American Dental Association (ADA) defines unbundling of procedures as "the separating of a dental procedure into component parts with each part having a charge so that the cumulative charge of the components is greater than the total charge to patients who are not beneficiaries of a dental benefit plan for the same procedure." Looking at this issue from another perspective, bundling is the exact opposite of unbundling and can occur on the insurance carrier end. Bundling is defined by the ADA as "the systematic combining of distinct dental procedures by third-party payers that results in a reduced benefit for the patient/beneficiary." Upcoding or overcoding is defined by the ADA as "reporting a more complex and/or higher cost procedure than was actually performed." Downcoding on the other hand is defined by the ADA as "a practice of third-party payers in which the benefit code has been changed to a less complex and/or lower cost procedure than was reported except where delineated in contract agreements." Recording of retro mylohyoid area during border molding - muscles in this area are Superior constrictor(directly) medial pterygoid and glossopalatinal(indirectly) On the buccal of mandibular border molding - u record Buccinator when the pt opens his mouth. When the posterior border(distal of tuberosity) of the max denture is thick, denture dislodges as the pt opens, reason is coronoid process hits the denture. Retro mylohyoid area - directly superior constrictor Gardner's, Peutz Jeghers syndromes have intestinal polyps. OKC has scalloping of radiolucency around the roots Black women, middle aged , anterior radioluceny(can be radio opaque) is cemento osseous dysplasia. Cafe au lait spot - Peutz Jeghers syndrome, Albright's syndrome, Basal cell nevus bifid rib syndrome(gorlin - goltz syndrome), Jeff's syndrome (severe form), neurofibromatosis Peutz Jegher's syndrome - addisson's disease Mucles responsible to record retromylohyoid area:direct A study is designed to determine the relationship bet. emotional stress and ulcers .To do this , the researchers used hospital records of pt's diagnosed with peptic ulcer disease and pt. diagnosed with other disorders over the period of time from july 1988 to july 1998 . The amount of emotional stress each pt. is exposed to was determined from these records. This study is A) Cohort B)Cross-sectional C)Case-study* D)Historical Cohort E)Clinical Trial

4. Why is the surgical stent required for an immediate denture? a. to give an idea of the anatomy of the region b. prevent hematoma c. to determine occlusion Ans: in immediate dentures its there to give an idea of the anatomy of the region NOT to determine occlusion that too is in the prosthodontics decks

5. Which tooth will the matrix band be a problem with when placing a two surface amalgam? a. mesial on maxillary first molar b/c of the cusp of carabelli b. distal on maxillary first premolar c. mesial on maxillary second premilar d. distal on mandibular first molar ------Mesial Of max premolar> Distal of max molar>.... 11.Asprin stops pain by a. stopping the unpward transduction of pain signal in the spinal cord b.intefere wiht signal intrepretation in the CNS c. stopping the local signal produtction and transduction d.stopping the signal transduction in the cortex Q's from A -Maximum dose of mepivicaine? 400mg Note: 400mg for prilocaine,300mg for lidocaine without epi,300mg for lidocaine with epi,90mg for bupivacaine

-Know about the following: Autonomy, maliciousness, nonmaliciousness, cohort, clinical trial

-Which of the following will heal in 4-6 weeks? Symphysis- Body Condyle Angel

-Pt on kidney dialysis, when can you perform TE? One day after dialysis Note: Pt with end stage renal disease(ESRD) >>> 1)are on steroid therapy 2)prone to bleeding 3) do NOT use NSAID because they cause nephrotoxicity

-Action of Listerine? it disrupts adhesion of bacteria to plaque is a phenolic compound Chlorhexidine :The mechanism of action is membrane disruption LISTERINE :Antiseptic mouthrinse is a broad-spectrum antimicrobial, and it kills bacteria associated with plaque and gingivitis by disrupting the bacterial cell wall.

Note: (Active Ingredients: Thymol(which is a phenol) 0.064%, Eucalyptol 0.092%, Methyl Salicylate 0.060%, Menthol 0.042%)

-Dental anxiety can be caused by Pt’s helplessness. What would reduce it? Telling Pt to raise her/his hand when feels pain

-What does St. John's Wort do? Decrease the body immunity Note: there is no option “anti depressant” in choices. in Pt with HIV it interact with anti HIV drugs such as Indinavir(increase immunity) and reduces their function so the immunity decreases

-Primary risk factor for periodontitis? Tobacco Alcohol Cardiac condition

-By recent studies, which one has correlation with periodontitis? Diabetes - diabetics are 15 times at risk Cardiovascular disease

-A kid is on recall appointment and is not cooperative. You should do voice control followed by? Alternating appraisal

-What is the side effect of pilocarpine (Tx of dry mouth)in toxic dose? Bradicardia and hypotension Apnea Cardiac shock Note: nontoxic side effects>>> excess sweating and salivation, bronchospasm

-Benzodiazepine works on? >>> inhibits GABA inhibitors

-Which of the following potentiates anticoagulant activity ? St. John's Wort Licorice (Tx of Dyspepsia, indigestion, GERD) and Upper respiratory infections Saw palmetto Note: never prescribe with coumadine, Increased anticoagulant effect of warfarin has been reported during concurrent ingestion of saw palmetto.

-Which of the following decreases the salivary flow? Scopolamine(also use for nausea and motion sickness)

-Know about lidocaine calculation

-Study group A and B give some agents for plaque control then compare which agent is more effective. Which study is that? Clinical trial

-Which of the following images shows better the mid-facial fracture? Pano >>>> best for mandible fracture CT might be Waters(occipito-mental projection)>>> IS THE BEST VIEW TO EVALUATE THE FACIAL FRACTURE and maxillary sinus Reverse towne’s>>>> for condyle fracture Submentovertex>>>>for zygomatic fracture

-Tx for ClassII furcation involvement (called cul-de-sac)? guided tissue regeneration Note:2nd maxillary molar has the worst prognosis in furcation involvement

-Know about difference between regenerative surgery and flap surgery? regenerative surgery - for regeneration with bone graft flap surgery - to get acess for better srp

-Pt is on rehab of cocaine. what you prescribe for pain? codeine advil

-Disable Pt comes in and not cooperative, how should you act? Permissiveness (give Pt freedom and treat in the way Pt feel comfortable) Consistency

-EPT>>> not in primary teeth

-Ankylosed tooth diagnoses best by? Percussion test(metallic sound) See on xray a thick lamina dura and no PDL space

-Know about apexification, apexogenesis, EPT versus thermal test(when false negative when false positive)

-what prevents corrosion in base metal alloy? Cobalt Chromium Zinc - increases hardness Tin Tn and In helps in - porcelain bonding Zinc increase Hardness -Know about ANB ,SNA,SNB SNA=82* SNB=80* Negative ANB>>> classIII ANB>4* then ClassII Cl 1 average 2 ; range 0-5 degree Cl 2 average 7 ; range 2-12 degree Cl 3 average -1 ; range -6 -0 degree

-Cheek biting caused by? Decreasing posterior horizontal overlap Note: Cheek biting causes by: Posterior teeth placed edge to edge. Tx: reduce facial of mand molar Inadequate VDO. Tx: new denture

-Biting on the corners of the mouth? Reset canine position

-lip biting may be due to the following: Reduced muscle tones Large anterior horizontal overlap

-Tongue biting caused by: Having posterior mandibular teeth too lingually (increasing posterior over jet)

- two different drugs with same dosages bind to the same receptor and cause same intrinsic affect however they have different affinities for the receptor: In which aspect these 2 drugs are similar? Efficacy ED50 Potency

-What reverse alcoholism? Antabuse(disufiram) it inhibits aldehyde dehydrogenese. Metronidazole inhibits this enzyme as well. Antabuse (disulfiram) interferes with the metabolism of alcohol resulting in unpleasant effects when alcohol is consumed.

-After caries removal sound tissue is on . How do you restore? Build up with GI and place composite

-Periapical lesion caused by all except? Occlusal trauma Maxillary sinusitis

-Sings of thyroid crisis? Diaphoresis(excess sweating),tachycardia,fever Bradicardia Untreated hyperthyroidism : atropine and excesive amounts of Epi should be avoided Thyrotoxic crisis signs Early symptoms: restlessness, nausa, abdominal cramp Later symptoms: high fever, diaphoresis, tachycardia, cardiac decompensation, finally Pt becomes hypotensive.

Note: Betadine contraindicationes: Not for use in children under 6 years of age and in patients with a known or suspected iodine hypersensitivity. Regular use is contraindicated in patients and users with thyroid disorders (in particular nodular colloid goitre, endemic goitre and Hashimoto's thyroiditis). -What differentiate hypothyroidism from hyperthyroidism and? Fever Cold to touch Low BP

-All are the actions of vasoconstrictor in one carpule of local anesthesia except: Minimizes toxicity and facilitate hemostasis Reduces the rate of absorption Increases duration of action What will prohibit the mesial drift of tooth toward edentulous area? Proper axio-occlusal contact(opposing and adjacent tooth) 2-Diabetic Pt under NO2 Tx. Precaution? Medication? Change in food? 4-in Neurapraxia which one is affected? -Axonal membrane -perineurium. -Nothing is affected A relatively mild form of nerve injury caused by compression of a nerve. It involves no structural damage to the nerve axon, although the myelin sheath may be temporarily disrupted. I is characterized by temporary loss of nerve function, tingling, numbness, and weakness. It usually heals quickly 5- least chance of needle injury? -cleaning up -recap -setting up 6-which one is less sedative? -promethazine(Phenergan) -Benadryl (Diphenhydramine) increase in sleep duration chlorpheniramine>promethazine>diphenhydramin. Phenergan is widely used to treat nausea/vomiting and comes in an injectable, can only be obtained with a prescription. Benadryl, commonly used for allergies (itchy eyes, runny nose,sneezing) Benadryl can be bought over the counter for as needed use.

9-mode of action of antihistamine? http://en.wikipedia.org/wiki/Histamine_receptor 10-radiographic appearance of crater defect? An osseous crater defect is a concavity in the crest of interdental bone confine within facial and lingual walls 12-Study among smokers and nonsmokers in a period of 6 years(e.g 2000-2006) to develop disease? -cohort -Cross sectional 14-which race has a higer F in DMFT index? Whites 15-picture of midline of floor of the mouth,the color is NOT blue -ranula -dermoid cyst 17 - Frey's Syndrome (Auriculotemporal Syndrome) neurological disorder that results from injury or surgery near the parotid glands , damaging the facial nerve. This syndrome is characterized by flushing or sweating on one side of the face when certain foods are consumed. Auricutemporal nerve a branch of V3,this nerve supplying sensory fibers to preauricular and temporal areas, also carries parasympathetic fibers to the parotid gland (initially this fibers carried by glossopharyngeal up to Otic ganglion) and sympathetic vasomotor and sudomotor(sweat-stimulating) fibers to periauricular skin. After parotid surgery or trauma the parasympathetic fibers may be severed. In their attempt to re-establish innervations ,these fibers occasionally become misdirected and regenerated along sympathetic nerve pathway. 18-Comparision between 2 proportion, which test we do? -t test: camparison of mean value of two groups - difference between 2 means; this can be compared between, control group and tx group or tx A vs. Tx B ... -chi square test - measures the association between 2 variables and comparision of groups when the data are expressed as counts or proportions. 19-in which syndrome Pt has multiple odontoma? Gardner's syndrome and esophageal stenosis syndrome 20-which syndrome Pt has calcified falx cerebri, multiple okcs, bifid ribs? - Gorlin Goltz syndrome or Basal cell nevus disease or basal cell bifid rib syndrome. - Nevoid basal cell carcinoma syndrome(Gorlin-Goltz syndrome) 21-radiographic projection from the base of the skull - submentovertex projection(The zygomatic arches stand out like the handles of a jug on this view)

ahooye vahshi 1. Which population has the most number of UNRESTORED caries a. black Note:unrestored >>>>black untreated caries >>>>blacks untreated caries in kids is hispanics filling >>>>white 2. A lesion that has re-mineralized will be a. Darker that the adjacent enamel b. Rougher and cavitated c. Softer d. Will be more difficult to penetrate in the future 3. A class II caries is a. Apical to contact b. Gingival to contact Both has same meaning??? 4. Caries exists below critical pH of 5.5 5. What is the bacteria that is not initially involved in caries but plays an important role? LACTOBACILLI>>>> gram positive facultative anaerobe, the # of this species has been used as a caries test 8. What is Gingival Plaque Index? a. Nominal like mild, moderate, severe b. Ordinal include numbers: like furcation involvement 1,2,3 c. Interval like Celcius degree d. Ratio e.g Kelvin degree, or BP measurement(can not be zero), PH, length(can not be negative),weight Note:gingival index: both ordinal and nominal 9. Pit and Fissure caries is described as two cones: a. Two bases are pointing toward the pulp b. Two apexes are pointing toward pulp>>>> in smooth surface (proximal caries) c. One apex toward the pulp and one base toward DEJ d. Both bases facing DEJ 11. How is transillumination helpful when examining a kid? a. Koplik spots b. Herpetic gingivomatitis 12. Difference between Infected and Affected dentin (affected dentin has bacteria present but smaller amounts and can be reversed. Infected dentin has large amounts of bacteria and needs to be removed because non-reversible) 13. The dentist accidently did not see Occlusal caries and he placed a sealant over it; Will the caries a. Stop b. Rapid Progression c. Slow Progression 14. What does caries detector stain? a. Denatured collagen b. gram negative bacteria Note: Research indicates that the dye in the caries detector bonds to the denatured collagen which is present in the outer infected dentin but which is not present in the inner uninfected dentin and normal dentin 15. Recently placed gold inlay; what is the most common reason for pain afterwards? a. Galvanic shock Sensitivity - choose this if only q says opposing dissimilar metal, other wise it is Fracture of the tooth has to be suspected 16. Everything cannot be controlled with light activated composites EXCEPT: a. Shrinkage b. How far polymerization goes 17. Control Polymerization 18. For a resin bonded FPD (Maryland Bridge) everything is necessary EXCEPT: a. Wrap around proximal b. Draw c. Occlusal rest d. Chamfer e. Bevel

19. How is bioavailability measured? a. How much drug is absorbed in the circulation b. Blood to urine ratio Note: efficacy of drug: level of binding a drug to its receptor 20. There are two drugs that with the same dosages bind to the same receptor and have same intrinsic affect however different affinities for the receptor: How are these two drugs the same? a. ED50 b. LD50 c. Potency d. Efficacy 21. Question asking the definition of antagonist vs. agonist; the answer choices also included partial agonist and partial antagonist 22. Therapeutic Index = LD50/ED50 or TD/ED and higher therapeutic index is better because wont do as much harm Note: A drug with high LD50 and low ED50 has high therapeutic index, therefore relatively safe. 23. When you stimulate alpha 1 receptors what happens? a. Vasoconstriction more reasonable b. Hypertension 24. There was a question that said something like how will Propranolol (is a non- selective beta blocker) affect the following? a. Ephedrine induced mydriasis (contraction of papillary muscle)>>> ephedrine is Sympathomimetic drug .with anesthesia causes hypotension. it suppress apetite(use for wait loss) b.   Phynylephrine-induced hypertension>>>> is alpha 1 receptor agonoist 25. There was a question that said something about how benzodiazepines have a anxiolytic effect: in moderate doses ANTIANXIOLYTIC and high doses is SEDATIVE 26. Sedative rebound (or something like that) a. Antiphysicotic 27. Some question about Atropine à treat bradycardia, and control excess salivation during dental procedures (Atropine-most common, also exhibit mydriasis (dilated pupils), contra for nursing mothers and patients with Glaucoma) 28. Zyban (bupropion) is used for smoking cessation. 29. Sodium Thiopental rapid-onset short ultra acting barbiturate(IV) for general anesthesia- for De sensation 30. What receptors do benzodiazepine act on? GABA 31. What is the most common type of seizures found in children? a. Grand mal b. Juvenile absence seizures c. Simple partial seizure- if there was not option of Febrile go with this Febrile seizure: A convulsion that occurs in association with a rapid increase in body temperature. Febrile seizures are common in infants and young children and, fortunately, are usually of no lasting importance. A febrile seizure occurs when a child contracts an illness such as an earinfection, cold, or chickenpox accompanied by fever. Febrile seizures are the most common type of seizure seen in children. Two to five percent of children have a febrile seizure at some point during their childhood. Why some children have seizures with fevers is not known, but several risk factors have been identified. Note: One of the most common seizure types is a convulsion. This may be called a "tonic clonic" or "grand mal Febrile (A febrile seizure is a convulsion in a child triggered by a fever). The movements are the same as in a grand mal seizure. 32. tricyclic Antidepressants ( Amitriptyline, Imipramine, Trimipramine)Tricyclic antidepressants (TCAs) inhibit the reabsorption (reuptake) of serotonin and norepinephrine by brain cells. To a lesser extent, TCAs also inhibit reabsorption of dopamine 33. Lithium- bipolar disorders 34. What do you often need to supplement with diuretics? Potassium Potassium sparing Diuretics : Spirinolactone , Triamterene, Amiloride 35. Which of the following ACE inhibitor? (look for something ending with –pril) like lisnopril 36. Which of the following drugs causes gingival hyperplasia? a. Calcium channel blocker : Nephidipine , cyclosporines 37. Patient comes in and is on Coumadin, what do you do? a. Stop for 1 day b. Stop medication of 3 days c. Do not need to stop medication 38. What drug is used for ANUG? a. Tetracyclin b. Penicillin ANUG debriment+peroxide ------in HIV use antimicrobial--- no attachment loss 40. Child comes in with an oral infection and is NOT allergic to Pen. What do you prescribe? a. Penicillin b. Amoxicillin c. Tetracyclin i. amoxicillin .. why not penicillin? 41. What drug has cross allerginicity with Penicillin? Cephalosporin- both have Beta lactamase ring. If pt has allergic to penicllin then pt has allergy to cephalosporin 42. What is the effect of Tetracycline? (on protein synthesis 30s) 43. What is the effect of Penicillin and Cephalosporins (cell wall synthesis) 44. Which drug should not be used with someone with Liver damage? a. Tylenol sulfonamides They are bacteriostatic rather than bactericidal 49. Know the effects of histamine and that it is derived from histidine? Note: histamine is bronchospastic and vasodilator 50. Cimetidine- H2 Blocker(reduce the acid secretion) for GERD(gastro esophageal reflux disease) 51. There was a question that said which drug would inactivate the latter? a. Antacids- Tetracycline note: Do not take iron supplements, multivitamins, calcium supplements, antacids, or laxatives within 2 hours before or after taking tetracycline. Antacids and milk reduce the absorption of tetracyclines. 52. In a gold MOD onlay, how are the axial pulpal walls? a. Converging b. Diverging 53. When pouring up a cast which of the following has the least wet ability with a slurry water? a. Polyether hydrophilic ,no byproduct b. Polysulfide byproduct>>> H2o c. Condensation Silicone poor wet ability, byproduct>>> ethyl alcohol d. Irreversible Hydrocolloid 54. A patient with stable posterior occlusion and only missing maxillary incisors you are making a bridge from #6-#11. What do you need to send to the lab? a. Semi adjustable articulator in CR with facebow, lateral records, and incisal guide b. Semi adjustable articulator in MIP with facebow, lateral records, and incisal guide c. A simple nonadjustable articulator in MIP 55. What is the purpose of a facebow to set the Maxillary arch to the a. Terminal hinge axis b. Mandibular arch 56. When looking at a patient from a vertical view, how many planes on their face? a. 3 (Horizontal view) b. 5 57. When a person is in physiological rest position….. something about interocclusal distance a. freeway space which is 2-3mm 58. Angular chelietis is caused by all of the following except: a. Fungal infection b. Decreased VDO c. Increased VDO d. Other options Note : Nutritional deficiencies- most common 59. 2 questions on working and non working interferences Note: BULL rule is for working side 61. What is the function of Hex on implants? a. To screw on implant b. anirotational 62. If there is an implant that is 4mm in width at least how many mm does the labiofacial bone need to be? a. 6mm b. 7mm c. 8mm 63. How does titanium of an implant help in osseointegration? a. Forms Titanium oxide layer b. Tensile strength 64. What is the cause of cheek biting in a complete denture? a. No enough horizontal overlap 65. A post palatal seal (post dam) is used for a. Tissue displacement b. Polymerization shrinkage of acrylic 66. Indirect retainer is used to prevent distal extension dislodgement away from the tissue TRUE 67. The flexibility of a clasp is determined by everything except? a. Length b. Diameter c. Shape d. Type of Metal e. Depth of undercut 68. What is the strongest type of metal a FPD can be made from? a. Type 3 b. Type 2 c. Type 4 FPD

"Crown and Bridge" Gold Alloys (Non-ceramic) ∗ Type I (soft) - min. 83% Noble Metal ∗ Type II (medium) - min. 78% Noble Metal ∗ Type III (hard) - min. 78% Noble Metal ∗ Type IV (extra hard) - min. 75% Noble Metal ∗ Type I - small inlays; very slight stress ∗ Type II - inlays, thick 3/4 crowns, complete crowns ∗ Type III - thin 3/4 crowns, abutments, pontics, complete crowns, short-span FPD's ∗ Type IV - RPD Frameworks, long span FPD's 69. Know everything about Kelly (Combination Syndrome) Combination syndrome was first described by Kelly1 as destructive changes in hard and soft tissues of patients with complete maxillary denture opposing an unstable bilateral free-end mandibular partial denture.2,3 The long-term result is extrusion of the remaining mandibular anterior teeth and the alveolar process surrounding them with loss of posterior mandibular bone. The plane of occlusion becomes reversed. Papillary hyperplasia of the hard palate develops. The premaxilla becomes atrophic as a result of the force exerted on this soft bone during occlusion. The maxillary tuberosity develops hypertrophy, creating a limited interarch space. If not corrected, the unstable occlusion can result in progressive posterior mandibular atrophy leading to greenstick fractures. The method of reestablishing a proper occlusal relationship is discussed in this article using a conventional maxillary denture and fixed mandibular implant restoration to correct the occlusal issues. 70. What do a veneer and all ceramic crown have in common? a. Incisal reduction b. Rounded internal c. Facial reduction 71. Biological width- 2mm why then we say we want the margin of prep 3mm away from crest of the bone? because of 1mm distance from the attached gingiva 72. You have an FPD and when you seat it, you want the pontic to: a. Blanch the tissue definitely NOT!! b.  Other options what we need to see??? 73. What is an active screw (post) vs. inactive post? 75. A resin bonded FPD in not seated all the way or is not stable or something like that. What could be the reason? a. microleakage 76. There is a veneer which is bonded with resin and the patient comes back after a month or so with a dark stain near margin,reason? a. Microleakage 77. When you want to seat a crown, the following contribute to it fitting passively EXCEPT: a. Die spacer b. Increasing gypsum investment material c. Fit checker 78. When you receive a crown back and want to seat it what is the first thing you check for? a. Shade (Aesthetics) b. Proximal contacts c. Margins 79. Know about metamerism and how it can affect color based on the light source? Note: The quality of some colors that causes them to appear differently under different light sources. For example, two color samples might appear the same in natural light, but not in artificial light. 80. If a kid comes in and has trauma to the face and the IA is damaged, where did the kid get hit? a. Angle of mandible IN KIDS: most mandibular fractures occurred at the condyle (55%), followed by the parasymphysial region (27%), then the body (9%), and angle (8%) IN ADULTS: Condyle : 29% > Angle 24.5%>Symphysis 22%> Body 16%> Ramus 1.7% > Coronoid 1.3% 81. Know the Maximum dose of lido with epi is 7mg/kg for an adult Note: text says 5mg/kg for (According to Malamed 4.4mg/kg) and 7mg/kg for articaine 82. Adding a vasoconstrictor to local anesthesia does all the following EXCEPT: a. Decreases rate of absorption b. Increases duration of action c. Minimizes toxicity and helps homeostasis d. all of above 83. Max amount of Nitrous Oxide for a kid a. 40 % b. 50% c. 70% Adult 84. The maximum amount of nitrous on the machine safety hinge is 70% 85. What does band and loop NOT do? a. Maintain space b. Does NOT create a vertical stop 86. Patient with white palate a. Patient wears denture all night 87. What is the average pulse rate for a child? a. 60 b. 80 c. 100 Note: depends on age Newborn (1d to 1yr)= 100-160 1yr-6yr= 60-140 6yr-11yr= 70-100 11yr and up= 60-100 89. Loss of which tooth important in retaining space a. Primary second molar 90. What do you do if mandibular central incisor is erupting lingually? a. extract primary teeth and use appliance to move forward b. Leave it was an option note: if primary retain more than 8 yrs of age then extract and place appliance to reposition permanent mandibular anterior 91.which of the following is most likely to be interpreted as by Pt? a.maxillary sinusitis b. tmj dysfunction c. otitis media Maxillary sinusitis - can cause pain or pressure in the maxillary (cheek) area (e.g., toothache, headache) 92.of the following which is most likely to have reffered pain? a.acute apical periodontitis b.irreversible pulpitis c. 93.when presents on radiograph a.normal pulp b.pulp has been injured in the past but has recovered Some authors believe that pulp calcification is a pathologic process related to various forms of injury, whereas others regard it as a natural phenomenon. 94.lateral is best differentiated from the acute apical abscess by? a-pulp testing b.radiographic appearance c.probing patterns 95. radiographically the acute apical abscess a. is generally of larger size than other lesions b.may not be evident c.has more diffuse margins than other lesion Epidermology question : A study is done to determine the affectiveness of a new antihistamine .To do this , 25 allergic pt’s are assigned to one of the two groups ,the new drug (13 pt’s) , placebo (12 pt’s) . The pt’s are followed for 6 months . This study is called 1. cohort 2.cross-sectional 3case-controlled 4.historical cohort 5. clinical trail ANS 5 ( assigned or give is the clue ) -Study among smokers and nonsmokers in a persons of 6 years(e.g 2000-2006) to develop disease? -cohort -Cross sectional ------Date: 02/15 1. which is a disorder commonly seen in sjogren’s? (xerostomia & altered glands not listed) • Rheumatoid arthritis • Xerostomia (dry mouth) • Keratoconjuctiva sicca (dryness of the eyes) 9. Which articular disease most often accompanies Sjögren’s syndrome? A. Suppurative arthritis. B. Rheumatoid arthritis. C. Degenerative arthrosis. D. Psoriatic arthritis. E. Lupus arthritis. 2. contraindication for max molar with class 2 furcation? hemisection wI crown hemisection = mand molar Mandibular molars to treat Class II or III furcation invasions For max molar - we do root amputation

3. what substance has corrosion resistance in pfm metal? Cobalt- chromium and are more rigid in comparison to gold and palladium (mosby’s 331) base metals (also called non-precious metals) are less resistant to corrosion. Base metal alloys are based on active metallic elements that corrode, but which develop corrosion resistance via surface oxidation that produces a thin, tightly adherent film, which inhibits further corrosion. Example: Cobalt-chromium alloys form a Cr20 3 oxide film, which passivates the surface. (Decks) Base metal alloy advantages are principally found only in their strength and low density.

4. how does fluoride play role in decreasing caries? makes enamel stronger (idiots)!

Mechanisms of action for fluoride in caries inhibition: (Decks)

1. The topical effect of constant infusion of a low concentration of fluoride into the oral cavity is thought to increase remineralization of enamel.

2. Fluoride inhibits glycolysis in which sugar is converted to acid by bacteria. Specifically, fluoride ion inhibits the enzymatic production of glucosyltransferase.

3. The incorporation of fluoride into the enamel hydroxyapatite crystal. Fluoride ions replace the hydroxyl radicals of the hydroxyapatite crystals in the enamel, producing fluorapatite. This form of enamel is less soluble in catabolic acids produced by oral bacteria.

4. Recent research suggests a bacteriocidal action ~ topical fluoride may also prevent caries by directly interfering with the growth and metabolism of cariogenic bacteria - organisms such as streptococcus mutans that produce acids responsible for decay. 5. what sleep med do you give to a pt the night before a dental appt?

Value is the single most important factor in shade selection. Intensity is included in the term value. Important points about staining porcelain: • Stains are metallic oxides that fuse to the porcelain during a predetermined firing cycle. • Drastic changes of the hue (color or shade) are often impossible. Orange stain is the most often used to change the hue. • Staining a porcelain restoration will reduce the value (as will using a complementary color). It is almost impossible to increase the value. • Chroma can be successfully increased by the use of stains. particularly in the gingival area.

6. tooth #30 is endo tx with restoration, pt is in pain when he bites, why? cracked tooth

Resorbable sutures evoke an intense inflammatory reaction. This is the main reason neither plain gut or chromic gut are used for suturing the surface of a skin wound. When suturing an extraction site in the anticoagulated patient, a non-resorbable suture is recommended.

Suture size is based on strength and diameter. This system uses "0" as the baseline, average size suture. As suture diameter decreases, "a's" are added or numbers followed by a "a" (for example, 000 and 3-0 are the same size). As suture diameter increases above "a," numbers are assigned to the suture material. Because suture material is foreign to the human body, the smallest- diameter suture sufficient to keep the wound closed properly should be used. Most oral and maxillofacial surgical procedures require the use of 3-0 or 4-0 sutures.

7. what meds do you take for asthma? Bronchial relaxation, airway dilation (asthma) - (Beta - adrenergic agonists – albuterol, salmeterol and metaproterenol

8. what does sodium hypochlorite do? disinfectant. germicidal solvent action, 5.25% solution provides excellent germicidal solvent action

9. what is a sign of ectodermal dysplasia? retained teeth Ectodermal dysplasia is a hereditary condition characterized by abnormal development of the skin and associated structures (hair. nails, and teeth, and sweat glands). It involves all structures which are derived from the ectoderm. It affects males more than females. Common clinical findings include hypothrichosis (decrease in hair), anhidrosis (no sweat glands, leading to heat intolerance), or oligodontia (complete or partial absence of teeth), depressed bridge of nose, lack of salivary glands and the child appears much older than what he or she is.

10. all these meds decrease saliva except? pilocarpine Pilocarpine (Sa/agen) is used to stimulate salivary flow in patients suffering from xerostomia due to radiation therapy in the treatment of head and neck cancer.

11. pt has round bump on midline of floor of mouth, what is it? ranula The ranula, a true retention cyst, characteristically occurs in the floor of the mouth and is unilaterally located.

12. patient is getting front tooth fixed ... whats the purpose of using an incisal guide table? Generally made of acrylic resin and are made to reproduce the surface of teeth (usually lingual concavity and incisal edges of ant teeth) tha have a direct influence in guiding the mand. Through ALL EXCURSIVE movements (Mosby’s 318) incisal guidance plays as important a role as the temporo-mandibular articulation in establishing a functional and harmonious occlusion, as much on the anterior teeth as the posterior teeth.

17. which is least likely to occur with occlusal trauma? gingivitis On the trauma of occlusion -----gingival reccetion

Radiographic signs of trauma from occlusion: • Widening of the periodontal ligament space • Sometimes thickening of the lamina dura • Angular bone loss and infrabony pocket formation • Root resorption • Hypercementosis Trauma from occlusion is reversible,

Other findings associated with excessive occlusal forces: • Altemating areas of resorption and repair of the alveolar bone •. Fibrosis of the alveolar bone marrow spaces • Cemental resorption leading to dentinal resorption • Cemental tears • Possible ankylosis Periodontal pocket formationàis an inflammatory lesions and are not caused by occlusal trauma or bruxism

1. most common characteristic of ? bilateral jaw swelling 2. 4. normal occlusal wear shows what? Attrition 5. what substance has corrosion resistance in PFM metal? chromium 7. what sleep med do you give to a pt the night before a dental appt? ambien CR (zolpidem tartrate) 8. 19 ylo girl has halatosis, interproximal recession and bleeding ... because? ANUG 11. best way to suture an incision? interrupted suture 15. best way to determine outcome of disease? A. med history of the patient (If the lab test was choose may be that) B. ESR LAB result 16. best reason to do an onlay? cuspal coverag 17. what is a sign of ectodermal dysplasia? anodontia, aligodonsia 19. mechanism of opiods? attach to mu receptors 20. main advantage of using GI cement? a. Fluoride Release 33. the drug enforcement agency is concerned with what? potential for abuse 34. where are you most likely to perforate on a maxillary central incisor? Buccal 37. what do you use to SC/RP implants? pl 38. dentist restored tooth and accidentally left caries ... what happens to caries? caries is arrested 39. where are the primate spaces? 40. osteoporosis is associated with which of the following diseases? Hyperparathyroidism 41. do not give which medication to lactating female? codiene and tetracycline 43. two things that account for a successful posterior composite restoration? a. type of resin and size of tooth b. size of tooth and type of prep c. type of resin and type of prep 44. what is the initial step when you have an acute perio abscess? a. incision and drainage b. anti-biotic therapy c. antibiotics then incision 45. how do you repair a porcelain veneer with composite? a. microetch, etch and silane b. sandblast, etch and pumice c. pumice, silane, etch 46. which is the best systemic anti-fungal? a. ketoconazole, and others areamphotericin B and fluoconozole b. nystatin c. miconazole d. clotrimazole 47. conical shaped caries w/ broad base with apex towards pulp is commonly seen in? a. root caries b. smooth caries c. pit/fissure caries 48. best clinical determinant of root caries a. sensitivity to cold b. sensitivity to sweets c. soft spot on tooth - visual and tactile methods are used for detect caries 49. 40 yo pt w/ all 32 teeth. No cavities. Has stain & catch in pit of molar. what do you do? a. watch and observe b. sealant c. composite 50. 12 yo kid w/ carious lesion on tooth #9 that shows pulp exposure w open apex... what do you do? a. rct b. apexiflcation 51. what is most numerous at site of chronic inflammation? a. plasma cells b. mast cells c. macropahges d. leukocytes What is the name of the federal funded medical care for the elderly and its coverage? a. medicare wI dental coverage b. medicare wlo dental coverage c. medicaid wI dental coverage d. medicaid w/o dental coverage 55. main difference and advantage of using GMT instead of Enamel hatchet? a. bi-angled cutting surface b. angle of the blade c. push/pull action instead of 56. tooth #30 has huge MOD amalgam and is deep. Hurts pt when he eats french bread. what is the cause? a. root fracture b. acute c. decay/abcess 57. when do you do calcium hydroxide therapy in an avulsed tooth? a. wait 2 weeks b. asap c. do it first and then replant tooth 58. chronic periodontitis most likely found in? a. black males b. black females c. hispanic males ♠♠d. hispanic females 59. what branch off facial nerve gets damaged the most during TMJ surgey? a. buccal b. temporal 60. when pt has mouth open, what oral structure would inhibit from capturing buccal flange when taking impression? a. masseter b. buccinator c. coronoid process d. forgot the other option Note: Masseter an buccinator are not functioning during opening of mouth. coronoid process has direct involvement in max molar buccal area during mouthopening and can interfere with impression taking. SANATH et al. 2010. Los Angeles. 61. surgical dressing is indicated for what? a. protect the wound b. help advance healing process c. stop bleeding 62. you see 3 mand incisors w/1 wide tooth. x-ray shows tooth has 2 canals. what condition is it? a. fusion b. gemination c. d. dilacerations 63. praising, smiling and congradulating is what type? a. social reinforcement b. aversive reinforcement c. behavior conditioning Positive reinforcement: 1-Social reinforcement: Praising, smiling, .... 2-No-Social(Token) reinforcement: Toy, Token 64. what do you do when pt on hospital bed has allergic reaction to iv antibiotics? a. take off iv antibiotics b. give epi 65. what are you most likely to see in a kid who has a mand canine tipped facially? a. gingival recession b. deep overbite c. ectoptic eruption 67. why do you do triangular access on incisors (max central inccisor?) a. to help with straight line access b. help expose pulp horn c. to follow the shape of the crown 68. metastasis to the oral cavity is most likely to end up where? a.lip b. tongue c. palate d. mucosa e. floor of mouth 69. how do you make a crown narrower? a. move line angles towards interproximal b. move line angles more facially 70. not enough room when edentulous pt closes. what do you do? a. tuberosity reduction b. adjust mandibular denture c. adjust maxillary denture 71. what do x-rays depict when viewing osseous craters ... or something? a. overestimates bone reduction b. underestimates bone reduction c. forgot the other choices 72. unconscious diabetic is treated with what? a. subcutaneous sugar b. give him Orange Juice c. give him insulin injection d. 50 % dextrose solution e. IV sugar solution or IM glucagon pt should wake up within 15mins 73. most common place of recurrent caries in a class 2 composite? a. occlusal b. facial interproximal c. lingual interproximal d. gingival 74. what type of bond is composite on tooth structure? a. chemical bond b. mechanical bond c. organic coupling d. adhesion 75. what is the most common reason for composite restoration failure? a. improper design (no) b. rubber dam not placed gingivally enough c. forgot the rest 76. what occlusion class will it be if you have an ANB of -8 degrees? a. class I b. classII c. class III d. class 1\ mod 1 e. class II mod 2 77. trauma to max incisor, x-ray shows no pulp, asymptomatic and no endo lesion. what do you do? a. rct b. observe c. extract 78. main advantage of doing direct composite over composite onlay? a. less shrinkage b. better marginal adaptation - best answer among the options c. greater hardness and wear resistance 79. pt trouble keeping her lower complete denture in because of a low ridge, how to fix this problem? a. ridge augmentation b. place anterior implants c. alveolar bone sectioning 80. mand complete dentures must be extended how far distally? a. just anterior to retromolar pad b. must sit on the retromolar pad c. must be 5mm beyond the retromolar pad 81. pt says "your fees seem high" ... how do you respond? a. "i believe my fees are fair" b. "shut your bitch ass up c. "i got to make a living too, ya know" d. "my fees are comparable to geographic area" e. "we can discuss why fees are so high" 82. an 82 y/o pt comes in with his son, who says his guardian is someone else,who okays certain tx's? a. it's okay as long a next of kin is says it is b. have to get consent from guardian c. as long as informed consent is signed, doesn't matter 83. pt has some condition that had blue sclera ... what disease does the pt have? a. osteogenesis imperfect b. marfan's syndrome 84. a dentist has an ethical obligation to report a colleague is all situations ... except? a. working under substance abuse b. advertising on electronic media c. abusing patients 85. you are extracting a 2nd mand premolar... what injections do you give? a. lingual, buccal and IA b. mental, long buccal and infiltration c. mental, buccal and IA 86. most common condition that occurs in the dental office? a. hypertention b. syncope c. anaphylactic shock 87. pt's max denture made her tissue inflamed and weird, you decide to make her a new denture after? a. you reline her old denture a. you place tissue conditioning material in her old denture c. you surgically remove her tissue 88. you inadvertantly extract the wrong tooth ... what do you do? a. replant it asap b. do rct and then replant it c. wait until next appointment to replant d. place implant asap 89. you extract a molar and bone fragments come out with it... what do you do? a. take out bone fragments and make sure its all gone b. flap back the tissue and see the entire area 90. which does not show empathy to the patient? a. open-mindedness b. sharing personal experiences c. reflection and showing understanding 91. patients with autism will usually show? a. decreased rate of caries b. heightened sense of lights and sounds c. the compassion to interact with people Children with autism are easily overwhelmed by sensory overload. This can cause “stimming” (flapping of arms, rocking, screaming, etc). Autistic children are hypersensitive to loud noises, sudden movement, and things that are felt. 92. a gingival bevel can be placed with all, except? a. sharp fluted instrument b. enamel hachet c. gmt d. thin diamiond bur 93. pt comes in with 1 year baby, how do you do exam? a. parent and dentist are knee to knee, baby's head is in parent's lap b. parent and dentist are knee to knee, baby's head is in dentist's lap 94. in what situation would a pt need to premedicate? a. mitral valve prolaspe b. prosthetic heart valve c. bicuspid valve disease d. rheumatic heart disease 1. pt crown (had for 10 yrs) is lighter than the other teeth ... how do you tx? a. vital tooth bleaching b. replace PFM c. direct composite over crown d. veneer the crown 2. proximal resistance form of amalgam restoration comes from what? a. convergence of buccal/lingual wall b. retention grooves in axiobuccal/axiolingual walls - for proximal resistance c. Dovetail - provides retention form 3. what is the purpose of a surgical stent? a. used as a guide to place implant b. used so the teeth won't move in surgery c. protect the adjacent teeth during surger ------Day: 17 Stages of anesthesia Analgesia /Amnesia Excitement/Delerium : begins with unconsciousness and ends with loss of eyelid reflex ,purposeless movements and hyper-reaction ,dilated pupils, reflex vomiting ,tachycardia and hypertension Surgical anesthesia – working in this stage Medullary paralysis:Cessation of respiration Denture sore throat : palatoglossal and superior constrictor of pharynx is inflammed (pt. cannot swallow) Port-wine stains associated with sturge-weber Cephalosporins – beta-lactam family, anaerobic. A prodrug is a drug made active by metabolism. What happens if you have more water? Less expansion Slower setting Less strength More porosity How much reduction venner in the middle 1/3 of facial? 0.5mm What does vertical pull headgear do? The Vertical Pull Headgear with Chin Cup has been designed specifically to correct vertical and a wide range of facial myofuntional problems, such as open bite. As soon as therapy begins, the Vertical Pull Headgear with Chin Cup holds the mouth closed continuously, reducing interdental habits and tongue protrusion. In addition, the intrusive force on the molars is increased. This appliance can quickly change a mouth breathing habit to nose breathing what is an open-ended question? Allows freedom of response

New patient comes into office, not of record, what do you do 1st visit? Full exam, record probing, med history, impressions. Define osseointegration. Why amalgam fails. Bad prep design. What drug causes dilation of eye or midriasis? Atropine, antimuscarinic How soon do you correct a noticeable ASAP most difficult part of seting denture teeth. What causes greatest occlusal wear? Porclain Amalgam is the strongest to wear What increases in pulp with age? Fibroblasts A couple x-rays telling you to identify landmarks. Not too difficult. Second day was like our tx planning final. Kinda subjective but I felt pretty well prepared. Know premeds and when to do it. When not to do it.

Width of Keratinized gingiva is : Free gingiva + attached gingiva Bone density changes : Subtraction Radiography Occlusal sealants succeed by altering the SUBSTRATE change Pt susceptibility Which of matterial cause less allergy? 1-Lido 2-Procaine 3- epi The most common between five? 1-Papiloma 2- 3-Leiomyoma 4-Lymphangioma 5-Neurofibroma The Most connective tissue tumor : (Not true tumor) TMJ pain are mostly related to: 1- VII 2- V3 3- V2 4-VIII What is the most common site of new cases of oral cancer? mosby 207 1-Palate 2-Tongue 3-Floor of mouth 4-lip Major mechanisms for the destruction of osseointegration are 1. Related to surgical technique 2.Similar to those of natural teeth 3.related to implant material 4.related to nutrition A painless ,well-circumscribed radiolucency and radioopacity in the posterior mandible of 11yrs old boy . what is the differential diagnosis ANS .Ameloblastic fibro -odontoma (If age above 50 ,its Paget's disease also remember Ameloblastoma occurs in oldies) "Ghost teeth " ( only one side of mouth affected) Where do we find most caries? 1- Ocllusal 2- Proximal Dermoid cyst Occurs in the midline in the floor of the mouth ( previously we discussed something else ?) The is a depression of the mandible on the side nearest the tongue. It was previously known as a latent and static bone cyst but is now known as a pseudocyst. The depression usually allows for the presence of a salivary gland Which of the following resembles an epiphysial growth plates? Synchodrosis Sutures Most beneficial aspect to brush tongue is reduce? Halotosis GIngivatis cancer What is diff of 330 and 255 bur? Long- 255 is longer thicker sharper The main reason of breaking of RPD clasp? High Mudule of Elastisity work Work Hardening HIV with recurrent ? Acyclovire Which medication is controbuting to Insomnia and lack of apetite? Dextro Amphethamine How far the brush and floss goes in sulcus? Brush 1mm , floss 2mm Is Propoxyphen contraindicated in pregnant women? 65years with loss denture? implant b) Augmentation c)vestibuloblasti What is the best to clean implant? a-water pick b-tooth pick c-floss d-proxibrush What is hyper `telorism? eyes too far in Gorlin and Down syn. chi test, t-test What give to pt allergy to codein? Propoxyphen What is 4 -7 years old afraid of ? a- pain b-unknown c-dental chai r d- sepration from parent ADHD is most common in? boy Boys have higher rates of ADHD than do girls. Former smoker has less chance of periodontitis compare with current smoker. Examination reveals a soft, fluctuant, tender swelling in the middle of the hard palate. The teeth test vital. Radiographs reveal a radiolucent area projected between the roots of the maxillary central incisors. The cyst that represents the most likely diagnosis is a Nasopalatinal cyst Nasiolabial cyst globulomaxillary cyst The pulpal floor is perforated during access preparation. The best course of action is to a. CONTINUE RCT, REPAIR THE PERFORATION AT A SUBSEQUENT APPOINTMENT ONLY IF ASSOCIATED PATHOSIS DEVELOPS. b .REPAIR THE PERFORATION, SCHEDULE PT FOR ANOTHER APPOINTMENT TO FINISH RCT c. REPAIR PERFORATION,INITIAL CLEANING/SHAPING SCHEDULE PT FOR FINISHING RCT Factors that affect the prognosis of perforation repair include location of perforation, time delay before perforation repair, ability to seal the defect, and previous contamination with microorganisms Immediate repair is better than delayed repair, because delay can cause breakdown of the periodontium, resulting in endoperiodontal lesions that are difficult to manage, and elimination of microbial contamination of the defect and proper sealing are critical to success ------18th tx of ANUG: ultrasonic, debridment, oral rinse chlorhexidine or hydrogen peroxide, ab tetra and metro tx of juvenile periodontitis: ab alone or with sc/rp ------Which of the following drugs best reverses the effects of benzodiazepines? A. Naloxone B. Flumazenil C. Midazolam D. Aminophylline E. Physostigmine Following flap surgery, new junctional epithelium can form on either cementum or dentin. Junctional epithelium is reestablished as early as one week. A. Both statements are TRUE. B. Both statements are FALSE. C. The first statement is TRUE, the second is FALSE. D. The first statement is FALSE, the second is TRUE No answer in courseware Ostectomy is a procedure that involves the E. use of an autograft. F. use of an allograft. G. use of a contiguous graft. H. removal of tooth-supporting bone. I. removal of non-tooth supporting tooth Which of the following drugs is administered orally to treat vaginal candidiasis? A. Fluconazole (Diflucan®) B. Griseofulvin (Grifulvin®) C. Clotrimazole (Mycelex® Troche) D. Miconazole (Monistat®) E. Nystatin (Mycostatin® osteotomy vs ostectomy------supporting -proper ------NOTES ENDODONTIC TREATMENT of Avulsed tooth(pathway of the pulp

Extraoral Time Less Than 60 Minutes :

Closed Apex Endodontic treatment is initiated at 7 to 10 days. In cases in which endodontic treatment is delayed or signs of resorption are present, “long-term” calcium hydroxide treatment is given before root canal filling˙. No chance exists for revascularization of these teeth, and endodontic treatment should be initiated at the second visit at 7 to 10 days. If therapy is initiated at this optimum time, the pulp should be necrotic without infection or at most only minimal infection. Therefore endodontic therapy with an effective inter appointment antibacterial agent over a relatively short period (7 to 10 days) is sufficient to ensure effective disinfection of the canal. If the dentist is confident of complete patient cooperation, long-term therapy with calcium hydroxide remains an excellent treatment method. The advantage of calcium hydroxide use is that it allows the dentist to have a temporary obturating material in place until an intact periodontal ligament space is confirmed. Long-term calcium hydroxide treatment should always be used when the injury occurred more than 2 weeks before initiation of the endodontic treatment or if radiographic evidence of resorption is present. The calcium hydroxide is changed every 3 months within a range of 6 to 24 months. Calcium hydroxide is an effective antibacterial agent and favorably influences the local environment at the resorption site, theoretically promoting healing. It also changes the environment in the dentin to a more alkaline pH, which may slow the action of the resorptive cells and promote hard-tissue formation. However, the changing of the calcium hydroxide should be kept to a minimum (not more than every 3 months) because it has a necrotizing effect on the cells attempting to repopulate the damaged root surface.

Open Apex For the open-apex tooth, endodontic treatment should be avoided and signs of revascularization should be checked. At the first sign of an infected pulp, the apexification procedure is begun. Teeth with open apices have the potential to revascularize and continue root development, and initial treatment is directed toward reestablishment of the blood supply .The initiation of endodontic treatment is avoided if at all possible unless definite signs of , such as periradicular inflammation, are present. The diagnosis of pulp vitality is extremely challenging in these cases. After trauma, diagnosis of a necrotic pulp is particularly desirable because infection in these teeth is potentially more harmful as a result of cemental damage accompanying the traumatic injury. External inflammatory root resorption can be extremely rapid in these young teeth because the tubules are wide and allow the irritants to move freely to the external surface of the root. Avulsed tooth should be soaked in doxycycline then Minocycline powder placed on the root surface before replantation. Patients are seen every 3 to 4 weeks for sensitivity testing. Recent reports indicate that thermal tests with carbon dioxide snow (–78°C) or difluordichlormethane (–50°C) placed at the incisal edge or pulp horn are the best methods of sensitivity testing, particularly in young permanent teeth.One of these two tests must be included in the sensitivity testing of these traumatized teeth. Recent reports confirm the superiority of the laser Doppler flowmeter in the diagnosis of revascularization of traumatized immature teeth.Radiographic (apical breakdown or signs of lateral root resorption) and clinical (pain to percussion and palpation) signs of pathosis are carefully assessed. At the first sign of pathosis, endodontic treatment should be initiated, and after disinfection of the root canal space, an apexification procedure should be carried out.

Extraoral Time More Than 60 Minutes

Closed Apex Treatment for closed-apex teeth with an extraoral time of more than 60 minutes is the same as with less than 60-minute dry time. Closed-apex teeth are treated endodontically in the same way as those teeth that had an extraoral time of less than 60 minutes.

Open Apex (If Replanted) If endodontic treatment was not performed out of the mouth, the apexification procedure is initiated. In open-apex, replanted teeth the chance of revascularization is extremely poor.Therefore no attempt is made to revitalize these teeth. An apexification procedure is initiated at the second visit if root canal treatment was not performed at the emergency visit. If endodontics was performed at the emergency visit, the second visit is a recall visit to assess initial healing only.

Crack toth syndrome: A common finding with a fractured cusp or cracked tooth is the frequent presence of pain upon release of biting pressure.

Test Cavity The test cavity method for assessing pulp vitality is very seldom used today. This method is used only when all other test methods are deemed impossible or the results of the other tests are inconclusive. An example of a situation where this method might be used is when the tooth suspected of having pulpal disease has a full coverage crown. If no sound tooth structure is available to use a bridging technique with the electric pulp tester and cold test results are inconclusive, a small class I cavity preparation is made through the occlusal surface of the crown. The patient is not anesthetized while this procedure is performed, and the patient is asked to respond if any painful sensation is felt during the drilling procedure. If the patient feels pain once the bur contacts sound dentin, the procedure is terminated and the class I cavity preparation is restored. This sensation signifies only that there is some viable nerve tissue remaining in the pulp, not that the pulp is totally healthy. If the patient fails to feel any sensation when the bur reaches the dentin, it is a good indication that the pulp is necrotic and root canal therapy is indicated. Treacher Collins syndrome genetic disorder downward slanting eyes, micrognathia, conductive hearing loss, underdeveloped zygoma, drooping part of the lateral lower eyelids, and malformed or absent ears. Turner tooth----Infection and trauma –Hand-Schuller-Christian triad nDiabetes insipidus nExophthalmos nBone lesions (Langerhans dis) Oral signs of hand-schuler-christ. = bad breath, sore mouth, loose teeth lesion are sharply punched out radiolucency and teeth appear as FLOATING IN AIR Amelobelastoma---- Hony comb-soap bubble Paget's: Billatral maxilla------Cotton wool Cherubism: billatral, mutilucular Gorlin syn or Basal Nevus Cell Syndrome: Bifid rib, OKC, BCC Gardner Syn: multiple facial , , , GI polyps à potential for colon carcinoma Erythema Multiform: young men, viral or drug, sudden onset, vermilion, intraoral not on gingiva, (target)bull eye on hands and feet Steven’s Johnson = severe form of Erythema Multiforme (affects eyes, mouth, and genitalia) PDL widening in Hyper paratyriod osteosarcoma and scleroderma(Trismus, widened PDL spaces, mask like face, Excess type I & III collagen) Eagle Syn: Stylohyoid ligament calcification Zoster = shingles = unilateral Multiple neuromas on lips, tongue or palate may indicate that pt has MEN III adrenal pheochromocitoma Tumors of Endocrine glands à esp. Medullary carcinoma of thyroid (can cause death) Actinic chelitis---> SCC Oral hairy & Mono, Burkit lymphomaà both caused by EBV Kaposi sarcoma by herpes 8 Garre's (prolifrative periostitis) and Ewing sarcoma are both onion skin Myxoma and ameloblastoma are soap bubble pattern Desquamative gingivitis includes pemphigoid, pemphigus and erosive lichen planus To change from long scale intensity (low contrast) to short scale intensity (high contrast) but maintain image density, the operator should decrease kVp and increase mAs –Auriculotemporal syndrome (Frey syndrome) Often after parotid gland surgery Periapical Cementoosseous Dysplasia OR periapicalcemental dysplasia OR periapical cementoma: This is a relatively common phenomenon that occurs at the apex of vital teeth. A biopsy is unnecessary because the condition is usually diagnostic by clinical and radiographic features. Women, especially black women, are affected.appears in middle age (around 40 years) . The mandible, especially the anterior periapical region, is far more commonly affected than other areas. More often, the apices of two or more teeth are affected.No Tx. REGEZI : also known as true cementoma, is a rare benign neoplasm of cementoblast origin. It occurs predominantly in the second and third decades of life, typically before 25 years of age. There is no gender predilection. It is more often seen in the mandible than in the maxilla and more often in posterior than in anterior regions. It is intimately associated with the root of a tooth, and the tooth remains vital. Cementoblastoma may cause cortical expansion and, occasionally, low-grade intermittent pain. Radiographically, this neoplasm is an opaque lesion that replaces the root of the tooth . It is usually surrounded by a radiolucent ring representing the periodontal ligament space and the advancing front of the tumor. TX: Because of the intimate association of this neoplasm with the tooth root, it cannot be removed without sacrificing the tooth. Bone relief is typically required to remove this well-circumscribed mass. Recurrence is not seen.REGEZI Glossodynia: tender or painful tongue Glossopyrosis: burning sensation of tongue Epstein-Barr virus It is known to cause infectious mononucleosis, is implicated in the causation of Burkitt's lymphoma and Nasopharyngeal carcinoma and Warthin's Tumor : Swelling in neck , more diffused Infectious Mononucleosis : Swelling in neck ,more rounded and localized Osteosarcoma : Radiographic - sunburst appearance Fibrous displysia: Ground Glass Appearance Candidiasis Pt has both burning sensation of tongue and altered taste sensation. Sjogren’s syndrome does NOT have burning sensation of tongue but has altered taste sensation. Pt complains of difficulty wearing the denture and shows cervical caries. Tx of sicca(dry mouth, eye) in Sjogren syn is pilocarpine Multiple myeloma: Bence jones proteinuria, punched out lucencies Most common salivary gland tumor = pleomorphic adenoma. M 8-bit digital image would have 256 shades of gray Most common salivary gland benign major or minor : Pleomorphic adenoma Most common malignant minor : Most common malignant major : Recurrance: OKC, Ranula Symptomatic Irreversible Pulpitis Teeth that are characterized as having symptomatic irreversible pulpitis exhibit intermittent or spontaneous pain, whereby rapid exposure to dramatic temperature changes (especially to cold stimuli) will elicit heightened and prolonged episodes of pain even after the source of the pain is removed. The pain may be sharp or dull, localized or referred. Typically there are minimal changes in the radiographic appearance of the periradicular bone. With advanced irreversible pulpitis a thickening of the periodontal ligament may be evident, and there may be some suggestion of pulpal irritation by virtue of extensive canal calcification. Deep restorations, caries, pulp exposure, or any other direct or indirect insult to the pulp, recently or historically, may be present and may be seen radiographically or clinically or be suggested from a complete dental history. Typically, when a symptomatic irreversible pulpitis remains untreated, the tooth will eventually succumb to necrosis. Percussion and palpation may or may not positive. Cold alleviates pain in severe cases Asymptomatic Irreversible Pulpitis Occasionally, deep caries will not produce any symptoms, even though clinically or radiographically the caries may be well into the pulp. Left untreated, the tooth may become symptomatic or even necrotic. In cases of asymptomatic irreversible pulpitis, endodontic treatment should be performed as soon as possible so that this conversion does not take place and cause the patient distress. Internal resorption and pulp polyp are examples. Necrosis This condition is subsequent to symptomatic or asymptomatic irreversible pulpitis. Under complete necrosis and before any pathosis extends into the periodontium, the tooth is typically asymptomatic. It will not respond to electric pulp tests or to cold stimulation. However, if heat is applied for too long, the tooth may respond, possibly relating to remnants of pulpal fluid or gases expanding and extending into the periapical region.Pulpal necrosis may be partial or complete and it may not involve all of the canals in a multirooted tooth. For this reason, the tooth may present with confusing symptoms, whereby pulp testing over one root may give no response and pulp testing over another root may give a vital response, and the tooth may exhibit symptoms of an irreversible pulpitis. After the pulp becomes necrotic, bacterial growth can be sustained within the canal. When this infection (or the bacterial toxins from this infection) extends into the periodontal ligament space, the tooth may become symptomatic to percussion or exhibit spontaneous pain. Radiographic changes may occur, ranging from a thickening of the periodontal ligament space to the appearance of a periapical radiolucent lesion. The tooth may become very hypersensitive to heat, even to the warmth of the oral cavity, and is often relieved by applications of cold. As previously discussed, this may be very helpful in attempting to localize a necrotic tooth when the pain is referred or nonlocalized. Periapical Disease Periradicular Periodontitis A tooth with acute periradicular periodontitis will have a very painful response to biting pressure or percussion. This tooth may or may not respond to pulp vitality tests, and the radiograph or image of this tooth will generally exhibit a widened periodontal ligament space but no periradicularradiolucency. A tooth with chronic periradicular periodontitis generally presents with no clinical symptoms. This tooth does not respond to pulp vitality tests, and the radiograph or image will exhibit a periradicular radiolucency, usually around the apical third of the root. This tooth is generally not sensitive to biting pressure but can “feel different” to the patient upon percussion. Periradicular Abscess

A tooth with an acute periradicular abscess will be very painful to biting pressure, percussion, and palpation. This tooth will not respond to any pulp vitality tests and will exhibit varying degrees of mobility, and the radiograph or image can exhibit anything from a widened periodontal ligament space to a periradicular radiolucency. Swelling will be present in the mucobuccal fold and facial tissues adjacent to the tooth. The patient will frequently be febrile, and the cervical and submandibular lymph nodes will be tender to palpation. A tooth with a chronic periradicular abscess (suppurative periradicular periodontitis) will not generally present with clinical symptoms. This tooth will not respond to pulp vitality tests and the radiograph or image will exhibit a periradicular radiolucency. The tooth is generally not sensitive to biting pressure but can “feel different” to the patient upon percussion. This entity is distinguished from chronic periradicular periodontitis because it will exhibit intermittent drainage through an associated sinus tract.

___Anti-psychotic drugs___ 1.phenothiazines -chlorpromazine(can cause Tardive dyskinesia) 2butyrophenones - haloperidol tx of schizophrenia/tourette syndrome 3.thioxanthenes 4.Diverse heterocyclic 1. lesion that occurs from tooth flexure? abfraction (yes) 2. what liquid is found in glass ionomer? polyacrylic acid (yes)

Glass ionomer cements are mixed powder-liquid component systems. The powder is a fluoro alumino-silicate glass that reacts with a liquid which is polyacrylic acid to form a cement of glass particles surrounded by a matrix of fluoride elements.

3. infection from premolars most like to drain into? sublingual area (mand. Molarsàsubmand area)

4. the drug enforcement agency is concerned with what? potential for abuse

5. where are you most likely to perforate on a maxillary central incisor? Facial

Note: The lingual wall of mandibular teeth is most easily perforated when preparing an access opening due to the lingual inclination of these teeth.

6. implant placement is placed where? lmm apical to adjacent cej 7. what do you use to SC/RP implants? plastic instruments 8. how far do implants need to be spaced apart one another? 3 mm apart and 1mm from crown of adjacent teeth see page 320 mosby

9. dentist restored tooth and accidentally left caries ... what happens to caries? caries is arrested

10. where are the primate spaces? max= between lateral & canine & mand= between canine & 1st molar

11. osteoporosis is associated with which of the following diseases? Hyperparathyroidism 12. do not give which medication to lactating female? codiene (yes) FDA pregnancy category C. This medication may be harmful to an unborn baby, and could cause breathing problems or addiction/withdrawal symptoms in a newborn.

13. all are advantages of using nickel titanium endo files over regular steel files except? a. flexibility (yes) b. bending memory (yes) c. direction of the flutes (no)?

The significant advantage of a nickel-titanium file is its unique ability to negotiate curvatures during continuous rotation without undergoing the permanent deformation or failure that stainless steel files might incur. A significant risk during NiTi rotary instrumentation is instrument separation. When the instruments are stressed over time, the crystalline structure can change or deform making the files weaker and more prone to reaching their elastic limit.

1. you separate an endo file 3mm from the apex and obturate above it... which case will show the best prognosis? a. vital pulp w/ no periapical lesion(yes) b. vital pulp wI periapical lesion c. necrotic pulp wI no periapical lesion d. necrotic pulp wI periapical lesion

2. two things that account for a successful posterior composite restoration? a. type of resin and size of tooth b. size of tooth and type of prep??? c. type of resin and type of prep (ash thinks: b/c type of resin re: occ wear)

3. what is the initial step when you have an acute perio abscess? a. incision and drainage (yes) b. anti-biotic therapy c. antibiotics then incision

4. how do you repair a porcelain veneer with composite? a. microetch, etch and silane b. sandblast, etch and pumice c. pumice, silane, etch

The enamel surface should be cleaned with pumice and water. • While protecting adjacent teeth with matrix strips, the enamel is acid-etched. The etched surface is washed and dried and a layer of unfilled bond resin is applied and thinned with oil-free air.

5. least effective sedative? a. diphenhydramine (benedryl) b. nitrous oxide (ash could be wrong, but doubted)

Nitrous oxide is a weak anesthetic and is used with other agents, such as thiopental, to produce surgical anesthesia.

Diphenhydramine (Benadryl) is representative of the sedating-type antihistamines, a class of drugs that causes significant xerostomia.

6. which is the best systemic anti-fungal? a. ketoconazole (yes) b. nystatin c. miconazole d. clotrimazole e. fluconazole

7. conical shaped caries w/ broad base with apex towards pulp is commonly seen in? a. root caries (same as smooth caries) b. smooth caries (look like v w/ apex towards pulp) (mosby’s 32) c. pit/fissure caries (inverted ‘v’ with apex towards occlusal) 8. best clinical determinant of root caries a. sensitivity to cold b. sensitivity to sweets c. soft spot on tooth (Ash thinks) (according to mosby’s pg 37: root surface caries appears early as a well defined discolored area adj to the gingival margins, near cej) The most commonly used clinical signs to describe root caries utilized visual (color, contour, surface cavitation) and tactile (surface texture) specifications (Banting, 1993). There are no reported clinical symptoms of root caries although pain may be present in advanced lesions.

Clinical Signs of Root Caries Clinical diagnosis is the process of recognizing diseases by their characteristic signs and symptoms. It is an imperfect process because there is considerable variation both in the signs and symptoms of disease in individual subjects and in the interpretation of those signs and symptoms by clinicians. Nevertheless, clinical observations are extensively relied upon for diagnosis in the absence of more definitive methods

Note: Most current research suggests that the microbial etiology of root caries is very similar to coronal Caries. in the past it was thought that Actinomyces species (viscosus and naeslundli) were most commonly associated with root surface caries.

9. 40 ylo pt w/ all 32 teeth. No cavities. Has stain & catch in pit of molar. what do you do? a. watch and observe (yes) b. sealant c. composite

10. 12 ylo kid w/ carious lesion on tooth #9 that shows pulp exposure wI open apex... what do you do? a. rct b. apexiflcation (yes) 1-How enough space provided for eruption of permanent anterior mandible? a.primate space b.lee way space c.mesial shift d.anterior maxillary teeth tilt labialy 2-Dentist charge fro crown $500.insurance only covers $400.Dentist waves copayment ($100) but still let insurance he charges $500 for crown. what this action called? a.Down codding b. Over billing c.Price fixing d.Unbundling 3- how many canal 2nd max primary molar has? a.1 b.2 c.3 d.4 Max first molar 4 canals Mand first molar 3 canals (25% 4 canals) Mand second molar 3 canal (25% 4 Canals) 4-Early childhood caries in? a. black b. Hispanic c.white Hispanics have the highest rate of ECC in both developed and developing countries with an average prevalence of 13%-29%, second only to Native Americans (Huntington, et al., 2002). 5-Pt with cafe au lai and some problem with iris(lisch nodules)...does not mention about GI polyps a.neurofibromatosis b.peutz jegher c.jaffe syndrome neurofibromatosis characteristics: lisch nodules: traslucent brown-pigmented spot of iris axillary freckling(crowe's sign).enlargement of fungiform papilla. enlargement of mandibular foramen 6-Dentist reviewed of his existing 1000 Pt chats .he noticed among these 1000,last year 200 had periodontitis but in a year after 300 have periodontitis, how much is the incidence? a.0.3 b.0.1 c. 0.2 7-what kind of study is above? a. descriptive b. analytical c. experimental 8-differential diagnosis of periodontal abscess and periradiculal abscess? a.percussion b.vitality test c.palpation 9-implant success rate after 10 yrs a. 85% is 5 years b.95% c 80% 10.what percentage of US population have access to fluoridate water? a.45% b.65% (42 states) c.85% 11-Pt states you are the 8th dentist that he met recently,he did not like none of them and like you very much and will bring all his family for check up.which disease Pt suffers from? a.narcism b.paranoid--no trust c.schizoid What is most odontogenic cyst that could end up to amelioblastoma? 1-Dentigerous 2-Residual A primordial cyst is a devleopmental odontogenic cyst. It is found in an area where a tooth should have formed but is missing. Primordial cysts most commonly arise in the area ofmandibular third molars. Under microscopes, the cyst looks like an . M….fin for heroin addiction ------????withowing syndrome-pt may get addict to this drug kid 5 years with systemic ds came to control plaque- sodium fluoride or chlohexidine Schizoid personality disorder (SPD) is a personality disorder characterized by a lack of interest in social relationships, a tendency towards a solitary lifestyle, secretiveness, and emotional coldness Paranoid personality disorder is a psychiatric diagnosis characterized by paranoia and a pervasive, long-standing suspiciousness and generalized mistrust of others. Bipolar disorder or manic–depressive disorder (also referred to as bipolar affective disorder or manic depression) is a psychiatric diagnosis that describes a category of mood disorders defined by the presence of one or more episodes of abnormally elevated mood. --- Aggressise brushing ---- http://www.cda-adc.ca/jcda/vol-69/issue-1/20.pdf 47C (1-5 Min) you can expect necrosis Kelly's combination(5 symptoms) : supra erupting lower anterior teeth, premolar area bone resorption , Maxillary tuberosity supra eruption , mandibular bone resorption , papillary hyperplasia Eye----Only alpha1 Lung----Only B2 Heart---B2, B1 Blood vessle------Alpha1, 2, B2 No parasym in Vessele (Alpha----contriction,,,,,,Beta-----Dilation) Papillion-Lefevre: Autosomal recessive,impaired T and B lymphocyte,reduced killing of bacterial and fungal infection,advanced periodontitis in primary and permanent dentition related to AA,hemoraghic gingivitis-teeth float in the soft tissue, excessive bone loss, spontaneously tooth exfoliation, palmar-plan-tar keratosis, keratosis of elbow and knee, may be confused with psoriasis, ectopic calcification of falx cerebri. Tx: rigorous OH, chloxexidine rinse and periodic antibiotic therapy Mucopolysaccharidosis (autosomal recessive) such as Huler and Hunter syndrome: , anterior gingival hyperplasia due to mouth breathing. Thin enamel with pointed cusp in posterior teeth. in some cases multiple impacted teeth in one large follicle forming a rosette-like pattern radiographically(in Hunter syndrome).Coronary heart disease(Huler) 1-A practitioner has been using bisect technique for taking xray for his patients. His distance was 8 inches using that system now if he wants to take x-ray utilizing parallel technique form a distance of 16 inches the time of exposure would be: ( double,half….) Same 2 -Asthma exhalation wheezing sound!! correct Exhale on both Asthma and COPD Asthma, emphysema and chronic bronchitis usually cause wheezing on the exhale. A wheeze or noise upon inhaling is usually caused by an obstruction in the upper part of the respiratory tract. 3- Best caries preventive measure for retained root under Compete Denture ( Fluoride therapy, cast copping) 4 -fear and anxiety (4-5 ? one of the item was fear is painful and anxiety is a disease) 6) rectal bleeding, granolumatous gingivitis and recurrent aphtous ulcer! What is the disease?\Crohn disease 7) linear in the casting ring! Its purpose? compensate for casting material expantion 8) Most common cell in necrotic pulp? PMN cells 9) Daily fat need/use for body 30% 10)all causes gingival hypertrophy except (dilantin, niohidipine, dixosin, cephalox) and among ca channel blokers Nifedipine and Diltiazem 11)pic of pamphigous, denture 12) how does “neuropraxia” affects the neurons and axons( the answer :both of them are normal It was asking if they loose their epithelium I guess!!!) 13) yrs old male who has had a history of trauma comes to ur office. The pulp is necrotic. On the xray it shows that the apex is open and it has a wide cancal! The tX would be: ( apexiiffication , routine RCT, Epico surgery) 14) Medicare is a federal thing that provide health care for elderly . It does not cover dental ( answer: both statements are true) 15) Which pulp horn in primary teeth is most susceptible for exposure during operative procedure - MB of first molar - MB of 2nd molar - DB of first molar - DB of 2nd molar 16) Most common cause of arch space lost in primary dentition ( decay( not sure though) / ectopic eruption 1st molar) 17) Cancer of the other part of the body metastases most commonly to ? lip FOM tongue Gingiva Bone:Molar region of mandible Soft tissue: attached gingiva then tongue.Breast cancer is the most metastatic to the oral cavity appearing in the mandible and then lung cancer is the second appearing in the soft tissue. 18) Which has the most thermal coefficient ( enamel, composite, amalgam, gold) Linear thermal coefficient is most for tooth- gold- amalgam- composite 19) Picture of a narrow Lucent canal next to upper lateral (nutrient cancal ( I guess the answr), incisive never canal) 20) A question pointing to the Ear lobe on the Pano 21) How develops( answer : fructan has adhesive property then it sticks to the tooth surface) 22) Perio disease is most common in black male 23) After trying the anteriori teeth in the mouth for complete denture patient. In order to determine condylar inclination we make a record of patient’s anterior guidance. What we have to take into consideration in the lab: ( Raise the pin on the articulator while setting the condylar inclination) 24) A tooth is not responsive to cold, not percussion and pulpation tender ( - necrotic pulp and chronic apical periodontitis – irreversible pulpitis and normal apex) there was not an item saying necrotic pulp and normal apex) 25) A child is allergic to ampicillin, what is the regimen for prophylaxis? ( know the dosage for cephalexin , azithromicin) 28) open technique impression for implant ( to transfer the exact angle of implant to the lab) http://idasmiles.com/files/ImpressionTechnique.pdf 29)what is the least important factor when evaluating for implant: ( concavity of mandible, bone density , distance to mandibalr cancal, bone width) ------25. In office bleaching changes the shade through all except a. dehydration b. etching tooth c. oxidation of colorant d. surface demineralization ----

31. After implant placement, an edentulous patient should a. avoid wearing anything for 2 weeks b. immediately have healing abutments placed over the implants c. should wear an immediate denture to protect the implant site All implant supported overdenture: Minimum 4 implants in Mand, 6 implants Maxilla Implant and tissue supported overdenture: For mandibular denture ,2 implants in Mand symphysis btw mental foramina, may be joined by a bar A minimum of 3 months healing time is usually recommended following fixture placement (nonimmediate loading) to allow for osseointegration of the implant fixture 17.what is the most definite way to distinguish amelloblastoma from OKC a.smear cytology b.reactive light microscopy c.reflective microscopy

16. Which of the following describes the character of dentinal tubules at the pulpal end when compared to those at the enamel end? a. More per unit surface area and more wider in diameter. b. Less per unit surface area but much wider in diameter. c. More per unit surface area and smaller in diameter. d. Less per unit surface area and smaller in diameter

Table 12-1 -- Mean Number and Diameter per Square Millimeter of Dentinal Tubules at Various Distances from the Pulp in Human Teeth DISTANCE FROM PULP (mm) Number of Tubules (1000/mm2) Number of Tubules (1000/mm2) Tubule Diameter (μm) Tubule Diameter (μm) DISTANCE FROM PULP (mm) MEAN RANGE MEAN RANGE Pulpal wall 45 30–52 2.5 2.0–3.2 0.1–0.5 43 22–59 1.9 1.0–2.3 0.6–1.0 38 16–47 1.6 1.0–1.6 1.1–1.5 35 21–47 1.2 0.9–1.5 1.6–2.0 30 12–47 1.1 0.8–1.6 2.1–2.5 23 11–36 0.9 0.6–1.3 2.6–3.0 20 7–40 0.8 0.5–1.4 3.1–3.5 19 10–25 0.8 0.5–1.2 histologically loss of retepegs is seen in 1)phemhigus 2)pemphigoid 3)lichen planus(sawtooth-shaped rete pegs of epithelium)

1) For internal bleaching: use sodium perborate & a) 10% hydrogen peroxide b) 35% hydrogen peroxide c) 10% carbamide peroxide**** 35% carbamide peroxide

1) Have lot of incisal overlap, what do you want to change to maintain balanced occlusion a) ↑ condylar inclination b) ↓ condylar inclination****

During the child's first visit, the dentist requested that the parents wait in the reception room. The child cried moderately, but tearfully, throughout the dental examination and prophylaxis. The dentist "gave her permission" to cry while he/she worked and then took no notice of her crying. Her crying diminished in intensity over time and then stopped. With respect ONLY to the crying behavior, the dentist has A. used positive reinforcement. B. used negative reinforcement. C. extinguished the behavior. D. ignored the problem.

50) You have HIV+ pt you can do all of the followin except a) treath with metronidozole b) free gingival graft c)prophylaxix to treat candidiasis 56) You give the nitroglycerin to the pt with angina and heart rate goes up what's the reason ? natural reflex to the decrease in blood pressure 77) Which durg will be used to treat an overdose of methotraxate       leucovorine

8)Which fluoride causes the most staining? Which one of these has the least sedative effect? I have no idea Diphenylhydramine Chlorpheniramine- Tripelennamine

New FMX Radio- pregnancy or 6 months ago had Taken one Which kid of caries has been reduced over time? Occlusal, root, proximal Max ridge in denture pt is ………….. over time? Narrow, wide , small , large 1. St johns - used for? · Short term memory loss · depression------not with benz and HIV medication 2. Diabetic undergoing surgery? o Take Clear fluids with same insulin intake o Take Clear fluids with ½ amount insulin? 3. Implants · Do not probe, disrupt attachment · Probe to check bone level (maybe) 4. different symptoms between hyperthyroid shock and hypothyroid shock? · Cold 5. fear vs anxiety? · Fear is local, anxiety is generalized? 6. implant stable? · 1 unit · 2 unit 7. pic of dorsum of tongue with 1 side totally red with white nasty patches and painful? · Phemigus · 8. what sound altered with altered vertical dimension? · S sound** · V · P 9. symptoms if too much codeine? · Insomnia (I think) Cold and claming skin · Irritable 10.Diabetes leads to · Blindness · Ataxia 11.Know where are primate spaces · Betw laterals and canines in max, betw canines and 1st molar in mand 12.Hep A is transmitted by? · Blood · Aerosols · Food 13.Daily recommended calorie from fat? 10% 20% 30% (I think) 14.Max depth of toothbrush and floss going subgingival? · Toothbrush 0mm, floss 2-3 mm · Toothbrush 2-3 mm, floss 0mm · Tooth brush 1mm, floss 2-3 mm (I have no clue, maybe this?) 15.Pilocarpine used for? · Increase in salivation 16.Stucture most likely damaged with gingival graft? · Greater palatine nerve (I think) · Less palatine nerve · Some other arteries 17.Know the level CD4 and neutrophils of HIV patients 18.If there’s space, what is most likely prevent the tooth distal from it to move into space? · Occluding to opp tooth? · Dense bone? · Non interference contact 19.Most associated with developing stage of gingivitis? · Macrophages 20.Best flap surgery for gingival recession? · Double papilla flap · Lateral pedicle 21.Know that PDL thickens as you get older 22.Mouth breathers leads to? · Short face · Open bite 23.Primary tooth most like to have high chance of pulp exposure? · Mand first molar 24.Want to extract all posterior teeth, what is the order? · 1st,2nd,3rd b/c easier access? · 1st,3rd,2nd b/c do easier tooth first? · 3,2,1st b/c …. 25.which technique to view midfacial? · Waters projection 26.Who has highest autoimmune diseases? · Adult male · Adult female · Teenage male · Teenage female 27.Which diseases are assoc with positive nikolsky sign? · Pemphigus · Fibrous dysplasia 28.If do class 2 on patient with high caries? · Resin bond composite · Resin GI at gingival and composite the rest 29.If have large caries near pulp, what do you do? · Restore with composite · Remove caries, place appropriate temprorary and follow up · Placed GI liner, restore with composite 30.Know apexification 31.What is function of reciprocal clasp? · Retention · Oppose force on buccal clasp or stability 32.Know all signs of hyperocclusion · Recession · Pain when biting down hard · Root resorption · Alteration of lamina dura · Alteration of perio space 33.Know antibiotic prophylaxis 34.patient with past heart attack, high bp, high chlosterol, asa type? · Type 3 35.asthma what kind of sound? · Inspiration wheezing · Expiratory wheezing · Inspiratory 36.Bennett shift, what is the path of movement · Mesially · Laterally · Inferiorly · Superiorly 37.Distobuccal flange of denture is determined by · Masseter 38.When do the max centals calcify? · At birth · 2-3 months · 6 years etc 39.when do plaque accumulate after eating? · 24 hours · 36 hours · 1 hour 40.Know perio diagnosis, what is considered hopeless? · Mobility of tooth · Furcation involvement? 41.How do you make crown narrower? · Move line angles more facially and deepen the embrasures · Move line angles proximally and deepen embrasures · Move line angles facially, widen embrasures · Move line angles proximally, deepen embrasures ------1. our office is fee schedule and pt needs new crown but pt used up all of her allowance (or something like that), what do you do? Charge her a higher fee Can’t not treat her Negotiate a lower fee Charge same fee 2. Want to compare 2 groups of people, male and female for something, what test do you look at? Multiple regression Chi square test T-test 3. What least likely treatment for class 3 furcation? Endo and root amputation Guided tissue SRP Extraction 4. What is gives a higher failure rate after perio treatment? Smoking Diabetic patient 5. What is first thing when you re-eval patient after perio treatment? OHI Plaque index Review medical history (I think) 5'-reason of using plaque disclosing agent? -motivate Pt to clean suspected area -observe degree of disease 6. What is most likely to cause gingival recession? Tooth brushing (I think) SRP Bruxism 7. Which drug is least sedating? Hydroxyzine Claritin- non sedating antihistamin 8. Is tetracycline good for prophylaxis for children? No 9. Know wrought wire clasp (pg 330 mosby) 10.Know all properties of porcelain 11.Pic of all max anterior lingual totally eroded but incisal edges are fine Pt sucking on lemon Attrition Bruxism Occlusal trauma 12-Inform consent is for which ethic principle? -Veracity -autonomy -beneficence -justice 13-Anesthesia done into the vein results in? -increased BP -increased HR -pain in the periphery 14-Pt with hemoglobin A1C of 12%,Pt just visited the MD,what kind of TX we can do? -FM ext. -Sc/Rp -Consult with MD prior tx In most labs, the normal range is 4-5.9 %. In poorly controlled diabetes, its 8.0% or above, and in well controlled patients it's less than 7.0%. 15-Buccal cheek of 60 yrs man,not wipe-able? -leukoplakia( more on floor 50%, tounge25%) -candida -white spongy nevous bilatral- auto dominat- place ANUG debriment+peroxide ------in HIV use antimicrobial--- no attachment loss Most common place oral cavity cancer------Tounge 1. Hunter – Hurler syndrome 2. Temp at necrosis happens during implant placement is 47 for 1-5 min 3. What do u use to dripp for implant space – hight torque slow speed drill, I had option of alkaline phosphate irrigation but it's salin 4. Specificity 5. Pic of bells palsy 6. Pic of odontoma♠ 7. Pic of periapical cementeal osseous dysplasia 8. Ameloblastoma 9. Risk factors for oral cancer 10. Green stain in porcelain – due to cu 11. Methotrexate drug interaction beta-lactamase 12. Study thoroughly about irreversible pulpitis, reversible, apical periodontitis, acute apical abscess, periodontal abscess 13. Vitality test 14. Erosion 15. Attrition 16. GIC- root caries 17. What do you do when you see a decalcification spot? Fluoride and leave 18. Community water fluoride level – 1ppm (1 mg per one liter) 19. Kid is 15 yrs old and living in a community with 0.75ppm of fluoride, what do u do? – no supplementation 20. TFO on implant,all of the following except. – gingivitis and periodontitis plaque disctuctive periodontitis 21. T- test 22. Free gingival graft receives it’s eptithelium from – adjacent tissue (blood supply from CT) 23. Where is apical positioned graft contraindicated – maxillary palatal region 24. In latest periodontal longitudinal studies, the research says – apically position flap is best, keratinized tissue is necessary for attached gingival and some other answers 25. Most common reason for cardiac arrest in children – respiratory depression, CHD, atrial septal defects, congenital heart disease ----- 26. Common between chronic periodontitis and gen – teeth, predisposing factors, local factors 27. Pt with café au lait spots, axillary freckling, lish nodules of iris – Neurofibromatosis 28. Dentist said, pt doesn’t have to pay his copay. He didn’t inform third party payer – what is he doing? = downcoding, unbundling, price fixing, over billing 29. Most common impacted tooth – Max canine, max lateral, mand premolar or max premolar(if wisdom was chioce that one) 30. What do u do in a composite restn. – make a prep depth of 2mm, just prep carious pits and fissures, involove O surface with M and D lesion 31. Lot of questions of pulp testing 32. Radiograph showing radiolucency in furcation area in a primary second mand molar – reason is periodontal, pulp necrosis, trauma 35. If you have to extract 1st, 2ed and 3rd molar, in which order do u extract? 1, 2, 3 to preserve tuberosity; 3, 2,1 for better visibility 36. In which direction do you luxate distoangular max third molar – distal palatal, distobuccal ,palatal, mesial 37. What is the diff between LED curing ling and conventional – choices were too long (it is an except question)

 Rapid Cure - light instantaneously on with no warm up required

 No heating of substrates - cold curing close up to cure head

 One peak wavelength of output power (multiple peaks possible as required by process)

 No fuming of cured material normally caused by excessive radiant power from conventional lamps

 Safe operation - no UVB, no ozone, no bulb break, low voltage, no mercury

 Predictable radiant output power with limited drop off with life

 Variable power - ability to create a cure profile with time e.g. reduced stress curing

 Even intensity of radiant power over curing area

 Long life time> 10,000 hrs of LED on time (LEDs only need to be on when curing)

 Minimal drop off in power over life

 Small size and light weight of cure heads

 Ability to create three dimensional flood areas for curing complex shape 38. What is compomer Compomers” are recently introduced products marketed as a new class of dental materials. These materials are said to provide the combined benefits of composites (the “comp” in their name) and glass ionomers (“omer”). 39. Why don’t you use GI resin cement in cementation of all ceramic restoration - its expansion could cause cracking of porclain 40. Lichen planus 41. Epidermolysis bullosa Epidermolysis bullosa (EB) is a rare genetic disorder caused by a mutation in the keratin gene. The disorder is characterized by the presence of extremely fragile skin and recurrent blister formation, resulting from minor mechanical friction or trauma. 42. Black woman with radioopacity in lower anterior – periapical osseous cemental dysplasia 43. PAN to identify – pterygomaxillary fissure 44. Stafne radiographic identification 45. Which cyst doesnot appear in radiograph – Nasolabial 46. Pear shaped Radiolucency on # 9 – incisive foramen 47. Which is the best technique to view maxillary sinus – options were both MRI and Water’s view 48. Best radiograph to view maxillay sinus disease – depends on the type of question being asked. if they are asking about the anatomic form then CT scan is best. If you are intending to find soft tissue pathology then MRI. Best radiograph: CT>water> Best technqie to maxillary sinus: CT>Water> Best for diagnosis for soft tissue: MRI both MRI and CT were there http://www.ghorayeb.com/ImagingMaxillarySinusitis.html ( comparative view of all three) Computer Tomography. Computed tomography (CT) scanning is the best method for viewing the paranasal sinuses. There is little relationship, however, between symptoms in most patients and findings of abnormalities on a CT scan. CT scans are recommended for acute sinusitis only if there is a severe infection, complications, or a high risk for complications. CT scans are useful for diagnosing chronic or recurrent acute sinusitis and for surgeons as a guide during surgery. They show inflammation and swelling and the extent of the infection, including in deeply hidden air chambers missed by x-rays and nasal endoscopy. Often, they can detect the presence of fungal infections. X-Rays. Until the availability of endoscopy and CT scans, x-rays were commonly used. They are not as accurate, however, in identifying abnormalities in the sinuses. For example, more than one x-ray is needed for diagnosing frontal and sphenoid sinusitis. X-rays do not detect ethmoid sinusitis at all. This area can be the primary site of an infection that has spread to the maxillary or frontal sinuses.asa Magnetic Resonance Imaging. Magnetic resonance imaging (MRI) is not as effective as CT in defining the paranasal anatomy and therefore is not typically used to image the sinuses for suspected sinusitis. MRI is also more expensive than CT. However, it can help rule out fungal sinusitis and may help differentiate between inflammatory disease, malignant tumors, and complications within the skull. It may also be useful for showing soft tissue involvement 49. Faint radiolucency with radioopaque border in max sinus – Mucocoele 50. U shaped radio-opacity/radioopaque in max sinus – zygomatic process

51. Pt has a chronic periapical abscess with sinus tract – tx is RCT, what is that tx for sinus tract : antibiotics, curettage, cautery, no tx 52. Apicoectomy 53. Which is the best test – EPT or thermal -cold is best 54. Benign lesion with Cauliflower appearance – papilloma 55. Perineural invasion is seen in – adenoid cystic carcinoma, Pleomorphic adenoma or low grade muco epidermoid carcinoma Another very important factor with adenoid cystic carcinoma is the tendency for perineural invasion. This tumor has a marked tendency to invade nerves. Perineural invasion is seen in about 80% of all specimens. 56. Post herpetic neuralgia cause by varicela zoster virus- shigles-extreme unilateral pain 57. Lichen planus pic 58. Desensitization - In psychology, desensitization (also called inurement) is a process for mitigating the harmful effects of phobias or other disorders. For medical purposes, desensitization is a method to reduce or eliminate an organism's negative reaction to a substance or stimulus. 59. Why do you give methadone to pt who is trying to stop heroine? – to decrease withdrawl symptoms 60. Patient tried using nicotine patches, gum but still couldn’t stop what do you do? Buproprion, or behavior counseling 61. When RPD is rocking on the fulcrum line, whats the problem?No option about the base problem with denture base problem with indirect retainer 62. Nitrous oxide safe mechanism level – 70% 63. In which condition Nitrous is contraindicated – Asthma, respiratory infections contraindication: contagious disease, emphysema, bronchitis, autistics Pt. 1st trimester of Pregnancy. 64. OKC 65. Periostat – doxycycline 20mg placed in periodontal pocket – what does it do? I think it inhibits collagenase 66. What is allograft? same species http://en.wikipedia.org/wiki/Bone_grafting 67. Something about freeze dried graft proteins, what is going to happens?how does it generate the bone? 68. Couple of questions about bisphosphanate http://www.ada.org/prof/resources/topics/osteonecrosis.asp http://www.ada.org/prof/resources/topics/ topics_osteonecrosis_bisphosphonate_report.pdf http://www.ada.org/prof/resources/topics/ topics_osteonecrosis_bisphosphonate_report.pdf Avoid invasive dental procedures while receiving bisphosphonate treatment. For patients who develop while on bisphosphonate therapy, dental surgery may exacerbate the condition. Clinical judgment by the treating physician should guide the management plan of each patient based on an individual benefit/risk assessment.

 Dental infections should be managed aggressively and nonsurgically (when possible).

 Endodontic therapy is preferable to extractions; and, when necessary, coronal amputation with root canal therapy on retained roots to avoid the need for extraction. 69. Dentist tells a patient that mecury is toxic and she should replace her amalgams with composite – which principle of ethics dentist does not follow? beneficence justice veracity 70. What do you prescribe for pts with ANUG? tetracycline,Debridement and mouthwash Hydrogen peroxide 71.Pt with manic depression (bipolar) disorder not willing to get treated for that is now getting dental treatment from you. What do you see in this patient a) mood swings b)depression c)excitement 72. One of your patient is having a dental problem and treating that is not under your capability and you are reffering that patient to a specialist, this type of behaviour comes under which of the following codes of ethics a)Autonomy b)Justice c)Beneficience d)Veracity e)nonmaleficience http://www.ada.org/prof/prac/law/code/principles_01.asp 73- Which of the following causes Cavernous sinus thrombosis A)Subcutaneous Abscess of upper lip b)Subcutaneous abscess of lower anterior region

Infections in upper front teeth are within the area of the face known as the "dangerous triangle". The dangerous triangle is visualized by imagining a triangle with the top point about at the bridge of the nose and the two lower points on either corner of the mouth. If your mother ever told you not to pick a pimple on your face or else you would get a brain infection, this is what it would look like if it actually happened 74- Pt brushes twice a day n also floss but stil;l he has calculus what do u suggest A) written instructions B) video C) demonstration of brushing techniques 75. First upper molar is supra erupted and the lower opposite molar is missing which of the following u will not do? A) up righting the molar B) rotation of molar C) Intrusion 76.In Implant Preparation which of the following can be used A)Hydroxyapatitie Irrigant b) High Speed Hand Piece c) Low torque Drill d)Air Coolant. 77-Cast restoration procedure which impression is least used ? Additional, polyether, Irreversable hydrocolloid(alginate), agar 78-Pt has hepatic dysfunction which pain medication can prescribe? a-Oxycodone b-naproxen c-acetaminophen http://www.hepatitis-central.com/mt/archives/2007/05/is_there_pain_r.html 79-Highest cleft palate incidence black Caucasians Asian 80-which of the following dental treatment can be done with pts taking bisphosponate A)Alveoloplasty B) endo C) Extraction D) Scaling 81- Confounders known n unknown ? case control, cohort, cross sectional http://www.experiment-resources.com/confounding-variables.html Confounding In Epidemiology a confounder is: not part of the real association between exposure and disease predicts disease unequally distributed between exposure groups

A researcher can only control a study or analysis for confounders that are: known measurable

Example: Grey hair predicts heart disease if it is put into a multiple regression model because it is unequally distributed between people who do have heart disease (the elderly) and those who don't (the young). Grey hair confounds thinking about heart disease because it is not a cause of heart disease.

Strategies to reduce confounding are: randomization (aim is random distribution of confounders between study groups) restriction (restrict entry to study of individuals with confounding factors - risks bias in itself) matching (of individuals or groups, aim for equal distribution of confounders) stratification (confounders are distributed evenly within each stratum) adjustment (usually distorted by choice of standard) multivariate analysis (only works if you can identify and measure the confounders) 82-Primary stress bearing area in mandible a-Retomolar pad b-alveolar ridge c-buccal shelf and incase the residual ridge is in good shape it also contributes to primary support. http://www.docstoc.com/docs/20113319/antomical-landmark 83-In which of the following muscles you can place the mandibular denture a-lateral pterygoid b-middle pterygoid c- buccinator (Buccal) d-maseter (Distobuccal) 84-picture of diffused pigmentation in anterior max and mad incisors in attached gingiva? Melanoma Racial pigmentation 85-5 yrs with systemic disease what MW used for plaque control a-NAF b- Chlorhexidine 86-which of the following is conditioned stimulus a) dentist b)dental chair 87)which of the following are necessary for a test to be accurate a)specificity b)reliability c)validity Note: Most common mid facial fracture is Zygomatic complex fracture (Peterson) A few thoughts on the conventional perioperative management of diabetics Hypoglycaemia is much more dangerous in a patient rendered unconscious than hyperglycaemia Therefore it is safer to err on the side of hyperglycaemia in patients undergoing surgery. Nevertheless ketoacidosis must be avoided. IDDM require insulin constantly or they will become ketoacidotic. Depending on the metabolic upset associated with surgery (i.e. stress response), the patient will require either a intravenous infusion or iv pump with sliding scale. The "Alberti" solution of combining insulin and glucose in a single bag of fluid has the advantage of safety (you can't get too much of either). The local modification of this regime is: 1/3 of total morning dose (or 40% of total daily dose) of insulin as actrapid in 1L Solution 18 + 10 mmol KCl over 8 hours; an additional subcutaneous sliding scale of actrapid can be added for tighter perioperative control. Intermediate duration-depot insulin (insulatard) should be avoided during the perioperative period. For most well controlled Type II diabetics, hypoglycaemia caused by OHAs and insulin pose a higher risk in the perioperative period than hyperglycaemia. "No glucose no insulin" BMJ 1996; 76; 198-202 OHAs should be withheld on the morning of surgery and on the evening before. It is the sulphonyureas that cause hypoglycaemia. Take special precaution with chlorpropamide (diabenase) and glibenclamide (daonil) as these have a very long duration of action, and should be stopped a couple of days in advance. Take home message: be paranoid of hypoglycaemia in patients on OHAs and remember that blood sugar levels that constitute euglycaemia for non diabetics will cause symptoms of hypoglycaemia in diabetic patients. ======1. Your office is fee schedule and pt needs new crown but pt used up all of her allowance (or something like that), what do you do? Charge her a higher fee Can’t not treat her Negotiate a lower fee Charge same fee 2. Want to compare 2 groups of people, male and female for something, what test do you look at? Multiple regression Chi square test T-test 3. What least likely treatment for class 3 furcation? Endo and root amputation Guided tissue SRP Extraction 4. What is gives a higher failure rate after perio treatment? Smoking Diabetic patient 5. What is first thing when you re-eval patient after perio treatment? OHI Plaque index Review medical history (I think) 6. What is most likely to cause gingival recession? Tooth brushing (I think) SRP Bruxism 7. Which drug is least sedating? Hydroxyzine Claritin 8. Is tetracycline good for prophylaxis for children? 9. Know wrought wire clasp (pg 330 mosby) 10.Know all properties of porcelain 11.Pic of all max anterior lingual totally eroded but incisal edges are fine Pt sucking on lemon Attrition Bruxism Occlusal trauma ======NBDE Part 2 Day 2 8 yrs old kid with autism Ectodermal dysplasia Asthma Hx of Heat Attack Sjogerns syndrome Pt with no med Hx but has attrition and lichen planus Day 1

 Penumbra – how to prevent this in x-rays o Should be produced from a point source to blurring of the edges of the image o Strong beam to penetrate o Xray should be parallel

 Beaten metal skull – crouzon’s

 NUG – tons of questions

 Impending doom – panic attack

 Asthma – why use corticosteroids – decrease inflamation

 Beta blocker mechanism of action

 Alpha -1 = vasoconstriction

 Dowel core = vertical stop

 Extruded gutta percha – observe

 Purpose of placing a post after RCT = retain core

 X-rays/pictures o Dentinogenesis imperfecta o Coronoid process o Ameloblastoma o AOT? o ? . Saw an xray of a primary tooth with a stainless steel crown and an erupting tooth underneath; giant RL in the furcation of the primary tooth.. o Odontoma

 Glucocorticoids C/I in diabetes

 C/I for implants = adolescent

 Extracted teeth in a preggo – give Tylenol for pain

 Ordinal and nominal - temperature, heart rate (??)

 Kernicterus = excess bilirubin in blood

 Borderline personality – patient went to several dentists and tells you she loves you and will refer you all her friends/family

 Voice control – know why you use it

 SLOB rule: palatal root appeared mesial to the MB root of a max first molar – where was the x-ray beam directed?

 Modified Widman flap – full thickness; how far from MGJ – coronal, esthetics

 Which flap CANT you do if there is little attached gingival o Partial thickness flap

 Diazepam (3Q)

 Status epilepticus (3Q) – diazepam (lido toxicity)

 Grand mal/digoxin OD – dilantin/phenytoin

 Petit mal - ethosuximide

 Tobacco - #1 risk factor for oral cancer

 Most likely site for SCC? I put Ventrolateral tongue (other choices were weird…palate (least)…)

 Case control study = odds ratio

 A 5 year old kid with 0.75ppm in water = no fluoride supplementation

 Fluoride in water depends on temperature of the air

 If you take a single therapeutic dose of aspirin, how long will it prolong bleeding time? (4 hours, 10 hours, 1 day, 1 week…)??

 Cant mix aspirin with garlic or ginsing

 Root amputation of MB root – cut at furcation and smooth for patient to keep clean

 Irreversible pulpitis (3Q)

 EPT does NOT indicate health of the pulp

 Tooth with closed apex gets intruded, what is most likely to occur? Necrosis (avulsion, resporption)

 Treatment of choice for primary tooth with caries causing pain for 24 hours…tooth is restorable…pulpectomy

 CaOH is NOT for direct pulp capping in primary teeth

 What daily oral rinse would you give to a medically compromised child for plaque control? (choices were CHX, Listerine, Nystatin, stannous fluoride)

 What population has the worst survival rate for SCC? (whites, blacks, native Americans…)

 Best way to see a horizontal root fracture? Multiple x-rays from vertical or horizontal…?

 Ectodermal dysplasia and cleidocranial dysplasia – which one has ano/oligodontia? o Ectoerdermal

 Cerebral palsy – patient will have spastic during treatment

 TONS of questions of primary herpetic gingivostomatitis

 Patient presents with a 1.5cm ulcer on the buccal mucosa…there are scars from previous ulcers just like this…dx = Major Aphthous

 Shows a picture of what looks like a giant SCC – incisional biopsy

 + Nikolsky (pemphigus was not a choice) – erythema multiforme

 Immunofluorescence = pemphigus

 If you blow air and it lifts the tissue and leaves a bloody area (something about evaluating the patient further…)  Candida – white and scrapes off

 Picture of a child with primary herp on the lips

 Culture sensitivity assay (2Q) – I think its for recurrent infections

 Why don’t we use broad spectrum antibiotics? Produce resistant bugs

 Why bevel for a gold onlay? Resistance; percent elongation for burnishing and remove unsupported enamel

 How to remove a gold inlay? Section isthmus and remove in 2 pieces

 Know about hemiseptum (perio) 1 wall defect 2=crater, 3=infrabony, 4=moat, circumfrencial

 Easiest defect to fill in? (choices: Hemiseptum, 3 wall intrabony defect, narrow shallow defect)

 How to replace large chunks of mandible? Freeze dried bone; autogenous

 What is the most osteogenic? (Choices: alloplast, autograft, etc)

 Cognitive restructuring

 There were like 4-5 Q on osteoradionecrosis

 Most common tooth for vertical root fracture? Mandbular molars mandibular premolar?

 Patient has flat posterior occlusal surfaces and excessive wear on maxillary anteriors – erosion  attrition

 Know the definition of attrition

 Biologic width = 2mm

 INR of ___ allows you to do extractions? 2.5

 PCD – 3Q

 How to tell if something is a perio lesion or endo lesion? Vitality tests

 After RCT and placement of cast post and core and PFM crown a patient presents with pain on biting 3 months later…why? Vertical root fracture

 Recently placed MOD amalgam; pain on biting = hyperocclusion

 Most common topical antimicrobial? CHX

 Focal radiopacity – no history of inflammation or anything else – Dx = idiopathic osteosclersosis o Condensing osteitis was a choice but I think that has to do with inflammation

 Most common sequela of dry socket? Pain

 Trephonation – cut into bone to relieve pressure at the apex of a tooth

 Excess vertical angulation of x-ray = longer roots?

 4Q on where you find localized aggressive periodontitis – incisors and first molars

 AA = Localized aggressive perio

 Most likely place in an x-ray to see nutrient canals? Mandibular anteriors  3Q on ASA classification

 A patient wears a nitroglycerine patch and takes nitroglycerine tabs 2-3 times/day o ASA classification? III o Stable or unstable angina? unstable

 3Q on angina

 Know signs of hypoglycemia

 Know trendelenburg position

 How to be sure youre far enough away from the IAN when placing an implant? Measure it on pano

 Lesion on gingival – if you press, it blanches and it bleeds easily – dx = pyogenic granuloma?

 Patient has a retruded tongue. What complication do you expect when he tries to wear a denture? (choices: speech problems, problems with the lower denture, problems with food bolus)

 When you lose teeth, your face sinks in. What happens when you lose alveolar bone? (choices: prognathia, increased muscle tone, decreased nasolabial fold)

 Child lost both his primary mandibular canines prematurely – why? Lack of arch space

 How to do measure the projected arch space for permanent teeth? Canine to canine

 Value is the most important in color

 Cannot decrease value ? by adding gray

 Most technique sensitive part of placing veneers? Preparation, color match, impressing..?

 What prevents lingual displacement of a ¾ crown? Lingual margin

 You see a RO on the mesial-occlusal portion of a maxillary first molar – wanted to know what it was? Choices: cusp of carabelli, sealant….(weird question)

 Which one is NOT RO? (choice: osteopetrosis – marble bone, extremely rare; osteoporosis, pagets – cotton wool)

 Bilateral swelling of parotid NOT caused by: (choices: anorexia, HIV, ..)

 Critical pH = 5.5

 What is that FIRST thing that happens when caries develops? Fermentation or drop in pH

 Interproximal caries – 2 triangles…know where the point of the triangles point

 “Mouthguard bleaching” = 10% carbamide peroxide

 Most common reason for mand fractures? Sports injury (other choices were domestic violence, workplace mishaps, self injury)

 If you get a blow to one side of the mandible, which condyle gets fractured .. or something?

 Something about nonverbal vs verbal communication – nonverbal is not as reliable  MSDS sheets – chemical information

 Leathery, dark brown = what kind of caries – anaerobic root caries; sclerotic dentin

 Know how caries indicator works??!??!! (specific mechanism) binds to inorganic stuff

 Components of GI CEMENT – alumina silicate and polycarboxylate cement w/fluoride

 Gingival index = nominal

 Most recurrent cyst – OKC

 Picture of epilus fissuratum

 ANB = maxilla-mand relationship

 Monteleukast (singulair) mechanism – inhibit leukotrienes

 Cheek biting – not enough horizontal overlap

 Mercury – most often ingested by inhalation

 Morphine and enkephalins – weird question

 Most reliable measure of HIV progression? CD4 count (viral load was also a choice…)

 How can you get a class III? Maxillary deficiency

 Osseous surgery for alveolar bone proper = ostectomy

 When would you give systemic tetracyclines AND do perio surgery?

 Implant hex is for anti-rotation

 Girl with caries into the pulp on tooth #3 – radiograph shows alternating RL/path at inferior border of mandible (a.k.a onion skin, bacterial)Garre’s Osteomyelitis

 # of vertical planes in the face? 5

 Know how beta blockers work for angina

 Patient with dentures – when he speaks the back teeth click – what should you do?

 Loss of both mandibular first primary molars – lingual holding arch (with permanent incisors and 1st molar)

 Loss of mandibular 2nd primary molar = distal show

 What is the LAST thing you do to correct cleft lip/palate?

 “ugly duckling” stage – maxillary centrals

 Le Fort II fracture – what nerve is most likely injured? infraorbital

 Heroin addict. Why shouldn’t you give Nubain? o mixed agonist-antagonists analgesics

 Inflammatory Resorption: o Necrotic pulp

 Convulsions - hypokalemia

 Replacement resorption: o Ankylosis

 Avulsed tooth splint time? o 7-10 days (mosbys) o Don’t remember answer choices

 Closed fixation splint time? o 6 weeks? o 9 weeks? o 12 weeks?

 Involved in caries progression but not an initiator? o Lactobacillus

 Consequence of overtriturating? o Stays soft a long time? No consequence

 Cleft lip/palate most commonly occurs in which type of occlusion o Class 3?

 Percentage of USA with Community water fluoridation o 40%? o 60%? o 80%? o As of 2008, 72.4%

 What does Gingival Index measure? o Disease severity? nominal

 Cleft lip & palate 1 in 700

 Neurofibromatosis o Café-au-lait o Axillary freckling (crowe’s sign) o Iris freckling (lisch spots)

 Mucus retention cyst = sialolith

 Submandibular gland = sialolith

 Adenoid cystic carcinoma o Spreads thru peri neural spaces

 OKC  recur

 Compound odontoma (Picture) – mini teeth

 Medically compromised child. What should you give for daily plaque control mouth rinse? o Nystatin? o Sodium fluoride? o Stannous Fluoride? o CHX?

 Epidermolysis bullosa  blisters, scarring and hypoplastic teeth

 Nutrient canals (picture)– mandibular anterior  Ugly duckling central  max centrals

 Cleidocranial dyplasia o Lots of Qs o Delayed tooth eruption and supernumary teeth

 Ectodermal dysplasia o Partial or complete anodontia

 Dentin dyplasia (radiograph)

 Perio vs Endo Lesion  VITALITY

 Traumatized tooth. Why is EPT going to give u a false reading? o Damage to blood supply? Damage to nerve supply?

 Cracked tooth – lower 1st molar

 Veneer o 0.3 gingival o 0.5 facial

 Cleaning and shaping  tapers adequate to prepare for obturation

 NaOCl o Dissolves organic o Does NOT remove smear layer

 EDTA o Chelator o Removes smear layer

 Indications for apicoectomy

 Hemisection  Class 3 or 4 furcation

 Avulsion success rate – depends on extra-alveolar dry time

 Why do we use fiber over cast post? o Modulus of elasticity same as dentin?

 Mandibular molars refer pain to ears

 Collimation o To decrease size of xray beam and volume of irradiated tissue

 Muscle is radioresistant

 Mandible more prone to osteonecrosis

 Increased horizontal angulation = overlap

 Waters view – maxillary sinus

 Maxillary sinus disease? CT, MRI, Pano?? I think MRI

 Arachidonic acid makes prostaglandins

 Young pt and old pt have chronic periodontitis – who has better prognosis? Older pt

 Gingivectomy – bleeding points outline the incision  Allograft

 Implant best bone – anterior mandible

 Subantimicrobial dose doxycycline – inhibits collagenase

 Most common impacted teeth o Man 3rd, max 3rd, max canine

 Healthy implant o Less than 0.1mm of bone loss per year

 Post herpetic neuralgia  consequence of herpes zoster

 Don’t give antibiotics in dry socket

 Articaine o Hydroxylation in liver? o Conjugation in liver? o Plasma esterase?

 Hypokalemia = seizures

 Amitriptyline – increased sensitivity to epi

 Nitrous has emergency stop so you don’t give more than how much nitrous? o 70%? 90%?

 Leeway space

 Fluoride chart – 2 Qs

 Primary teeth begin to calcify 4 months in utero

 Trauma to max incisors – more likely with class 2 div 1

 Treat crossbite immediately

 Most common missing teeth o Mand 2nd premolar o Max lat

 BSSO o Paresthesia – side effect

 Problem at morphodifferentiation = size and shape abnormalities

 Best way to know when tooth will erupt: o Root formation? . 2/3 formation – erupt thru bone . ¾ root form – erupt thru gingival

 Failure to thrive – first 6 months = enamel hypoplasia

 Diastema – wait till canines erupt before doing anything

 Distance between implants – 3mm

 When should you check mobility of implant? \ o At impression appt?  Increased VDO = Decreased freeway space

 F and V sounds – position of anterior teeth

 Indirect retainers – assist direct retainer to prevent displacement of denture base in occlusal direction

 Base metals – long span bridge

 FPD fracture – bad design

 Increased water: powder in gypsum o Increased setting time o Decreased setting expansion

 Incidence o Number of new cases/total # of people

 Longitudinal – incidence

 Statistical significance o To reject the null hypothesis

 Sterilization vs disinfection

 Know case study (ratio) vs. cohort. (relative risk) Etc

 PPO versus individual practice association

 Telling a patient to switch from amalgam to composite because of amalgam toxicity? o Veracity?

 Knowing when to refer a patient? o beneficence

 Classical conditioning, desensitization, operant conditioning, etc

 Agonist – have intrinsic activity; antagonists don’t

 Efficacy – level of binding to receptor

 Alpha 1 – vasoconstrictor

 Phenothiazines (anti-psychotics) – block dopamine, tardive dysk

 Fluoxetine (SSRI) – inhibits reuptake of serotonin

 Grand mal seizure tx – phenytoin/dilantin

 Acetominophen – liver toxicity

 H2 receptor blockers – decrease gastric acids

 Anti-arrythmia – increase refractory period of cell

 Nitrites/nitrates – dilate mostly veins

 Saw palmetto C/I with Coumadin

 Gingko C/I with Aspirin

 Herpes – valacyclovir

 Impending doom – panic attack

 Pregnant women – syncope o Inferior vena cava

 2 year old uncooperative patient o Have parent hold them down? o Have assistant hold them down? o Papoose them?

 Which is more common social disorder? o Depression or anxiety?

 Can pregnant patients take BZD? no

 Methotrexate o Folic acid o Turns urine red

 Functional cusp bevel? o Protect the pulp? o Prevent overreducing? o Structural compatibility?

 PFM FPD multiple failures o Improper metal framework design

 ANB: maxillomandibular relations

 What does not resorb? o Rugae? o Horizontal palate?

 How far should you place ceph from the kid…..5 ft, 6 ft, 60cm, 15cm

 Hyperkeratosis of palms and feet with in kids? o Papillion-lefevre

 Patient comes in with nitroglycerin patch and is taking nitroglycerin patches. Which ASA classification? III

 Lipid solubility

 Diffusion

 Class III caused by maxillary insufficiency

 Which do you refer immediately to a physician temporal arteritis

 Composition of GI cement

 Why do you add cross linking to monomers? o Increased strength?

 Hypothyrodism in kids? o Give vitamin D

 Least likely to cause oral cancer o HIV  If you have an adverse drug reaction who do you report it to? o FDA?

 People on cortical steroids… o Adrenal insufficiency

 Corticosteriods work o By decreasing inflammation in the airways

 Acute asthma attack…. o Albuterol? o Amiphyilline o Theophyilline – severe asthma o Phenylepherine?

 Immunofluorescence? o Pemphigus

 IV bisphosphonates for 3 years o RCT

 Where do you do SRP? o JE to….

 Lower denture support? o Buccal shelf

 St. John’s Wart: what’s it used for? depression

 Which of the following potentiates anticoagulation? – St. John’s Wart (other options: licorice, camomile)

 Which opoid is contraindicated for nursing mothers? – (all 4 options were opiods) Demerol, propoxyphine, pentozine, codeine (oxycodone)

 Hypoglycemia, what’s a risk factor? – hypopituitary (other options: hypotension, hypophoso)

 Leukotrienes (potentiate inflammation and cause attacks)– asthma

 What does/could uprighting a molar cause? Excessive open bite

 Lots of questions on what the studies are

 False neg/false positive

 Impending dome, what is it? panic attack

 Fear vs Anxiety – anxiety for situation, fear is emotional for general?

 Lots of questions

 Necrotic pulp – strict anaerobes

 Toothbrushing -1mm; Flossing - 2-3mm

 Generalized gingivitis – 30%

 Hopeless prognosis – class II mobility

 Multiple Myolma – punched out lesions, plasma cells  Neuromas – “men”

 Neurofibromatosis – café au lait, axial freckling

 Steven – Johnson syndrome (erythema multiforme) what areas does it affect: eyes, genitals, mouth

 Frey’s Syndrome – sweat when you get hungry around oricular temporal

 Know t-cell count/CD4 for AIDS pt – bad = 200-500 HIV <200 AIDS

 Know INR – 2.5

 What percent of daily intake is fat? - options: 10, 20, 30, 40, 50%

 40 yr old with pit and fissure – fluoride, no tx, prr, monitor

 pt allergic to codeine, what do you give her for pain? Demerol (not vicodin - hydrocodone, percocet - oxycodone, Tylenol - codeine)

 Ortho – histology of tooth and movement (pdl, bone, pulp, resportion)

 Enamel hypoplasia to affect max central – 3-6 mo

 Asthma – how to control? Options: Nitrous, voice control, general anesth

 Case: Autism – how to control? Options: iv sedation, nitrous, voice control, bdz

 Case: Autism – what is not contributing to his fear? Age, medical condition, past dental history, parental fear

 Case: ectodermal dysplasia (congenitally missing teeth)– what is not appropriate/ meaningful question to ask the patient: can you pull your shoulders together o Ortho question – how do you proceed with closing the gap only between the centrals – ortho immediately then frenectomy

 Recurrent caries on a class II – proximal box

 Dowel core – vertical stop

 Why not ept on tooth after trauma? – options: neural damage or blood

 Case: space maintenance; band and loop

 Fixed- repeated questions

 What do you want porcelain opposing – porcelain or composite

 Precision attachment RPD?

 Carious exposure, when is vital tooth therapy indicated: immature tooth, low bleeding, low caries

 Case: sensitive to cold, sensitive to sweets, caries – what do you do after you remove caries: crown, endo and crown, temp, MOD; history of caries – endo and crown; not a ton of restorative – crown depending on tooth structure left; not a lot of structure- endo, post, crown

 Remove Tori: max (y incision) mand (retract and grind); Denuded area of bone: let it heal, send to oral surgeon, graft, rinse with h2o2 and eugenol

 Nitrous oxide, use too much, what do you show symptoms of: multiple sclerosis

 Odontoma radiograph  ANB -6=class III

 What would Opiod overdose cause? CONSTIPATION, resp depression, euphoria, sedation, dysphoria, analgesia, urinary retention, increase intracranial pressure

 Emphysema, can you give an Opiod? No

 Pt with difficulty breathing (asthma, inhaler) can’t: lido, epi, more than 2+appts/week, lay flat

 What xray for midface fracture – water’s view

 Sinusitis - CT

 Why triangular axis for max central? Straight line access, shape of tooth, etc remove pulp horns

 Tooth avulsed, replanted within an hr? splint, wait, rct or splint, rct

 What’s the most common? subluxation, crown fracture, or root fracture

 Know the order of repair of cleft lip/palate o Which do u do last? Repair cleft? Repair lip? Repair nasal deformity? Remove teeth?

 Trephination definition

 Herpangina from coxsakie virus

 ANB measures max /man plane

 Prego o U can give Tylenol if extraction

 Know ordinal, ratio, and interval things o Ex: bp and pulse measured in ratio o Temp measured by what? Not sure

 When do u do a sensitivity test? o I put when recurrence of infection after intial AB tx failed?

 Tons of questions on primary, acute herpes gingivostomatitis

 ANUG o Tons of questions  13 year old with 1-3 mm pockets and bleeds a lot o Chronic gingivitis

 7 year old with swollen , fever, malaise, easy bruising? o Acute leukemia, chronic leukemia? Peridontits? o I put acute leukemia

 candida

 Crouzon- beaten metal skull

 Gardners- supernumerary teeth, inte polyps, epidermoid cysts, etcc.

 Which disorder doesn’t have premature exfoliation of primary teeth? o I put cliedocranial dysplsia

 Few questions on PA cemental dysplasia

 Lots of endovitality, pulp dx tests etcc…..

 Lots on pt compliance, and pedo (Read mosby’s pt amnagment section)

 Studies o Case controlodds ratio o Cohortrelative risk o Spec, sens o Etc…

 Where is def on dep and indep variable? o I put methods

 Which LA makes GABA o Cocaine?

 Crown: rt ration acceptable= 1:1

 Pt has no post teeth in max arch and u need to make a CR record, what do u do o Make a record base and occlusal rims

 Good samaritain law

 Impending doom-panic attak

 Ugly duck stage o Diastema in max CI and lat I

 Veneers, which stage is most technique sensitive? Finishing, prep, impression, cementation, shade selection

 Veneers on max anteriors and PM’s, which tooth do u make slightly darker for nat look? o Canines?

 Value, chrom, hue o Value= whitness/blackness? o Value is most imp

 Pic of odontoma

 Pic of mand tori

 Pic of zygomatic arch

 Pic of ameloblastoma

 Pic of ranula or hemangioma? o They said 3 cm sac in ant floor of mouth filled with blue fluid, I said henamgioma bec of blue fluid?

 3 year old with spont pain, what tx? Pulp cap, RCT? Pulpotomy?

 4 year old loses both primary 1 molars, what space maintenance? None needed so young?

 BZD needed in pt with liver ds, which can u administer? o Diazepam? Oxazepam? Don’t rem other choices, look up  Asthma o Steroids to reduce inflammation of airway

 Metaleukast rx mechanism

 Sulfonyly urea drug mech

 Macroglossia and Megaloblastic anemia- vit b def

 Koplicks spots in rubella (no oral lesions in mumps and rubeola)

 Easy perio stuff

 Implants o Don’t want to damage IAN, what o u do? Look on pan, measure bone on PA with grid? o No metal (SS) used to clean o Disadvantage of cement retained? Not retrievable

 Initiator in PVS? o Lead dioxide? Sulfur?

 Synersis and imbition occur in irreversible hydrocolloids o Pour immediately

 Luxation o Splint for 10 days

 Intruded tooth =necoris

 Lots of old questions

 Hypoglycemia vs hyperglycemialots of questions o Type 1 diabetic- most likely to have hyperglycemia

 Ginsing CI aspirin

 Diabetes CI steroids

 Bacteriostatic drugs stop pr synthesis

 Pen CI tetracylcline

 B blockers- decrease force of contraction

 ASA classification, 2 questions

 9 year old with infection of perm 1M with lamination of RO and RL lesion around bone? o Proliferative periostitis

 Turners tooth o From infection

 Tooth with most caries?

 Space analysis from perm canine-canine

 Best px of oral cancer is seen in which population? Blacks, whites, Hispanic, Indians?

 Lesion that is pedunculated? Papilloma?

 Cleft palate assoc with what in preg? Def in folic acid, or alcohol?

 Penumbra is decreased by what? o I put dec object-target distance, not sure

 Something about ph and pka

 Ciritcal ph= 5.5

 Lots on loc aggressive PD o Bacteria= AA o Location= perm 1M and incisors o Not from plaque o Can be from neutrophil problem o Give systemic antibiotic to kill PD pathogens all over mouth not just in localized area

 Acute PA pain tx? Remove pulp

 Dx test used to? Reproduce pain or eliminate pain

 Apexification/genesis

 Immunoflourescence= pemphigus test

 Pemphigoid= positive nikolsky sign

 Ectodermal dysplacia-oligoontia

 Neurobiromatosis o Lisch nodules, café au lait, axillary freckles

 Grand mal seizures-phenytoin

 Zantac- H1 recpetor for peptic ulcers

 Perm max central is linual to normal position, why? Prolonged retention or early exfoliation of primary tooth?

 Cracked tooth- mand molars

 MB root fracture of max 1 M, root amputation needed. Cut off root at chamber and smooth for hygiene

 Dowel-vertical stop

 Palatal grooves of which teeth associated with PDD? o Not sure, max molars, max I’s, etc?

 2 M has min keratinized tissue and u need to do PD sx, what can u not do? o Distal wedge? Apically positioned flap (I put this), etc..  Free ginig graft gets blood supply from donor or recipient site? CT or epithelium? o I put recipient CT

 Full thickness flap= periosteum+ CT+ epithelium

 Compontent of GI

 Components of resin o Bisgma, silane coupler and initiator

 Resin has therm coeff of expansion that’s the highest

 Erosion, abfraction, abrasion

 On xrays root caries on proximal look like cervical burnout

 Non-malefience o CE courses and referral

 Change dates for ins companies= fraud

 Hesitant pt, can u help them fill out forms?

 Liquid sterilization o I put gluteraldehyde, also had alcohol, etc..

 Non working interfenrences occur where?

 Protrusion interfences occur where?

 Pt moves jaw to left which muscle is resp? o Left or right Lat pterygoid? I put left

 ORN mainly in mand

 Iv bisph for over 3 years RCT of root tips  Implant Can be placed in INR=2? Not above?

 Least caries seen in mand ant teeth

 Highest recurrence OKC

 RCt done on max PM and has MO amalgam and years later has RL lesion? Re=tx conventionally

 How do u know how to adapt a perio instrument? o Terminal shank? Tip of instrument? Handle?

 Specificity – identifies which individuals are free of a disease Which disorder doesn’t have premature exfoliation of primary teeth - cliedocranial dysplsia Endo Diagnosis – know this section very well Cohort study – relative risk Minimum acceptable crown:root ratio = 1:1 Picture of an odontoma Panoramic x-ray – pterygomaxillary fissure Panoramic x-ray – hyoid bone x-ray – periapical cemental dysplasia which area has the highest incidence of implant success – anterior mandible Picture of a lesion in the anterior mandible – differential diagnosis: adematoid odontogenic tumor, central giant cell granuloma, nasopalatine duct cyts When mixing alginate, how do you slow the setting process – use cold water When mixing gypsum how do you decrease setting expansion – use more water in the mix Which material cannot be used for metal castings? – irreversible hydrocolloids A lot of questions on hypoglycemia, hyperglycemia Penicillin – cannot be combined with tetracycline Ginseng – cannot be combined with aspirin Poorest prognosis for oral cancer – Blacks What happens when you increase voltage – shorter wavelengths, higher energy waves Best method for diagnosing pulpal pathology – thermal tests (pretty sure this is the answer) Ectodermal dysplasia – oligontia External bleaching completed for 2 weeks – placement of veneers following a 2 week period Grand mal seizures – phenytoin Least likely tooth to be congenitally missing – I put canine If patient is taking bisphosphanates (case study) – should complete RCT If anterior incisor erupts lingual to normal – due to prolonged retention of primary teeth Free gingival grafts – blood supply comes from donor CT Know psychology well – a lot of questions on conditioning, de-sensitizing, etc Know ethical principles well – autonomy, beneficence, non-maleficience Jaw moves forward – I put that both condyles are translating Highest incidence of vertical fracture – mandibular molars Nonworking interferences – maxillary lingual cusps, mandibular buccal cusps (confirm this in the prosth section) ANUG – don’t use antibiotics if there are no systemic effects Picture of lichen planus First action when a file separates on the lingual surface of a premolar – answers were weird, best answer was to offer the patient referral to an endodontist Primary teeth – large pulp horns Kernicterus – excessive bilirubin in blood A lot of fluoride questions Vertical BW x-ray (in a case study) – more alveolar bone is exposed Lesion on buccal mucosa, no scarring – minor apthous ulcers Angular – treat with topical antifungals (antifungal infection) Functional cusp bevel – increases resistance Patient with a PFM crown that is overcontoured (case study) – not enough reduction of the preparation, too much metal, too much porcelain Easiest defect to fill – 3 walled defect Best prognosis for guided tissue regeneration – class II furcation Endo-perio lesions – a lot of questions (can be separated via vitality tests) Chronic periodontitis – scaling and root planning but no antibiotics (most effective for juvenile periodontitis) X-ray with nutrient canals A lot of questions of in-office emergencies – syncope, allergic reaction, etc Most common respiratory difficulty in an office – asthma Side effect of Ritalin – appetite suppression (case study question) Non-verbal communication – not as reliable as verbal communication Remineralized enamel – I put that it is more resistant to decay (not sure if this was the right answer) Picture of a sinus tract (case study) – tooth was previous traumatized Prostaglandins – stimulates prostaglandin synthesis Implant hex – used for anti-rotation Heroin addict, should not give morphine – mixed agonist-antagonist reaction Convulsions – caused by hypokalemia Implant success rate after 10 years – 95% Percentage of Americans with fluoridated water – most correct answer was 65% (I would look this up because the answers were weird) Closed fixation due to fractured jaw – 6 weeks Where is a sialolith most likely – submandibular gland Incidence of cleft lip/palate – 1/700 OKC – most recurrent Radiograph of – dentinal dysplasia Traumatized – either damage to the nerve supply or damage to blood supply (I wasn’t sure which was correct) NaOCl vs. EDTA Fiber post – similar modulus of elasticity to dentin Maxillary sinus – best viewed using Water’s view

 There was another question that didn’t include Water’s view as an option – I put MRI Best way to view TMJ – I put MRI Graft from one individual to another (not genetically identical) – allograft Healthy implant – less than 0.1 mm of vertical bone loss/year, no mobility Gingevectomy – bleeding points help outline the incision Muscle – most radioresistant cell Young person with perio disease vs. older person – older person has a better prognosis Diastema – wait until canines erupt Best way to predict when tooth will erupt – look at root formation (roots typically 2/3 formed when teeth erupt) Crossbite – should be treated immediately

 One question gave options about treating a crossbite and asked what not to do – I put observation as an answer Articaine – metabolized by plasma esterase Fluoxetine – serotonin reuptake inhibitor Least likely to cause oral cancer? – hairy leukoplakia, dentigerous cyst, HIV Adverse drug reaction – reported to the FDA Best esthetic result (case study) – porcelain crowns Know angles classification, ANB Radiograph of slight radiopacity of the maxillary sinus – I put maxillary sinus pseudocyst 1. Cross link bonding in PMMA: Benzoyl peroxide 2. Know the HEMA and BIS-GMA 3. Most reason for hyperemia under denture is candidosis 4. The least important factor about root under overdenture is the crown shape-root, perio, caries control are important 5. Primary role of anterior teeth, esthetics- secondary, ver and hor overlap 6. Cheek biting in CD, lack of horizontal overlap 7. Working side condyl adjustment can be done either by protrusion or working side record 8. Minimal reduction for PFM 1.5mm for incisor reduction 2 mm- due to translucency effect of the incisor l edge 9. Coping does not fit in FPD, cut and index of the prosthesis 10.Incorrect procedure before fixing the indirect veneer, etching tooth with HF acid. All others were correct 11.Best long term prognosis of complete edentulous pt considering the ridge resorption , esthetics and comfort is the over denture on implant 12.Be aware of deep of restoration in operative. Exp, class one cavity filling with amalgam on the x-ray, deep is not in to the dentin. Very shallow, retreatment with increasing the deep 13.Reduction for gold crown 1 mm functional cusp, 1.5 mm onlay 14.Be aware of crown to root ratio on the x-ray in treatment planning( day 2) 15.Know the gauges for sprues and needles ( 8- 10- 12 for sprue, 25- 27- 30 for needles) 16.Small under cut of bout tuberositis I CD. What to do? 1- surgically reduction of both 2 reduction of one of them 3- compensate with waxing on the denture 4- soft liner ( Sultan thinks 3) 17.Indirect retainer is not needed in tooth born RPD 18.Know the altered cast technique 19.The most effective feature for crown retention is axial convergent ( 6’) – not the pin or groove or box, however between the last three the box is more retentive than groove and pin respectively 20.Most disadvantage of polyether impression materials is inability to proper disinfection due to dimensional instability in the present of moisture – extreme stiffness and inability to remove form undercut was not in the answer choices 21.Additional radiographic method for supernumerious tooth is CAT Scan or occlusal 22.Be aware of earrings and piercing in e-ray 23.Leas radio resistance tissue is the bone marrow then thyroid gland 24.90 KVP and 15 mA from 8 inches distance and 0.5 second exposure. If the dentist will increase distance , to 16 inches and he/she wants the same density and contrast what would be the exposure time ( 0.5 , 1, 2, 4 second) – Sultan thinks 2 second?? 25.Percentage of reduction to the pt by using the rectangular collimation instead of round ( a question from the space)??? 10 -30 -60 -90 ( sultan guesses 60) 26.Know the chronic periradicular abscess and chronic periradicular periodontitis 27.Best long term graft is auto graft 28.First pulp vitality test is the thermal test not EPT 29.You must know and be able to recognize on the OPG the different type of the bone D1 ….D4 30.Be able to distinguish cervical burn out and class 2 caries. 31.Pt has recession on the facial aspect of tooth 14, no sensitivity and no esthetical concern of the pt, no treatment 32.Scaling and root planning can be either done in control and maintenance stage of therapy 33.After extracting the tooth 14, tooth 15 will move mesially bodily if pt is young, and mesiallly or mesially with mesiolingual rotation if the pt is older, 34.For extracting mesiodent can be done several periapical radio with different angle and occlusal for determination of the location of the tooth. 35.For restoring the class 4 cavity of fracture of upper inisals the most important consideration is the protrusive movment not working side 36.Bilateral band and loop is better befor age of 6-8 and lingual arch is better after age of 8 when the incisor teeth have been erupted due to lingual eruption of these teeth ( lingual arch may interfere with them 37.Intrusion of tooth 8 in the pt 25 years. 1-Splinting 2-reposition 3-extraction 4 endo , Sultan thinks 2 38.Hawley appliance will not treat the single rotated tooth, since it can just tip the tooth using finger spring 39.Lateral upper incisor start calcification after canine do, in 10 month, canine and central approximately 4-5 moth 40.In the pulp will move: 1- apically 2- coronally 3- apically and coronally 4- mesio distal ( Sultan checked it, the answer is 1, floor of the cavity will move apically form Wikipedia 41.Know the PT, PPT and blooding time very good, and there indication. Lots of questions. 42.First manifestation of cavernous sinus thrombosis is the pain in eye ( Sultan thinks ) elevate temperature was not in the answers 43.Radiolucency between the apex of two teeth can be all of the following except: 1- keratocyst 2- lateral perio 3- cementblastoma 4- radicular ( Sultan thinks 3- cementoblastoma can be either completely radiolucence or with the radio opacity in the center 44.Most serious complication of Ludwig’s angina is edema of glottis 45.For esthetics the tip of the platform of implant should be in what level comparing with the gingival margin of adjacent toth: 1 - 2 mm coronal 2 - 5 mm coronal , 3- at the same level , 4 – 1-2 mm lower 46.Never do immediate loading after sinus lift. 47.Bisphosphonate drug effect: inhibit the activity of osteoclasts 48.Different between under filled class 1 composite and occlusal wear in the photo( day 2) check the occlusion , if there is a contact with opposing tooth it is olcclusal wear , if the opposing tooth is under occlusion or it has been missed then under filling 49.Doctor says to the pt that all her/his amalgam restoration should be replaced by composite because amalgam has mercury and the following has toxicity. Answer is Veracity , doctor lied 50.How long wait after bleaching to restore the tooth with composite? 1 - Restore before,2 - immediate after, 3 -next day , 4 - week after ( Sultan thins 4) 51.Know the delay expansion of amalgam and it’s reasons and outcome 52.Know number of hand instruments and surgical forceps 53.With increasing the number of blade in bur- less reduction and more smooth. Remember the amalgam polish bur 54.Memorize the coefficient of thermal expansion, I mean the numbers, from dental decks operative. There are some stupid questions about them like what is the Coefficient thermal expansion of amalgam. Answer is 22-28 ppm/c * 10 55.Try to memorize the percentage , effects, indication and contraindication of NaF, APT and Stannous fluoride 56.NSAID are contraindication with steroids , both will lead to GI problem – day 2 ( Sultan think) 57.The steam under pressure cycle? 1- 100 t and 10 psi , 2- 100 t and 20 psi, 3- 121 t and 10 psi, 4- 121 t and 20 psi. ( in the materials it has been mentioned that 121 t and 15 psi is the standard cycle, however Sultan thinks 4 is correct) 58.The governmental budget for dental treatment is coming from? 1- medicade 2- medicare 3- PPO 4- HMO ( Sultan does not think anything in this situation) 59.Read tuft pharmacology, very helpful 60.My exam on 30.04.2012/01.05.2012 61. ANUG debriment+peroxide ------in HIV use antimicrobial--- no attachment loss 62.Interimplant distance? 3mm, 4mm, 5mm 63.Prevalence of cleft lip/palate in US? 1/700 64.Brown discoloration in the margin of veneer after 1 year? Breakout of coupling agent, discoloration of the amine, reduction of the cement in the margin of the veneer 65.Most cases of oral cancer? Black men 66.Mechanism of digitoxin? Positive inotropic effect 67.Which one is not a symptom of aspirin toxicity? Bradycardia 68.Which LA causes MH? Prilocaine 69.All Symptoms of LA toxicity except? Bradycardia and respiratory depression, nausea and vomiting, dizziness, 70.NO contraindicated in? Sickle cell anemia, Hemophilia, nasal congestion, asthma 71.What happens as a result of long term mercury toxicity? Hair loss, weight gain, vision impairement 72.Low occlusal plane is most likely to cause what? Excessive stress on the ridge (?),… 73.As patient ages: increased show of maxillary incisors, decreased show of mandibular incisors, 74.Statistical test used to compare the mean DMFT among three groups: t test, chi square, anova 75.Know some facts about ginseng 76.Bleaching of the endodontically treated teeth most likely results in: External cervical root resorption, inflammatory root resorption, internal cervical root resorption 77.Rapport= sympathy 78.Several questions about informed consent, modeling, fear and anxiety 79.The most common reason for heart attack in children? Drowning (?), 80.Factor most likely influencing cerebrovascular accident: age, gender, medication, sth else 81.Demineralized freeze dried bone has: Insulin like growth factor, tumor necrotizing factor, bone morphogenic protein, sth else 82.Case: 9 year old boy presents with ant. Maxillary open bite, incisal protrusion and posterior cross bite. He has sickle cell anemia. 83.What is the dental anomaly associated with his systemic condition? 84.He takes folic acid, why? 85.Know the number of forceps 86.Case: Patient with diabetes type2, Hep C, Depression: ASA 3 or 4? 87.Patient with loose partial denture has a 1cm lesion on the lateral border of tongue. The lesion could be: recurrent aphthus, trauma of denture, leukoplakia… (doesn’t say anything about pain or how long it has been there) 88.14 year old girl has asthma, severe crowding of anterior teeth, ectopic eruption of Canines, dental angle classification 1. Has white patches on her oral mucosa: Best treatment for her ectopic canines? The reason for the whit patches? With ANB=6, what is her skeletal classification? Also one question about her facial profile (dolicocephal, convex, nasolabial angle, nasal deviation) 89.1 Liter of 1ppm fluoride equals how many miligrams of fluoride? 90. As2la+dftr+syndromes ----- 3-4 days Pharma ----3 days As2lt oral pathology+ as2lt pt management---- 1 day Clinical cases + pict ----1day x ray pic S: patient presented in our clinic for SCRP A/O:RMH: nochange Bp:130/79 P:77 P: anesthetized w 1 carpule lidocaine w epi 1:100000 infiltrations. LL AND UL Qaudrent, SCRP UR LR SIDE. Patient handles the procedure well. Treatment plan was discussed w the patient and patient agrees on the given treatment. N.V. picture, impression the most common disorder for adult is: 1 depression,2 dementia 2.how much incisor preparation for veneer is: 1 0.3 2 0.5 3 1.0mm 3.patient got trauma on lower lip and make nodule firm is:1 traumatic granuloma 2 mucoele 4.most common sign for inflammation of cavernus sinus is:1 opthalmologia 2 partial loss vision 5.most common cause infection on cavernus sunis in :inflamation in upper teeth 6.if u have patient 1 year old,how can u make check his teeth:put his head on my legs and remain his body with his parents 7.referral pain on superficial temporal area from:?maxillary second premolar 8.which one of them pay more percent on dental treatment in US:1 people 2 third party 3 government insurance 9.which analgesic durg give for pregnant woman?acetopatamin 10.nikolosky sign:pemphigus vulgaris 11.patient take biphosphonate how u can extract his anterior teeth? I answered by amuptution the crowns and leave the roots by flap 12.if we double the distance between the patient and X-ray device,how many time the intensity will decrease:1 2 2 4 3 8 4 16 13.most important factor for post. comp.:technique and case selection 14.one of side effect of N2o:noseia 15.one of side effect use of corticosteroid:??? 16.which test we make for extraction is:INR 17.how the warfarin work as anti-coagulant? on vit k 18.how can u treat patient after u treated and come back to u after 24hrs?1. 2 dose of amoxicillin 2g 2. 1 week for erythromycin with qid 3 No treatment 19.white patiches on cheek when u streched that disappear,what is it:leukoderma 20.prilocaine:metheglobinimia 21.presence of vasoconstrictor in anasthesisa work as less:1 systimic absorption 2 metabolism 22.stridor:laryngospasem 23.side effect of cortico steroid therapy:oestoporosis 24.if u made separation of file while in RCT treatment HOW do u deal with patient?1 offer him referal to specialist 2 record it and continue your treatment 25.if u want to diagnosis TMJ,which method do u use it?1 CT scan 2 panoramaic x-ray 3 MRI 26.which tooth most common accidentally fracture:upper central incisor 27.when u got pregnant woman with hypertension what does that mean:precamlsia 28.patient with down syndrome has:mid face defect 29.patient with down syndrome has:small cranial base bones???